2ちゃんねる スマホ用 ■掲示板に戻る■ 全部 1- 最新50    

■ このスレッドは過去ログ倉庫に格納されています

分からない問題はここに書いてね445

1 :132人目の素数さん:2018/07/16(月) 01:11:06.88 ID:Dv9n2PFO.net
さあ、今日も1日がんばろう★☆

前スレ
分からない問題はここに書いてね444
https://rio2016.5ch.net/test/read.cgi/math/1528207105/

2 :132人目の素数さん:2018/07/16(月) 01:26:40.41 ID:pqCdpuwH.net
削除依頼を出しました

3 :132人目の素数さん:2018/07/16(月) 05:56:27.45 ID:PRetBooJ.net
羽生善治とマキシム・コンツェビッチはどっちの方が頭が良いのでしょうか?

4 :132人目の素数さん:2018/07/16(月) 12:07:03.49 ID:xRPXJSNA.net
Let A ∈ R^(n×n)be a matrix with characteristic polynomial
λ(s)=det⁡(sI-A)=s^n+a_1 s^(n-1)+...+a_(n-1) s+a_n.
Assume that the matrix A can be diagonalized and show that it satisfies
λ(A)=A^n+a_1 A^(n-1)+...+a_(n-1) A+a_n I=0,
Use the result to show that A^k,k≥n, can be rewritten in terms of powers of A of order less than n.

A^n+a_1 A^(n-1)+⋯+a_(n-1) A+a_n I=0を使ってA^k=の式を示すところが分からないので教えてほしいです

5 :132人目の素数さん:2018/07/16(月) 12:38:42.21 ID:1GilTRbF.net
D=d/dx δ=d/dt
x=exp(t)としたとき

x^2*D^2=δ(δ-1)となることがわかりません。教えていただきたいです。

6 :132人目の素数さん:2018/07/16(月) 12:59:39.33 ID:w+2Oxqr6.net
0.577< γ < 0.578
を満たすことを証明せよ。
ただしγはオイラー定数とする。

どなたか解けそうでしょうか?

7 :132人目の素数さん:2018/07/16(月) 15:35:11.76 ID:wED3Ydtq.net
2以上の自然数a,bに対して、2以上の自然数p,qの2変数関数
f(p,q)=|a^p-b^q|
を考える。
min{f(p,q)}とa^(q-1)*b^(p-1)の大小を比較せよ。

8 :132人目の素数さん:2018/07/16(月) 16:25:45.23 ID:wED3Ydtq.net
>>6
計算機使えばラク
もしくは結果にγが現れる積分の被積分関数を評価

9 :132人目の素数さん:2018/07/16(月) 17:47:35.36 ID:F1RiQhE1.net
min{f(p,q)} はp,qを走らせたときのminだよね?a,bのみによる定数だよね?
それとa^(p-1)b^(q-1)というp,qによる変数を比較せよって何?

10 :132人目の素数さん:2018/07/16(月) 19:30:56.96 ID:oeMtiw+0.net
1回のじゃんけんで決まる勝者の数が最大になるのは何人でじゃんけんをしたときか?

11 :132人目の素数さん:2018/07/16(月) 19:50:23.51 ID:tJvLAYbS.net
f(n) = 3*(n/2)*((2/3)^n - 2*(1/3)^n)*n/2 はn=3で最大値1

12 :132人目の素数さん:2018/07/16(月) 19:51:53.36 ID:U/eKKuPN.net
有界じゃなくね
一人だけグーで残りパー出す人数増やせばよくね

13 :132人目の素数さん:2018/07/16(月) 20:45:13.26 ID:h/g2+USd.net
宇宙

14 :132人目の素数さん:2018/07/16(月) 20:52:59.40 ID:E8xJPNMk.net
スマソ、期待値が抜けてた。

1回のじゃんけんで決まる勝者の数の期待値が最大になるのは何人でじゃんけんをしたときか

15 :132人目の素数さん:2018/07/16(月) 20:56:44.25 ID:E8xJPNMk.net
1回だけのじゃんけん、アイコがでてもやり直さない。

アイコだったら勝者の数は0、人数が増えると勝者数も大きくなり得るけどアイコの確率も増える。

16 :132人目の素数さん:2018/07/16(月) 21:22:35.87 ID:wED3Ydtq.net
n以上(n+99)以下のすべての自然数が合成数であるような自然数nが存在することを証明せよ。

17 :132人目の素数さん:2018/07/16(月) 21:38:53.94 ID:SyQcEJj8.net
100!+2

18 :132人目の素数さん:2018/07/16(月) 21:49:10.65 ID:wED3Ydtq.net
放物線y=x^2+cと放物線x=y^2の共有点がちょうど3つ存在する。

(1)cの値を求めよ。
(2)2つの放物線で囲まれる各領域について、その面積の小さい方を求めよ。

19 :132人目の素数さん:2018/07/16(月) 21:49:28.52 ID:wED3Ydtq.net
>>17
素晴らしい

20 :132人目の素数さん:2018/07/16(月) 22:05:51.28 ID:4A8iDNN8.net
>>18
y=x^2 + c & x=y^2 が3つの共有点をもつ
⇔y = y^4 + c が異なる3つの実数解をもつ
⇔-y^4 + y = c が異なる3つの実数解をもつ
不可能??

21 :132人目の素数さん:2018/07/16(月) 22:29:38.72 ID:1hA+i+Ed.net
不等式|x-3|≦1/2(x+a)を満たす整数xがちょうど3個となるようなaの範囲を求めよ。

22 :132人目の素数さん:2018/07/16(月) 22:33:56.44 ID:hfZTE2tY.net
局所単項イデアル整域についてなんですけど、
これをRとすると、局所環なので極大イデアルはただ一つで単項ですよね(Mとします)
するとPIDなのでM=(m)と書けます
PIDの性質からRの既約元はmただ一つとなります。これから任意のr∈Rはr=um^nと書けてM=Rとなってしまう気がするのですが、この議論のどこが正しくないのでしょうか?

23 :132人目の素数さん:2018/07/16(月) 22:36:59.22 ID:hfZTE2tY.net
uは単元です

24 :132人目の素数さん:2018/07/16(月) 22:40:09.68 ID:qddE8O0a.net
>>22
あってるやん。
∀r ∃!n ∃!u uは可逆元かつ r = um^n。
ここまで正しい。
ここから R=M が間違い。

25 :132人目の素数さん:2018/07/16(月) 22:43:59.37 ID:qddE8O0a.net
追記
nは0以上の整数
  ^^^^^^^^

26 :132人目の素数さん:2018/07/17(火) 00:06:31.62 ID:MdAjqePD.net
放物線y=(x-a)^2+bと放物線x=y^2の共有点がちょうど3つ存在する。

(1)a,bの関係式、または値を求めよ。
(2)2つの放物線で囲まれる各領域について、その領域の面積のうち小さい方を求めよ。

27 :132人目の素数さん:2018/07/17(火) 01:15:56.30 ID:GyPvcBOe.net
>>6

マクローリン展開で
1/k - log(k/(k-1)) = 1/k + log(1 -1/k) = - 1/(2・k^2) - 1/(3・k^3) - 1/(4・k^4) - …
k=2〜n でたす。
H_n -1 - log(n) = - (1/2)Σ[k=2,n] 1/(k^2) - (1/3)Σ[k=2,n] 1/(k^3) - (1/4)Σ[k=2,n] 1/(k^4) - …
n→∞ とする。
γ -1 = - (1/2){ζ(2)-1} - (1/3){ζ(3)-1} - (1/4){ζ(4)-1} - …
これに
ζ(2) = ππ/6 = 1.644934
ζ(3) = "Apery" = 1.2020569
ζ(4) = (π^4)/90 = 1.08232323
ζ(6) = (π^6)/945 = 1.017343062
ζ(8) = (π^8)/9450 = 1.0040773562
などを代入して {ζ(8)-1}/8 まで計算すると
γ < 0.5776213723
j>8 のときは、ζ(j) -1 < 1.1/(2^j) だから γ > 0.577
なお γ = 0.5772156649…

>>17
n+99 = 100!+101 は素因数 53611,11588539 をもつ合成数。
(n+100 まで合成数になる)

28 :132人目の素数さん:2018/07/17(火) 01:55:31.24 ID:GyPvcBOe.net
>>16

n = 99! + 2
(99!+101 は素因数 379, 613 をもつ。)

n = 98! + 2
(98!+101 は素因数 62653, 188447201, 2472450529 をもつ。)

n = 97! + 2
(97!+101 は素因数 3331, 1456739 をもつ。)

n = 96! + 2
(96!+97 は素因数 22534022186749 をもつ。96!+101 は素因数 173, 277, 2343172279793 をもつ。

29 :132人目の素数さん:2018/07/17(火) 02:13:03.02 ID:GyPvcBOe.net
>>5


x・D = D・x -1 = δ

xx・DD = x(x・D)D = x(D・x -1)D = (x・D)^2 -(x・D) = δ^2 - δ,

30 :132人目の素数さん:2018/07/17(火) 02:31:23.82 ID:rQ0bE5l8.net
ポール・ディラックさんとヴェルナー・ハイゼンベルクさんはどっちの方が頭が良いですか?

31 :132人目の素数さん:2018/07/17(火) 04:59:46.40 ID:g8VYQL3L.net
>>22
24さんのコメントの通り。
加えて言うと、少なくとも1はMに含まれてないので
明かにM⊂R(真の包含関係)ですよね。

更に、>>22で言われているように任意のr∈Rはr=um^nと書けますが
rがRの単元であることとn=0であることは同値です。
n=0となる元(すなわちRの単元)はM=(m)に含まれません!
以上の推論より「Mの補集合=Rの単元全体」が成り立ちます。

32 :132人目の素数さん:2018/07/17(火) 07:31:41.07 ID:Aegngsr/.net
Z(p)考えたら良いだけ

33 :132人目の素数さん:2018/07/17(火) 08:19:36.33 ID:gBrQ9E6A.net
101!+2の間違いでは

34 :132人目の素数さん:2018/07/17(火) 08:34:26.07 ID:uRYd60Ta.net
>>4
A^n = -(a_1 A^(n-1)+…+a_(n-1) A+a_n I) を繰り返し用いて次数下げ
もしくは多項式 x^k を x^n+a_1 x^(n-1)+…+a_(n-1) x+a_n で割った余りを用いる

35 :132人目の素数さん:2018/07/17(火) 09:28:18.86 ID:EtciPtQN.net
食塩水の問題がさっぱり分かりません
どのように勉強すれば良いのでしょうか

36 :132人目の素数さん:2018/07/17(火) 09:33:05.80 ID:GyPvcBOe.net
>>21
y=|x-3| と y=(x+a)/2 の交点は
 x = (6-a)/3,6+a (a≧-3のとき)
 なし   (a<-3のとき) 
-1≦a<0 のとき 条件を満たす整数xは {3,4,5}

37 :132人目の素数さん:2018/07/17(火) 10:27:10.41 ID:ftYspcax.net
>>35
赤チャート

38 :132人目の素数さん:2018/07/17(火) 12:16:58.29 ID:x5s3oTso.net
コレ教えて
小4の問題
スレチだったらごめんなさい


https://i.imgur.com/9Esv8dC.jpg

39 :132人目の素数さん:2018/07/17(火) 12:31:56.01 ID:R4ojXphj.net
ド・ブロイ波とコンツェビッチ不変量はどっちの方がカッコイイですか?

40 :132人目の素数さん:2018/07/17(火) 12:40:36.60 ID:ZLX8UJxb.net
>>38
千の位までの概数にしたいなら、百の位を四捨五入すればいい
多分どこで四捨五入すればいいかこんがらがって変なことしてるんだろ

41 :132人目の素数さん:2018/07/17(火) 14:38:59.92 ID:eordzLRZ.net
>>34
ありがとうございます
次数下げを繰り返し用いるってどんな感じになりますか?

42 :132人目の素数さん:2018/07/17(火) 17:49:41.76 ID:r0r+jfhX.net
空間に3直線l,m,nがあり、どの2直線も交わらない。
lは原点を通り、mは(1,0,0)を通り、nは(0,1,0)を通る。
l上に点Aをとり、m,nの上でそれぞれ点P,Qを自由に動かしたとき、点Aのとり方によらず△APQ≧1/4であるという。
lとmの距離が取りうる値の範囲を求めよ。

43 :132人目の素数さん:2018/07/17(火) 19:16:25.96 ID:osxpuL7s.net
シンコの盛りは6月と7月。まだいけます。
コノシロの幼魚で4cm、5cmくらいの小さいのをシン
コと言います。
7cm、10cmぐらいはコハダ、13cm前後はナカズミ、1
5cm以上が成魚でコノシロ。
出世魚だから名前が変わります。
養殖のトラフグ食うんなら、天然のカワハギの方が旨
いんじゃねぇかな。ハギは魚の中でも激安、いまだに
数百円で買えるし、釣り好きならタダでいくらでも持
ってこれる。(*近年は養殖ハギが出回っています)
奥様方は、旦那が大漁してたくさん持ち帰っても嫌な
顔しちゃいけません。美味い上に、いくら食べても心
配ないからです。高たんぱく超低脂肪、ビタミンB6とビタミンDが豊富、脂肪酸やコルステロールは少ない
。優良食材です。
マグロ、ありません。ウニ、ありません。コハダ、あ
りません。赤貝、ありません、アナゴ、ありません…
…。かつては、お客さんが怒って帰ってしまったこと
もあったそうです。これは鮨屋ではない、と。しかも
、店があるのは東京の端っこ、川崎市との境目です。
わざわざ出向かなければいけない場所なのです。

44 :132人目の素数さん:2018/07/17(火) 20:59:39.46 ID:qLE42k1Y.net
>>42
l,m,nは平行。その単位方向ベクトルをd、(0,0,1)をzとして
△APQの面積の最小値=1/2 d・z。
∴ dの満たすべき方程式は d・z ≧1/2。
よって(1,0,0)をxとしてd・xの範囲は
-√3/2≦d・x≦√3/2。
ここでl,mの距離 = √(1- (d・x)^2)。 以下ry

45 :132人目の素数さん:2018/07/17(火) 21:45:02.68 ID:avnZehjv.net
ああああああああああああああああああああああああああああああああああああああああああああああああああああああああ

46 :132人目の素数さん:2018/07/17(火) 22:49:18.77 ID:GyPvcBOe.net
>>16

n = Π[k=1,26] p_k + 2
 = 2・3・5・7・11・13・17・19・23・29・31・37・41・43・47・53・59・61・67・71・73・79・83・89・97・101 + 2
 = 232 862 364 358 497 360 900 063 316 880 507 363 072,

47 :132人目の素数さん:2018/07/17(火) 23:05:24.58 ID:r0r+jfhX.net
>>44
l,m,nが平行なのは、平行でないと△APQがいくらでも小さくなるからですか?

48 :132人目の素数さん:2018/07/17(火) 23:15:07.12 ID:GyPvcBOe.net
100! + 2 = 9.33262154… × 10^157     >>17

101! + 2 = 9.42594776… × 10^159     >>33

LCM{2,3,…,101}+ 2
     = 64・81・25・49・11・13・17・19・23・29・31・37・41・43・47・53・59・61・67・71・73・79・83・89・97・101
     = 7 041 757 898 200 960 193 617 914 702 466 542 659 236 802
     = 7.04175790 × 10^42

Π[k=1,26] p_k + 2
     = 2.32862364 × 10^38      >>46

49 :132人目の素数さん:2018/07/17(火) 23:41:47.70 ID:6ClT67yt.net
>>47
まちごうた。平行なのは3本のうちのいずれか2本。
その方向ベクトルがd。

50 :132人目の素数さん:2018/07/18(水) 00:09:31.81 ID:QRQo+1y+.net
>>35
塩を嘗めて頑張る。臥薪嘗塩(がしんしょうえん)

51 :132人目の素数さん:2018/07/18(水) 00:28:24.61 ID:7ZBiGCL7.net
>>41
A^k
= A^(k-n)*A^n
= A^(k-n)*(-a_1 A^(n-1)-…-a_(n-1) A-a_n I)
= -a_1 A^(k-1)-…-a_(n-1) A^(k-n+1)-a_n A^(k-n)
これで k 次式が高々 k-1 次式になった。
k-1>nなら A^(k-1) に対して同様にすれば高々 k-2 次式になって、以下同様。

実用性はあまりないけど、最も思いつきやすい方法だろうと思って一応書いておいた。

52 :132人目の素数さん:2018/07/18(水) 04:21:04.07 ID:emDINoBd.net
(1)A以上のどのような自然数も、ある自然数m,nを用いて5m+17nと表せる。Aを求めよ。
(2)B以上のどのような自然数も、ある自然数l,m,nを用いて5l+17m+3nと表せる。Bを求めよ。

53 :132人目の素数さん:2018/07/18(水) 04:42:59.27 ID:CD1l8p23.net
(1)
85は無理(∵85=5m+17n→85-5mは85未満の正の85の倍数)
86 = 25+51、87 = 70+17、88 = 20+68、89 = 50+34、90 = 5+85
(2)32は無理(∵32=5l+17m+3n→32-5l-3n=は32未満の正の17の倍数→32-5l-3n=17以下ry)
33 = 10+17+6、34 = 5+17+12、35 = 15+17+3

54 :132人目の素数さん:2018/07/18(水) 13:03:18.95 ID:QRQo+1y+.net
>>53

(1) k≧0 に対して
86+5k = 5(5+k) + 17・3,
87+5k = 5(14+k) + 17・1,
88+5k = 5(4+k) + 17・4,
89+5k = 5(10+k) + 17・2,
90+5k = 5(1+k) + 17・5,
と表わせる。

(2) k≧0 に対して
33+3k = 5・2 + 17・1 + 3(2+k),
34+3k = 5・1 + 17・1 + 3(4+k),
35+3k = 5・3 + 17・1 + 3(1+k),
と表わせる。

55 :132人目の素数さん:2018/07/18(水) 13:22:40.96 ID:QRQo+1y+.net
>>42
l,m,n の上でそれぞれ点A,P,Qを自由に動かしたとき、△APQ≧1/4 であるという。
だろうな…

56 :132人目の素数さん:2018/07/18(水) 13:50:23.01 ID:QRQo+1y+.net
>>16

n = Π[k=1,25] p_k + 2
 = 2・3・5・7・11・13・17・19・23・29・31・37・41・43・47・53・59・61・67・71・73・79・83・89・97 + 2
 = 2 305 567 963 945 518 424 753 102 147 331 756 072
  = 2.305568… × 10^36,
(n+99 は 素因数 191,3343 をもつ合成数)

n = Π[k=1,23] p_k * p_25 + 2
 = 2・3・5・7・11・13・17・19・23・29・31・37・41・43・47・53・59・61・67・71・73・79・83・97 + 2
 = 25 905 258 021 859 757 581 495 529 745 300 632
  = 2.5905258… × 10^34,
(n+87 は素因数 179,223,116791 をもつ。n+99 は素因数 613,9979811 をもつ。)

57 :132人目の素数さん:2018/07/18(水) 14:01:33.13 ID:QRQo+1y+.net
>>44 >>47

l,m,nが一葉双曲面(*)上にある場合も、△APQ は最小値を持つんぢゃね?


* 神戸ポートタワーの形

58 :132人目の素数さん:2018/07/18(水) 14:31:43.03 ID:p5f4bXTX.net
l m n上の動点P,Q,Rにおいて△PQRが正の最小値をもつとする。
しかしどの2本も平行でないとする。
lを含む平面α1、α2をとり
Q1 = m∩α1、Q2 = m∩α2、 R1 = n∩α1、 R2 = n∩α2
とおく。
直線Q1R1とlが平行でないならその交点をP1とすれば △P1Q1R1の面積は0で矛盾。∴Q1R1//l。同様にQ2R2//l
よってQ1R1//Q2R2。
∴Q1Q2R1R2は同一平面にある。
∴m,nは同一平面上にある。
仮定よりm//nでないからm,nは交点Xを持つ。
よってP∈lを任意にとりQ=R=Xとすれば△PQR=0。矛盾。

59 :132人目の素数さん:2018/07/18(水) 18:16:01.47 ID:4eALE3K0.net
問39の解き方が解答を読んでも分からないので教えて下さい
https://i.imgur.com/YFt9v18.jpg

60 :132人目の素数さん:2018/07/18(水) 20:25:56.85 ID:5foVOZPS.net
今さっき思ったんですが
超越数^実数乗=自然数 って成り立つケースありますか?

61 :132人目の素数さん:2018/07/18(水) 20:34:50.85 ID:KKcBgmah.net
不等式とかハサミウチの原理を使う問題で、自分で、
どっかから公式とか引っ張り出して評価するのって、
なんか系統だったアプローチはありますか?
この手の問題ですが、使えそうな公式が思い付くか否かが、今のところ非常に運頼みになっています。模試
に出たら必ず後回しです。ただ本番で解くべき問題だ
ったらピンチです。
ピンチをチャンスに変えたいです。アドバイスあればお願いします。

62 :132人目の素数さん:2018/07/18(水) 20:42:39.94 ID:86AumhTR.net
>>60
e^log2=2

63 :132人目の素数さん:2018/07/18(水) 20:58:53.62 ID:IqzEoBDZ.net
食塩水の問題がさっぱり分かりません
食塩/食塩+水=濃度は理解出来るのですが
途中で蒸発させたり、足したり等になるとダメです
そもそも、何をxにすれば良いのかさえ理解出来ませんし解答見ても分かりません

64 :132人目の素数さん:2018/07/18(水) 21:12:51.46 ID:mcY60aIH.net
ここに書いてある問題
10年ぐらい前まではいたkingもういなくなったの?

https://ameblo.jp/contemporaryjp

65 :132人目の素数さん:2018/07/18(水) 21:13:15.35 ID:zkoAX8jV.net
>>63
求めたい量をxとします
問題文に書かれてる内容を式に起こします
方程式を解きます

することはこれだけなんですよね

問題文を描いて頂けたら、具体的に説明しますよ

66 :132人目の素数さん:2018/07/18(水) 22:19:49.12 ID:IqzEoBDZ.net
『20%の食塩水に10グラムの塩を混ぜると24%の食塩水になった。20%の食塩水は何グラムだったか?』って問題です
よろしくお願いします

67 :132人目の素数さん:2018/07/18(水) 22:24:10.55 ID:pZrMe4h6.net
(24 - 20):(100 - 24) = 10:190
190g だな

68 :132人目の素数さん:2018/07/18(水) 22:33:01.32 ID:zkoAX8jV.net
>>67
こうやって横着させようとするからわからなくなるんです
馬鹿正直にやれば絶対解けるということがわからなくなります

>>66
まず、何を求めれば良いのかを確認しましょう

>20%の食塩水は何グラムだったか?

とありますから、20%の食塩水の重さを聞かれているようですね
ですから、20%の食塩水の重さをxグラムとします

次は問題文を式にしましょう
>20%の食塩水に10グラムの塩を混ぜると24%の食塩水になった。

20%の食塩水に塩を混ぜたら、割合が変わって24%になってしまったようですね
%=(塩の重さ)/(食塩水の重さ)ですから、最終的には24%になったということを式にしてみましょう

24/100=(x✖20/100+10)/(x+10)

塩を加える前は
塩の重さ:x✖20/100
食塩水の重さ:x

塩を10g加えると
塩の重さ:x✖20/100+10
食塩水の重さ:x+10

となりますね

これで方程式ができましたから、あとはこれを解くだけですね

69 :132人目の素数さん:2018/07/18(水) 22:40:51.31 ID:pZrMe4h6.net
何で説教されなきゃならんのだろう?
てか、何様?

70 :132人目の素数さん:2018/07/18(水) 22:42:11.61 ID:YDXnqK0x.net
たっぷり改行して長文連ねて挙句解くだけですねっつって人に解かせるくらいだから偉いんじゃないの

71 :132人目の素数さん:2018/07/18(水) 22:42:29.33 ID:zkoAX8jV.net
>>69
ある無矛盾な公理系τの任意のモデルに対してある論理式φが常に真となるならば、τからφがLKにおいて証明可能となることを示せ

わからないなら黙っててくださいねー

72 :132人目の素数さん:2018/07/18(水) 22:43:07.14 ID:zkoAX8jV.net
>>70
質問者の知りたいことは、答えではなく解き方です
そんなこともわからないから、わからない人が増えるんですよ?

73 :132人目の素数さん:2018/07/18(水) 22:50:28.46 ID:pZrMe4h6.net
バカの一つ覚え

74 :132人目の素数さん:2018/07/18(水) 22:53:14.10 ID:pZrMe4h6.net
(20/100)x + 10 = (24/100)(x + 10)
とは立式しないところがイカにも

75 :132人目の素数さん:2018/07/18(水) 22:53:22.61 ID:J4LdIefM.net
数学者とユーチューバーはどっちの方が凄いですか?

76 :132人目の素数さん:2018/07/18(水) 22:55:12.24 ID:zkoAX8jV.net
>>74
あなたは全ての過程すっとばして比で求めようとしてましたね

方程式で解きたいという質問者の要望を無視して
恥ずかしくないんですか?

77 :132人目の素数さん:2018/07/18(水) 23:01:05.88 ID:j7TxdV37.net
やっと回答できるレベルの算数問題が来たんだ
せいぜい得意げにさせて差し上げなさい

78 :132人目の素数さん:2018/07/18(水) 23:03:58.69 ID:J4LdIefM.net
オックスブリッジとハーバードって世界ではどっちの方がブランド力ある?

79 :132人目の素数さん:2018/07/18(水) 23:04:54.87 ID:pZrMe4h6.net
>>76
質問に対する答えではないんだけど?
レス番号つけてないでしょ?

勝手に思い込んで恥ずかしいね

80 :132人目の素数さん:2018/07/18(水) 23:08:31.61 ID:zkoAX8jV.net
>>79
では、何を書いたんですか?
独り言は落書き帳とかツイッターに書いてください?

81 :132人目の素数さん:2018/07/18(水) 23:09:36.94 ID:pZrMe4h6.net
>>80
何で指示されなきゃなんないの?
このスレのタイトル読めよ

82 :132人目の素数さん:2018/07/18(水) 23:12:16.00 ID:zkoAX8jV.net
>>81
わからない問題はここに書いてね、だそうですね
>>67のどこが問題なんですか?

83 :132人目の素数さん:2018/07/18(水) 23:14:46.62 ID:J4LdIefM.net
宇宙とオックスフォード大学の総長はどっちの方が凄いですか?

84 :132人目の素数さん:2018/07/18(水) 23:34:50.22 ID:QRQo+1y+.net
>>64

KingMathematician ◆5lHaaEvFNc (2004/02〜06)
http://science3.5ch.net/test/read.cgi/math/1086433573/

KingMathematician ◆LoZDre77j4i1 (2011/11〜2018/06)
http://kamome.5ch.net/test/read.cgi/math/1306069479/
http://hayabusa6.5ch.net/test/read.cgi/doctor/1365946519/
http://rio2016.5ch.net/test/read.cgi/rikei/1380709667/
http://5ch.pw/5/193/1442983575/

「10年ぐらい前までいたking」かどうか、分からない問題…

85 :132人目の素数さん:2018/07/18(水) 23:50:16.87 ID:QRQo+1y+.net
>>64
KingMathematician ◆LoZDre77j4i1 (2011/11〜現在)
雑談スレ53
http://rio2016.5ch.net/test/read.cgi/math/1442983575/923-

* 召喚呪文は「king氏ね」です。

>>78
 もちろんオックスフォードです。

>>83
 もちろんオックスブリッジ総長です。

86 :132人目の素数さん:2018/07/19(木) 01:38:03.46 ID:nPcIIs+O.net
>>16 を改造

n以上(n+99)以下のすべての自然数が合成数であるような自然数n
のうち最小のものは求まるか?

87 :132人目の素数さん:2018/07/19(木) 01:42:09.17 ID:jz3wZoKD.net
>>86
時間無制限なら求まる。

88 :132人目の素数さん:2018/07/19(木) 01:56:10.22 ID:bPHWIJAD.net
とおもったら簡単に求まった。
n=370262
みたい。

89 :132人目の素数さん:2018/07/19(木) 02:01:30.32 ID:sc+uRxb0.net
数学の問題かはちょっと微妙ですがベルマンフォード法で
ループを節点数引く1回繰り返すと、負閉路が存在しないなら最短路長の更新が止まることの証明って誰か分かりますか?
一応グラフ理論の問題なので、数学の問題かなとは思いますが

90 :132人目の素数さん:2018/07/19(木) 02:26:07.94 ID:X5uFSwny.net
>>89
wikipediaで見たらそりゃそうだと思える。
xが始点として点yのxからの最短経路に含まれる辺の数をl(y)としてl(y)への経路長の更新はl(y)回目の緩和で止まることをl(y)についての帰納法で示す。

(I) l(y) = 1のとき。
一回目の緩和でyへの最短経路が定まるから明らか。
(II) l(y) < kで正しいとしてl(y) = kのとき。
yへの最短経路を
x=z0→z1→…→z(k-1)→zk=y
とする。
帰納法の仮定からz(k-1)への最短経路はk-1回の緩和操作で決定している。
次のk回目の緩和操作でz(k-1)→zk = yが発見されて確定する。
よってyへの最短経路はl(y)回目の緩和操作で確定する。

以上により必要な緩和操作の最大値はmax{l(y)}であるがyへ至る最短経路の点の数は節点数以下であり辺数l(y)は節点数−1であるから主張は示された。

91 :132人目の素数さん:2018/07/19(木) 02:28:52.62 ID:RhgpsBq+.net
R^3内の球面S^2の接束TS^2ってどんなものになりますか?

92 :132人目の素数さん:2018/07/19(木) 04:02:39.77 ID:nPcIIs+O.net
>>88

n = 2・2・3・3・5・11・11・17 + 2 = 3.70262 × 10^5,

n+5 = 479 x 773,
n+11 = 43 x 79 x 109,
n+21 = 379 x 977,
n+27 = 349 x 1061,
n+29 = 19 x 19489,
n+35 = 353 x 1049,
n+39 = 29 x 113 x 113,
n+41 = 367 x 1009,
n+47 = 67 x 5527,
n+51 = 47 x 7879,
n+57 = 547 x 677,
n+65 = 107 x 3461,
n+71 = 37 x 10009,
n+77 = 199 x 1861,
n+81 = 59 x 6277,
n+89 = 179 x 2069,
n+99 = 383 x 967,
その他は{2,3,5,7,11,13,17}のいずれかで割り切れる。

93 :132人目の素数さん:2018/07/19(木) 05:00:03.02 ID:gQgvJqHa.net
>>82
わからないんですか?笑

94 :132人目の素数さん:2018/07/19(木) 06:24:37.69 ID:Nf/HCYYu.net
>>74
10gの食塩を足すのは1回だけなのに、何故左右の式に10があるのですか?(2回足すのですか?)
夜通し考えましたが分からないです

95 :132人目の素数さん:2018/07/19(木) 07:12:26.78 ID:MTskTnwL.net
食塩の方程式で悩むということは厨房かなぁ?

食塩の濃度=食塩の重さ/水と食塩の重さ
が未だ理解できていないか?

書き込みの方程式以下の解説文章を読んでいないか?

問題文に書かれている操作の意味を文字式で書き表す能力が未熟か?

のいづれかかな?

96 :132人目の素数さん:2018/07/19(木) 08:46:26.86 ID:462ScOzz.net
>>94
>>68はなぜ無視するのですか?

97 :132人目の素数さん:2018/07/19(木) 09:59:41.52 ID:Nf/HCYYu.net
>>96
塩の重さ=食塩水✖濃度は何とか分かりますが
では10g足した時の塩の重さって
食塩水xg+10g✖濃度にならないのですか?

98 :132人目の素数さん:2018/07/19(木) 10:07:46.71 ID:462ScOzz.net
>>97
10gは食塩水ではなく、食塩ですよね
つまり、塩の重さです

99 :132人目の素数さん:2018/07/19(木) 10:30:57.09 ID:462ScOzz.net
>>97
ああ、カッコがないから読み違えました


ええっと、まず最初にあった塩の重さは、x✖20/100ですよね
それに後から塩を10足しただけです
x✖20/100+10@

塩を加えた後でも、塩の重さ=食塩水の重さ✖濃度は成り立ちます
(x+10)✖24/100A
となるわけですね

で、@とAが同じになるという式を使うと、>>74になるわけです

100 :132人目の素数さん:2018/07/19(木) 10:40:29.55 ID:RqxjuNi0.net
>>94
混ぜる前と混ぜたあとを比べてる

101 :132人目の素数さん:2018/07/19(木) 13:57:21.27 ID:gh6xjgV8.net
お願いします。

平面α上に正四面体Eがあり、各面をX,Y,Z,Wとする。
最初はXがαに接するように置いてある。(αに接する面をを下の面とする)
下の面の三辺から等確率で一つ選び、その辺を軸として隣の面がαに接するまで四面体を回転させる。Xが再びαに接するまでこの操作を繰り返す。このとき、回転させる回数の期待値を求めよ。

102 :132人目の素数さん:2018/07/19(木) 14:19:17.04 ID:e38r9ly9.net
>>101
確率p_nを漸化式を立てて求める。
次に期待値の定義通りnp_nを1〜∞で足す。
いずれも容易
容易でないと感じるなら今まで何して生きてきたのか疑うレベル

103 :132人目の素数さん:2018/07/19(木) 14:31:27.25 ID:nPcIIs+O.net
死海の塩分濃度は約30%もある。(230〜270g/L、湖底では428g/L)

この近くにユダヤ人国家を建設しようという運動を「塩ニズム」という。

パレスチナ問題の底流にある思想。

104 :132人目の素数さん:2018/07/19(木) 14:54:46.09 ID:nPcIIs+O.net
>>26

放物線 x=yy 上の点(x0,y0)を考える。ただし
 x0 = (y0)^2, y0 < 0,
この点における接線の傾きは 1/(2y0),
放物線 y = (x-a)^2 +b でこれを満たす点を含むものは
 y = y0 + (1/2y0)(x-x0) + (x-x0)^2
よって
 a = x0 - 1/(4y0),
 b = y0 - 1/(16x0),
 x0 = (y0)^2,
 y0 < 0,

105 :132人目の素数さん:2018/07/19(木) 14:58:34.84 ID:hI6vENZY.net
計算問題です

水柱実験を用いて大気圧を測定すると高さ10mになる

水の密度を1g/cm3として大気圧を求めよ

@
1cm^2あたりに1000cm*1cm*1cm*1g/cm^3=1000gの重さがかかることになる
1kg重/cm^2 なので 1000kg重/m^2 が大気圧

A
1m^3の水の重さは1000kg
これを1m^2の床におくと1000kg重/m^2の圧力がかかる
10mの高さならこの10倍で10000kg重/m^2 が大気圧

2つの計算が異なるのは、どこかで計算を間違ってしまったと思うのですが、どこがおかしいのか教えて下さい・・・・

106 :132人目の素数さん:2018/07/19(木) 15:01:36.49 ID:hI6vENZY.net
Aの水1000kgの形状は立方体を想定してます。10コ重ねて10mの高さにする想定です。

107 :132人目の素数さん:2018/07/19(木) 15:04:47.55 ID:ao35wcmm.net
>>105
1kg重/cm^2 なので 1000kg重/m^2 が大気圧 ←ここ間違ってる

108 :132人目の素数さん:2018/07/19(木) 15:46:28.77 ID:nPcIIs+O.net
>>26
(1)
>>104 から x0,y0 を消去すると
  a^3 - b^3 + (ab - 9/16)^2 + (3/4)^3 = 0,

109 :132人目の素数さん:2018/07/19(木) 15:55:34.21 ID:e38r9ly9.net
>>108
これ簡単な設定なのにこんなに汚くなるのか

110 :132人目の素数さん:2018/07/19(木) 15:58:58.00 ID:hI6vENZY.net
>>107
あ、ホントだ・・・失礼しました。

111 :132人目の素数さん:2018/07/19(木) 16:32:53.24 ID:e38r9ly9.net
a_n=(1+(1/n))^n
で与えられる数列{a_n}(n=1,2,...)について、以下の問いに答えよ。

(1){a_n}は単調増加数列であることを示せ。

(2)次の極限が0でない有限の値に収束するとき、pの値を1つ求めよ。
lim[n→∞] {(a_n)-e}*n^p

(3)(2)において、a_nを
(1/n)*{Σ[k=1,...,n] a_k}
に置き換えた場合、(2)の極限が収束するpの値は存在するか。
存在するなら、その値を1つ求めよ。

112 :132人目の素数さん:2018/07/19(木) 16:53:15.58 ID:C3kNyH91.net
問題⇒『1つの長椅子に5人ずつ座ると165人が座れず、7人ずつ座ると35人分の空席が出来ます。
長椅子は全部で何脚あるでしょうか?』
2人分の差が重要な気がしますが、それ以上分かりません
すみませんが教えて下さい、よろしくお願い致します

113 :132人目の素数さん:2018/07/19(木) 17:40:26.47 ID:nPcIIs+O.net
>>111
(1)
{1,1,…,1, 1-1/n}のn個でAM-GMする。
 (n-1)個

{(n-1)/n}^n{(n+1)/n}^n = (1 - 1/nn)^n ≧ 1 - 1/n,

a_{n-1} = {n/(n-1)}^(n-1) < {(n+1)/n}^n = a_n,

(2) p=1
{(a_n)-e}n → -e/2 (n→∞)

114 :132人目の素数さん:2018/07/19(木) 17:49:52.59 ID:nPcIIs+O.net
n>>1 のとき
 (1+1/n)^n - e 〜 -e/(2n),
 (1+1/n)^(n+1/2) - e 〜 e/(12nn),
なので
 e = lim[n→∞] (1+1/n)^(n+1/2)
と定義しよう。

115 :132人目の素数さん:2018/07/19(木) 18:20:42.30 ID:VtcXR+Zy.net
ディラックさんとノイマンさんはどっちの方が頭が良いですか?

116 :学術:2018/07/19(木) 19:00:28.86 ID:OaFnBQ/Q.net
あたまの良さより数学的頭脳の構造世界を描いて戦わねば。なるまい。

117 :132人目の素数さん:2018/07/19(木) 19:20:20.71 ID:VtcXR+Zy.net
ITストラテジストの資格試験に一発合格した15歳と技術士情報工学部門の資格試験に一発合格した15歳はどっちの方が凄いのでしょうか?

118 :132人目の素数さん:2018/07/19(木) 20:03:09.63 ID:rwON1jXu.net
日本人は全員ゴミ

119 :132人目の素数さん:2018/07/19(木) 20:21:29.65 ID:0Zz6e9HU.net
π(n)=n/(1+1/2+……+1/n)
ってどうして?

120 :132人目の素数さん:2018/07/19(木) 22:04:38.96 ID:JQCZSU4L.net
g(x) は x = a で微分可能とする。
f(y) は y = g(a) で微分できないとする。

このとき、

f(g(x)) は x = a で微分できないことを証明せよ。

121 :132人目の素数さん:2018/07/19(木) 22:12:02.06 ID:JQCZSU4L.net
g(x) = x^3 は x = 0 で微分可能である。
f(y) = | y | は y = g(0) = 0 で微分できない。

f(g(x)) = |x^3| は x = 0 で微分可能である。

122 :132人目の素数さん:2018/07/19(木) 22:32:12.67 ID:bv44p76X.net
すいません、こんな算数小2くらいの問題あってるか解答おねがいします。恥ずかしいですが
20枚の山札から初手4枚を引くことから始まるゲームをやっているのですが、20枚の中に入ってる特定のカード1枚を初手に引く確率を解く問題です。

20分の1×4の20分の4を約分して5分の1 A,初手を5回引いたら1回の確率で出る ということであってますか?

123 :132人目の素数さん:2018/07/20(金) 00:08:42.42 ID:ve0N5Gps.net
x~n(10, 6²) であるとき,p(6 < x < 18) をexcelを用いて小数点以下6桁まで求めなさい

124 :132人目の素数さん:2018/07/20(金) 00:20:21.64 ID:smLQGUhz.net
>>108 訂正

 a^3 -b^3 +(3/4)^3 = (ab -9/16)^2,
あるいは
 a^3 +(-b)^3 +(3/4)^3 -3(3/4)a(-b) = (ab +9/16)^2,
左辺を因数分解すれば
 (a-b+3/4) {(a-b+3/4)^2 -(9/4)(a-b+3/4) +3(ab +9/16)} = (ab +9/16)^2,

(略証)
y0 - (1/2) + 1/(4y0) = z0 とおく。
>>104 から
a -b + 3/4 = z0・z0,
ab + 9/16 = (1/y0)(y0 +1/2)^2・z0・z0 = (3/2 + z0)・z0・z0,
これから z0 を消去すると
ab + 9/16 = (3/2)(a-b+3/4) + (a-b+3/4)^(3/2),

やっぱり汚いか…orz

125 :132人目の素数さん:2018/07/20(金) 00:37:43.83 ID:smLQGUhz.net
>>123

μ=10,σ=6 として

P(6<x<18) = {1/√(2πσ^2)} ∫[6,18] exp{-(x-μ)^2 /(2σ^2)} dx

= {1/√(2π)} ∫[(6-μ)/σ,(18-μ)/σ] exp(-tt/2) dt

= 0.6562962427272092171

126 :132人目の素数さん:2018/07/20(金) 01:05:02.42 ID:smLQGUhz.net
>>124
またまた訂正…orz

ab + 9/16 = (3/2)(a-b+3/4) − (a-b+3/4)^(3/2),

127 :132人目の素数さん:2018/07/20(金) 01:27:38.32 ID:smLQGUhz.net
>>103

遠藤周作「死海のほとり」新潮文庫(1983/June)
 424p.724円

http://www.shinchosha.co.jp/book/112318/

死海のほとりの視界は良いか
死海のほとりに歯科医はいるか
死海のほとりで司会をすれば
死海のほとり…

128 :132人目の素数さん:2018/07/20(金) 02:04:44.82 ID:UT/JYK0q.net
https://i.imgur.com/MDydyzC.jpg

この問題の意味がわからず困っています。それぞれがエルミート行列、ユニタリ行列だったと仮定してそれらのジョルダン標準形と変換行列を求める(このプリントで求める行列は4つ)ということでしょうか?
そうだとしても言っていることがわからず前に進めません。
解き方の方針、解答を教えていただきたいです。

129 :132人目の素数さん:2018/07/20(金) 02:17:54.28 ID:H/7fBuKq.net
>>111
どなたかこの問題の(3)は解けませんか?

訂正:
(2)の極限が0でない有限の値に終息する

130 :132人目の素数さん:2018/07/20(金) 02:18:37.50 ID:04ikzHck.net
ユークリッド空間の部分位相空間[0,2π)とS1について
f:[0,2π)→S1がf(x)=(cosx,sinx)で定まっているとき
fは同相写像ではないことを示して下さい

131 :132人目の素数さん:2018/07/20(金) 03:42:36.73 ID:gHpi+Aqt.net
死と数学はどっちの方が偉大ですか?

132 :132人目の素数さん:2018/07/20(金) 03:57:16.11 ID:oxYWBdds.net
s1は区間のはしとはしをつなぎ合わせてできて、逆に言えばs1をちぎると区間になるんだから

133 :132人目の素数さん:2018/07/20(金) 03:57:22.81 ID:H/7fBuKq.net
ある四面体の各頂点から対面に垂線をおろしたとき、その垂線の足は必ず対面の三角形の重心になっているという。
このとき、この四面体は正四面体でたるか。
また、この問いにおいて「重心」を「内心」「外心」「垂心」に変えた場合はどうか。

134 :132人目の素数さん:2018/07/20(金) 06:42:00.16 ID:2kuLSCRJ.net
H(S^1) = Z[t]/(t)、H([0,π)) = Z

135 :132人目の素数さん:2018/07/20(金) 07:24:17.22 ID:63cgSWRN.net
a[n] = a + o(1/n) → s[n]/n = a + o(1/n)

136 :132人目の素数さん:2018/07/20(金) 07:26:06.00 ID:uxL1ZGoy.net
a[n] = a + b/n + o(1/n) → s[n]/n = a + b/n + o(1/n)

137 :132人目の素数さん:2018/07/20(金) 08:11:17.35 ID:52GDdLiO.net
>>112
(165+35)/2=100
100脚

138 :132人目の素数さん:2018/07/20(金) 08:18:47.26 ID:l8ELNvQy.net
楕円の斜線部分の面積を出したいのですが、どのような計算式になるでしょうか。
ぜひ教えてください。
http://o.8ch.net/17krm.png

139 :132人目の素数さん:2018/07/20(金) 08:52:03.11 ID:vRNzEitE.net
gふうぶうほ
http://o.8ch.net/17ksg.png

140 :132人目の素数さん:2018/07/20(金) 08:53:18.05 ID:vRNzEitE.net
ひいjh
http://o.8ch.net/17ksh.png

141 :132人目の素数さん:2018/07/20(金) 09:04:59.24 ID:CWEbGqmy.net
あああ

142 :132人目の素数さん:2018/07/20(金) 10:01:01.41 ID:l8ELNvQy.net
中卒には無理ぽ

143 :132人目の素数さん:2018/07/20(金) 12:07:58.02 ID:f+wQAaIs.net
答えてくれる人はいないのね

144 :132人目の素数さん:2018/07/20(金) 12:32:19.41 ID:H/7fBuKq.net
a,bは互いに素な自然数とする。
数列{a_n}を、
a_1=a
a_2=b
a_(n+2)=p*a_(n+1)+q*a(n)
と定義する。
すべてのnに対してa_nとa_(n+1)が互いに素となるために自然数p,qが満たすべき必要十分条件を求めよ。

145 :132人目の素数さん:2018/07/20(金) 13:53:29.74 ID:L49r4jwp.net
f(x)=(1-x)^-1とおいた時のf(x)の無限次のマクローリン展開の求め方を教えてください。

146 :132人目の素数さん:2018/07/20(金) 15:37:32.02 ID:pAV4EvnZ.net
>>138
円に変換すれば簡単
>>145
等比級数Σx^nの和公式

147 :132人目の素数さん:2018/07/20(金) 15:58:07.47 ID:H/7fBuKq.net
x,yについての連立方程式
(1-s)x-ty=0
tx+(1-s)y=0
が-1≦x≦1かつ-1≦y≦1の解を持つような、実数s,tが満たす条件を求めよ。

148 :132人目の素数さん:2018/07/20(金) 16:02:06.09 ID:UT/JYK0q.net
どなたか
>>128
を……

149 :132人目の素数さん:2018/07/20(金) 16:04:28.96 ID:ZN+Ey0cW.net
>>147
問題おかしくね
それだと常に(0,0)が解になるけど

150 :132人目の素数さん:2018/07/20(金) 16:19:01.23 ID:PUew2ycz.net
>>148
この消しゴム良く消えるよね、俺も使ってる

151 :132人目の素数さん:2018/07/20(金) 17:48:48.02 ID:GloVKkCh.net
>>144
q と pb が互いに素。

152 :132人目の素数さん:2018/07/20(金) 17:54:40.93 ID:GloVKkCh.net
>>148
確かに問題の意味わかんないね。
それぞれがエルミート、ユニタリーと仮定してなんて無理やん。
どっちもエルミートでもユニタリーでもないもん。
何が言いたいのかサッパリ分からん。

153 :132人目の素数さん:2018/07/20(金) 18:03:01.37 ID:kjSK0Ibe.net
この場合のifは副詞節ではなく名詞節を導いていて、Findの目的語になってますよね

エルミートかユニタリーかどうか判別せよ、ということです

ここの人たちって、本当英語わからないんですね

154 :132人目の素数さん:2018/07/20(金) 18:18:01.69 ID:GloVKkCh.net
>>153
文法的には確かにそう見えるんだけど、その場合Findなんて使ってるの見たことないんだよ。
おかしいだろ?判定せよでFind使うか?

155 :132人目の素数さん:2018/07/20(金) 18:58:27.39 ID:mPeAoag9.net
>>146
すみません…わからないです

156 :132人目の素数さん:2018/07/20(金) 19:11:24.45 ID:H/7fBuKq.net
nを自然数とし、x,yについての連立方程式
xy=3*2^n
y=ax+b
を考える。
この連立方程式の解(x,y)がともに自然数で、かつ、解xに対しyが
(y/2x)≦y≦(2x/y)
を満たすという。
実数a,bの満たすべき条件を求めよ。

157 :132人目の素数さん:2018/07/20(金) 19:35:31.62 ID:lU2vyNh+.net
https://i.imgur.com/9WTzRm1.png

lim(x→0) (sinx - xcosx ) / x^3

lim(x→0) (sinx - xcosx ) / x^3

=(sinx/x)*(1/x^2) - (cosx)/x^2

=(1/x^2) - (1-2sin^2(x/2))/x^2

=(1/x^2) - 1/x^2 + 2sin^2(x/2))/x^2

= 2sin^2 (x/2) / x^2

= 2sin^2 (x/2) / 4*(x/2)^2

= 1/2 (x→0)

というふうに変形して、これで答えだ、と思ったのですが、間違っていました。

どこで間違ってしまったのか教えて下さいm(_ _)m

高校生です

158 :132人目の素数さん:2018/07/20(金) 19:49:07.80 ID:ZN+Ey0cW.net
>>157
極限とるときに一部のxだけ極限をとるとか残すとかしては駄目

159 :132人目の素数さん:2018/07/20(金) 19:49:23.36 ID:H/7fBuKq.net
>>157
sinx/x^3=(sinx/x)*(1/x^2)
は合ってるが、この後
(sinx/x)*(1/x^2)=1*(1/x^2)
としてるのが間違い
x→0の極限は式全体に一斉に適用するのに、この時点では(sinx/x)だけに適用してしまってる

160 :132人目の素数さん:2018/07/20(金) 20:17:38.44 ID:lU2vyNh+.net
>>159

ありがとうございます。

1/2 + (sinx/x - 1)/(x^2)を計算しないといけないということですね。

161 :132人目の素数さん:2018/07/20(金) 20:20:54.04 ID:lU2vyNh+.net
結局sinxのマクローリン展開?での大小評価をつかって解くしかないという感じでしょうか?

↑の変形って正攻法で攻略できますかね?できる方いたら教えて下さいm(_ _)m

162 :132人目の素数さん:2018/07/20(金) 20:42:14.34 ID:pHF3+nIQ.net
>>154
152 名前:132人目の素数さん [sage] :2018/07/20(金) 17:54:40.93 ID:GloVKkCh
>>148
確かに問題の意味わかんないね。
それぞれがエルミート、ユニタリーと仮定してなんて無理やん。
どっちもエルミートでもユニタリーでもないもん。
何が言いたいのかサッパリ分からん。


恥ずかしいですね

163 :132人目の素数さん:2018/07/20(金) 21:33:29.12 ID:KfZJfsF5.net
最高裁長官と望月新一氏はどっちの方が賢いですか?

164 :132人目の素数さん:2018/07/20(金) 23:54:59.79 ID:smLQGUhz.net
>>138

楕円を (x/a)^2 + (y/b)^2 = 1 とする。(a>0,b>0)

まづ、円 y = ±√(aa-xx) を考えて [-a,t] で積分する。>>146
∫[-a,t] 2√(aa-xx) dx
= ∫[-a,t] {√(aa-xx) - xx/√(aa-xx) + aa/√(aa-xx)} dx
= t√(aa-tt) + aa・arcsin(t/a),

これをy方向に b/a 倍する。

>>145

f^(n)(x) = n!・(1-x)^(-1-n),
f^(n)(0) = n!
f(x) = 1 +x +x^2 +x^3 +…  >>146

165 :132人目の素数さん:2018/07/21(土) 00:02:04.28 ID:9atlNeyv.net
高校数学ってロピタルの定理使うのNGなんだっけ?

166 :132人目の素数さん:2018/07/21(土) 00:03:49.67 ID:4/chbJgW.net
>>164 訂正

= t√(aa-tt) + aa{arcsin(t/a) + π/2},
   三角形      扇形

167 :132人目の素数さん:2018/07/21(土) 00:13:51.30 ID:4/chbJgW.net
>>150
 渋いね。通だな。

MONO消しゴム (トンボ)
http://www.tombow.com/products/mono/
(wikipediaにもある)

168 :132人目の素数さん:2018/07/21(土) 02:02:41.15 ID:Jd9LDWtI.net
n個のボールをn個の箱にでたらめに入れる。
1つの箱に何個のボールが入っても良いものとし、ボールは必ずいずれかの箱に入るとする。

(1)どのような状態が最も起こりやすいか。

(2)(1)以外の各状態が起こる確率の中央値をP、(1)か起こる確率をQとする。P/Qおよびlim[n→∞] (P/Q)を求めよ。

169 :132人目の素数さん:2018/07/21(土) 07:07:00.41 ID:Jd9LDWtI.net
四面体ABCDの面△ABCの重心をG、△ACDの外心をO、△ADBの内心をI、△BCDの垂心をHとしたとき、四面体GOIHは正四面体であるという。
このとき、四面体ABCDは正四面体であることを示せ。

170 :132人目の素数さん:2018/07/21(土) 07:24:47.37 ID:Jd9LDWtI.net
Oを座標空間の原点とする。
空間で線分OAと線分APが、1≦OA+AP≦2かつ1/2≦OPとなるように自由に動く。

(1)点Pの動きうる領域の体積を求めよ。

(2)さらに下記の条件を満たす点Kが存在するとき、点Pの動きうる領域の体積を求めよ。

[条件]OK≦AP≦PK

171 :132人目の素数さん:2018/07/21(土) 08:21:25.79 ID:pyQzQ2V1.net
>>168
(1) どの状態も確率1/n^n。
(2) P = 1/n^n、Q = 1/n^n
?????

172 :132人目の素数さん:2018/07/21(土) 08:37:55.24 ID:al55q46x.net
>>170
(1) Pの軌跡は半径1/2の球。
(2) AP = rとして半直線POの|PK| = |PO|/2 + rの点Kが常に条件を満たす。
これでいいの?????

173 :132人目の素数さん:2018/07/21(土) 09:40:34.18 ID:Jd9LDWtI.net
原点をOとする座標空間に線分ABがあり、その端点はそれぞれA(0,1,1)、B(1,2,2)である。
また線分OPと線分OQが、OP+OQ≦3を満たし、線分OQ上で線分ABと共有点を持つ(各線分は端点を含むとする)。
以下の問いに答えよ。

(1)折れ線OPQが動くことのできる領域Dの図形は回転体であることを示せ。

(2)↑ABに直交するベクトルで、点(a,1+a,1+a)を始点とするものを1つ求めよ。

(3)Dの体積を求めよ。

174 :132人目の素数さん:2018/07/21(土) 09:56:07.87 ID:6t36aLAR.net
高校生が「志願したい大学」 関東の総合1位は早大

 文系は青学、理系は日大 進学ブランド力調査 高校生新聞

https://headlines.yahoo.co.jp/hl?a=20180719-00010000-koukousei-soci

175 :132人目の素数さん:2018/07/21(土) 10:03:58.07 ID:Jd9LDWtI.net
>>173
これかなりの傑作です。

176 :132人目の素数さん:2018/07/21(土) 10:30:30.54 ID:OnYnrjHv.net
>>175
球に扇型刺さってるんじゃないの?
回転体ちゃうやん?

177 :132人目の素数さん:2018/07/21(土) 10:31:18.00 ID:Jd9LDWtI.net
>>173
(誤)OP+OQ≦3
(正)OP+PQ≦3

178 :132人目の素数さん:2018/07/21(土) 10:37:24.16 ID:OnYnrjHv.net
>>177
それだとQが線分AB上を動く時のOQを焦点とする回転楕円体の合併やろ?回転体なんぞになる?

179 :132人目の素数さん:2018/07/21(土) 10:49:18.28 ID:Jd9LDWtI.net
>>178
じゃあ以下の設問を削除
(1)回転体であることを示せ

180 :132人目の素数さん:2018/07/21(土) 10:56:28.60 ID:OnYnrjHv.net
>>177
OQ+QPじゃないの?
それでもAB軸の回転体に三角形ささってるけど。
線分PQの通過領域?

181 :132人目の素数さん:2018/07/21(土) 11:07:28.61 ID:UAZFfnf/.net
>>179
くだらん

182 :132人目の素数さん:2018/07/21(土) 11:08:14.44 ID:sJGqV5r9.net
用意している解答例を見せろ
そうすれば問題文と解答のどこに不備があるかはっきりする

183 :132人目の素数さん:2018/07/21(土) 12:10:24.51 ID:z0zVJ4ZE.net
まぁたぶん平面OABを含む平面上中心O、半径3、中心角が∠AOBの扇型から△OABを抜いたものと中心A、半径3-√2、中心角ryの扇型の合併の回転体の体積もとめさせたいんだろうなぁ?
計算煩雑でうまい回避法もなさそう。
ちょっとやる気起きない。

184 :132人目の素数さん:2018/07/21(土) 12:29:02.12 ID:Jd9LDWtI.net
一辺の長さが1の立方体ABCD-EFGHの辺AB、AD、辺CG上にそれぞれ点P,Q,Rをとり、一辺の長さが1の正三角形PQRをつくる。
△PQRを直線PRのまわりに一回転させてできる立体をKとするとき、Kの内部で立方体の内部でもある領域の体積を求めよ。

185 :132人目の素数さん:2018/07/21(土) 13:33:41.87 ID:OnYnrjHv.net
>>183
BからCまでですでに長さ1なのにAB上とCG上に端点を持つ長さ1の線分ってBCしかないやん。

186 :132人目の素数さん:2018/07/21(土) 14:17:33.36 ID:OnYnrjHv.net
RはAE上かなぁ?だとするとまたそれはそれで問題発生する希ガス。
領域は通過領域とPRの垂直二等分面の共通部分を底面とし、PとRを頂点とする錐だけど底面積逆三角関数使わんと表示できん希ガス。

187 :132人目の素数さん:2018/07/21(土) 16:23:58.77 ID:4/chbJgW.net
〔問題2896〕

∫[0,π] 1/√{1+sin(x)} dx の値を求めよ。

http://suseum.jp/gq/question/2896
すうじあむ

188 :132人目の素数さん:2018/07/21(土) 21:26:43.46 ID:sKvdky7h.net
(a+b)2(a-b)2(a2+b2)2
展開したいんですが…頭がこんがらがってしまう。出来れば過程とかもわかりやすく…お願いします

189 :132人目の素数さん:2018/07/21(土) 21:38:59.99 ID:9atlNeyv.net
>>188
a^2をa2と表記してるとエスパーして解答します。
記号は正確にお願いします。

(a+b)^2(a-b)^2(a^2+b^2)^2
={(a+b)(a-b)}^2(a^2+b^2)^2
=(a^2-b^2)^2(a^2+b^2)^2
={(a^2-b^2)(a^2+b^2)}^2
=(a^4-b^4)^2
=a^8-2a^4b^4+b^8

190 :132人目の素数さん:2018/07/21(土) 22:19:38.15 ID:Jd9LDWtI.net
四面体ABCDの面である△ABCは、CAを斜辺とするAB=1,AC=tの直角三角形である。
残りの3つの面が、それぞれ鋭角三角形、直角三角形、鈍角三角形であるようなtの条件を求めよ。

191 :132人目の素数さん:2018/07/21(土) 22:37:37.12 ID:Jd9LDWtI.net
すべての面が、三辺の長さがそれぞれ4,5,6である鋭角三角形からなる四面体OABCを考える。
点Oをxyz空間の原点、点AをA(4,0,0)とする。また点BをB(b1,b2,0)とし、OB=5かつb1>0かつb2>0とおく。また点Cのz座標は正である。
この四面体を平面x+y+z=kで切った切り口が多角形となるとき、断面積S(k)をkで表せ。

192 :132人目の素数さん:2018/07/21(土) 22:42:49.66 ID:mYp2AaXl.net
>>190
もはや数学の問題の体を成してないな。
この文章で何を伝えたいのだろう?

193 :132人目の素数さん:2018/07/21(土) 22:45:36.09 ID:Jd9LDWtI.net
凸四角形Sには内接円が存在し、その周長はLである。

Sを何本かの直線で分割し、分割された図形全てが三角形となるようにする。
また、それらの三角形すべてに内接円を描き、それらの周長の和をMとする(すなわち、Mは分割の仕方により異なる)。

Mを最大にするような分割の仕方を考え、その場合のMをM'とするとき、LとM'の大小を比較せよ。

194 :132人目の素数さん:2018/07/21(土) 22:54:51.35 ID:Jd9LDWtI.net
nを2以上の整数とする。
3辺の長さがそれぞれn,n+1,n+2である三角形をTnとし、すべての面がTnである等面四面体Vnを考える。
Vnの体積をK(n)と、Vnの内接球の体積をL(n)をそれぞれ求めよ。

195 :132人目の素数さん:2018/07/21(土) 23:01:14.35 ID:Jd9LDWtI.net
四面体Vの表面上または内部に点Pをとる。
Pを通る直線のうち、その直線の周りにVを一回転させてできる立体の体積を最小にするものをlpと名付ける(複数存在する場合はいずれの直線もlpとしてよい)。
また、Vをlpの周りに一回転させてできる立体の体積をK(lp)とする。
点Pを色々動かすとき、K(lp)が最小になるPの位置はどこか。

196 :132人目の素数さん:2018/07/21(土) 23:07:35.16 ID:Y/dsb3jV.net
>>193
直線で分割すんの?対角線ではなく?

197 :132人目の素数さん:2018/07/21(土) 23:07:46.89 ID:L1OiQ9i0.net
ID:Jd9LDWtI
いつも大量に問題だけ書いてくけどさ
ちゃんと答えはあるんかい?
ただ駒を適当に並べただけなのに「詰将棋です」と言ってるようなものもたまに混じってるようだけど

198 :132人目の素数さん:2018/07/21(土) 23:16:21.40 ID:ZAC5rhyg.net
わからないんですね

199 :132人目の素数さん:2018/07/22(日) 00:00:00.43 ID:OxLipG16.net
今日は四面体の神秘に挑みました
みなさんのレヴェルに合わせて、
明日はもう少し易しくします

日本の数学力低下、著しい

200 :132人目の素数さん:2018/07/22(日) 00:01:47.46 ID:7cGty+2f.net
>>190
残りの3つの面の形は、tの値 と 頂点Dの位置 の両方に依存する。
一方だけでは決まらない。

>>198
分からない   >>192

201 :132人目の素数さん:2018/07/22(日) 00:04:04.49 ID:V4VVrTcL.net
分からない問題を書くスレなんだから答えがあるわけないでしょ

202 :132人目の素数さん:2018/07/22(日) 00:05:05.23 ID:V4VVrTcL.net
訂正
あるわけない→用意されてるわけない

203 :132人目の素数さん:2018/07/22(日) 00:17:52.09 ID:kW/emsSA.net
わからないんですね

204 :132人目の素数さん:2018/07/22(日) 00:39:13.59 ID:7cGty+2f.net
>>194

Vnの4頂点を
 A (p,q,r)
 B (p,-q,-r)
 C (-p,q,-r)
 D (-p,-q,r)
としてみる。
 2√(pp+qq) = AD = BC = n,
 2√(qq+rr) = AB = CD = n+1,
 2√(rr+pp) = AC = BD = n+2,
から
 p(n) = √{(nn+2n+3)/8},
 q(n) = √{(nn-2n-3)/8},
 r(n) = √{(nn+6n+5)/8},
となる。

K(n) = (1/3)(2p)(2q)(2r) = (8/3)pqr,

205 :132人目の素数さん:2018/07/22(日) 04:06:04.26 ID:7cGty+2f.net
>>194

>>204 より、平面BCDの式は
 x/p + y/q + z/r = -1,
O(0,0,0) から下した垂線OHの長さは
 OH = 1/√(1/pp+1/qq+1/rr)
内接球の半径Rは
 R = OH = 1/√(1/pp+1/qq+1/rr)
なお、垂線の足は(-(1/p)RR,-(1/q)RR,-(1/r)RR)
 L(n) = (4π/3)R^3

206 :132人目の素数さん:2018/07/22(日) 05:50:55.24 ID:AMei3CJ/.net
>>201
もちろん答えがない問題書くのは構わないけど、答えが出ない可能性があるならそれは明示すべきやろ?
丸一日考えて答え出ないかもしれないって後で分かった時どんな気分になるか、考えて見たらいい。

207 :132人目の素数さん:2018/07/22(日) 06:49:52.28 ID:zZSTWGNn.net
>>195 なんて明示的な答えでると思えないよね。

208 :132人目の素数さん:2018/07/22(日) 09:40:30.73 ID:9ULrfEGX.net
>>197
>ただ駒を適当に並べただけなのに「詰将棋です」と言ってるようなものもたまに混じってるようだけど
ホボそれ

209 :132人目の素数さん:2018/07/22(日) 11:49:29.99 ID:y7v+08YF.net
フェルマーの最終定理について調べ物をしていまず。

以下の2サイトで「証明した!」って主張されていますが、
それぞれ具体的にどこが間違っているのか分かりません。
教えて頂けませんか?

ttp://d.hatena.ne.jp/keptan125/20170912/1505227676
ttp://fermats-last-theorem.blog.jp/archives/4436874.html

210 :132人目の素数さん:2018/07/22(日) 11:52:32.51 ID:e1/MQ8xm.net
自力で解決しろよ

211 :132人目の素数さん:2018/07/22(日) 11:56:09.09 ID:kW/emsSA.net
>>209
とりあえず上は、三角形にならないときが考えられてないんじゃないですか?

1,1,100とか

212 :132人目の素数さん:2018/07/22(日) 12:00:36.63 ID:y7v+08YF.net
>>211
フェルマーの最終定理(x^n+y^n=z^n | n>=3)が成り立つと仮定したなら、
x,y < z < x+y
であるので、(x,y,z)を3辺とする三角形は成り立ちます。

213 :132人目の素数さん:2018/07/22(日) 13:42:11.00 ID:OxLipG16.net
四面体Vの表面上または内部に点Pをとる。
さらにPを通る直線を1つとり、その直線を軸としてVを一回転させてできる立体を考える(すなわち、その立体はPおよび直線の取り方により異なる)。
このようにして出来る立体のうちで、その体積が最小になるものをKとおく。立体Kの回転の軸はVの重心を通ることを示せ。

214 :132人目の素数さん:2018/07/22(日) 17:00:05.71 ID:bJVmuZsS.net
ノルム空間(X,‖・‖)上の点列{x_n}について、{x_n}がコーシー列ならば階差列{x_n+1-x_n}が0に収束することを示しなさい。

215 :132人目の素数さん:2018/07/22(日) 17:19:38.36 ID:kW/emsSA.net
コーシー列の定義より明らか

216 :132人目の素数さん:2018/07/22(日) 18:11:09.79 ID:XHMrpicM.net
>>213
こんなん成り立つ?
例えばパラメータa.eで4点を
(0,0,e),(0,0,-e),(2,a,0),(2,-a,0)
として、0<e<<1,a>>0のとき、うすい三角錐ふたつ貼り合わせた形で、おそらく回転体の体積を最小にする軸はz軸に平行になると思うし、軸のy座標は0になると思うけど、
(t,0,0)を通るz軸に平行な直線は台形となんか中に凸な曲線で囲まれたとんがった図形の回転体になる。
この回転体の体積求めて、それの最小値がaの値にかかわらずつねにt=1になるなんて事おこる?
ホントに持ってる答えあってんの?
そもそも四面体を回転させたときの回転体の体積なんてこんな簡単な例でも各種パラメータが恐ろしくかかわってくるのに、それの最小値が常に “重心通るとき” になんてなるなんて起こる?

217 :132人目の素数さん:2018/07/22(日) 22:12:30.57 ID:9ULrfEGX.net
>>209
間違いを指摘するのは大変でおおよそ無駄
自力で解決せねばね

218 :132人目の素数さん:2018/07/22(日) 23:00:02.63 ID:OxLipG16.net
I_1=∫[-∞,∞] exp(-x^2) dx
I_2=∫[-∞,∞] {exp(-x^2)}{x^2/(1+x^2)} dx
に対して、比(I_2)/(I_1)の値を小数点以下第1位まで求めよ。第2位以降の桁は切り捨てよ。

219 :132人目の素数さん:2018/07/22(日) 23:06:34.98 ID:OxLipG16.net
>>216
反例を与えれば良いのです

220 :132人目の素数さん:2018/07/22(日) 23:50:45.13 ID:XHMrpicM.net
>>219
だから>>216が反例だって。aが十分大きければ軸を(1,0,0)から右にずらしていく方向への変分が負になるやん。

221 :132人目の素数さん:2018/07/23(月) 00:07:18.45 ID:4rZpnird.net
>>218

I_1 = √π = 1.772453850905516

(これは、数学者にとっては 2x2=4 と同じぐらい明白なことである。---ケルビン卿)

I_2 = I_1 - ∫[-∞,∞] exp(-xx) /(1+xx) dx
 = 1.772453850905516 - 1.343293421646735
 = 0.42916042925878

∴ I_2 / I_1 = 0.42916042925878 / 1.772453850905516
 = 0.2421278438586879

222 :132人目の素数さん:2018/07/23(月) 07:24:05.18 ID:yeQRS8kt.net
aを-1≦a≦1である実数とする。
x,yについての連立方程式
sx-(1-t)y=a
(1-t)x+sy=√(1-a^2)
が|x|<1かつ|y|<1の実数解を持つように、実数s,tが満たすべき条件を求めよ。

223 :132人目の素数さん:2018/07/23(月) 10:15:23.88 ID:w0WUKAOq.net
s√(1-a^2) - t a ≠ 0 または s a + (1-t)√(1-a^2) ≠ 0

224 :132人目の素数さん:2018/07/23(月) 13:44:32.34 ID:BjQJiFib.net
奇数点が2nの時n筆書きできる証明ってどうやるの?

225 :132人目の素数さん:2018/07/23(月) 13:50:22.60 ID:4rZpnird.net
>>194

qr = (1/8)√{(8pp-18)(8pp-2)},pp = (nn+2n+3)/8,
より
K(n) = (8/3)pqr
  = (1/3)p√{(8pp-18)(8pp-2)}
  = (1/3)(n+1)√{(nn+2n-15)(nn+2n+3)/8},    

>>205 から
R = OH = p√{(4pp-9)/3(4pp-3)}
 = √{(nn+2n-15)(nn+2n+3)/24(nn+2n-3)},

或いは、
3辺の長さが n,n+1,n+2 である三角形Tn の面積は、ヘロンの公式から
Tn = (1/4)(n+1)√{3(nn+2n-3)},

R = OH = (3/4)K(n)/Tn
 = √{(nn+2n-15)(nn+2n+3)/24(nn+2n-3)},

226 :132人目の素数さん:2018/07/23(月) 14:39:41.08 ID:yeQRS8kt.net
I_n=∫[0→1] 1/{(1+x^2)^(2n)} (n=1,2,...)
とおく。次の命題の真偽を判定せよ。

命題:どのnに対してもある有理数p_nとq_nが存在して、I_n=p_n*π+q_nと表せる。

227 :132人目の素数さん:2018/07/23(月) 16:11:29.41 ID:4rZpnird.net
>>226

J_m = ∫[0,1] 1/(1+xx)^m dx とおくと I_n = J_{2n},

2m(J_{m+1} - J_m) = -2m∫[0,1] xx/(1+xx)^{m+1} dx
  = [ x/(1+xx)^m ](x=0,1) - J_m
  = 1/2^m - J_m,

J_{m+1} = {(2m-1)/2m}J_m + 1/(2m・2^m),
また J_0 =1,J_1=π/4 ゆえ真


J_2 = (π+2)/8,
J_3 = (3π+8)/32,
J_4 = (15π+44)/192,

228 :132人目の素数さん:2018/07/23(月) 17:37:06.67 ID:oj9m1qnv.net
cosh(ax)=bx
でxの求め方が分かりません。
教えて頂けないでしょうか。

229 :132人目の素数さん:2018/07/23(月) 18:57:51.55 ID:a5jAmAvz.net
>>224
即興だけど、大体こんな感じかと。

二つの奇数点選び、その間をつなぐ道をひとつ選ぶ。
その道に現れた辺をグラフから除く。
残ったグラフは非連結かもしれないが、それぞれの連結成分は偶数個の奇数点を含む。
この操作によって奇数点は2個減るので、あとは帰納法。

230 :132人目の素数さん:2018/07/23(月) 20:32:22.50 ID:yeQRS8kt.net
nを自然数とする。
曲線Cn: x^(2n)+y^(2n)=1 上の有理点は、nに関係なく、(1,0)、(0,1)、(-1,0)、(0,-1)の少なくとも4つが存在する。
では、これら以外の有理点が存在するかどうかを考察しよう。

(1)n=1のとき、上記以外の有理点が存在する。その例を1例挙げよ。

(2)n=2のとき、曲線Cn上には上記4つの有理点しか存在しないことを示せ。

(3)一般のnに対してはどうか。

231 :132人目の素数さん:2018/07/23(月) 20:43:18.25 ID:yeQRS8kt.net
3辺の長さがそれぞれ7,8,9である三角形をTとし、すべての面がTからなる等面四面体Vtがxyz空間の平面z=0上に置かれている。
Vtの4頂点をO,A,B,Cとすると、Oは空間の原点と一致し、A(7,0,0)である。
またB(s,t,0)とおくと、OB=8かつt>0である。

(1)Bの座標およびCの座標を求めよ。

(2)Vtを平面x+y+z=kによって切り分けた2つの部分の体積が等しいとき、その切断面の面積を求めよ。

232 :132人目の素数さん:2018/07/23(月) 20:45:50.73 ID:yeQRS8kt.net
>>231
書き忘れ
点Cのz座標は正

233 :132人目の素数さん:2018/07/23(月) 20:54:24.86 ID:NGPxwaeJ.net
NASAの研究者になるのとオックスフォード大学の数学教授になるのはどっちの方が難しい?

234 :132人目の素数さん:2018/07/23(月) 21:27:25.38 ID:UvK/YGIW.net
「任意の多様体Mに対してその接束TMは向き付可能であることを示せ」
とっつきがなくてどう解けばいいのか分かりません

235 :132人目の素数さん:2018/07/23(月) 21:48:23.13 ID:CO3G9BSd.net
>>234
勘で座標変換のヤコビやーんが全て正のアトラスが取れるのでは?

236 :132人目の素数さん:2018/07/23(月) 23:27:34.43 ID:4rZpnird.net
>>227

J_m = {(2m-3)!!/4(2m-2)!!}π + (有理数)

>>230
(1)
(x,y) = ( (1-tt)/(1+tt),2t/(1+tt) ),tは有理数。

(2) (3)
 フェルマーの最終予想(現・ワイルズの定理)と同値?

237 :132人目の素数さん:2018/07/24(火) 06:32:12.64 ID:0QsolIAt.net
1から6までの目が等確率で出るサイコロを4回振り、n回目に振って出た目の数をa_nとする。
このときAB=a_1、BC=a_2、CD=a_3、DA=a_4となり、かつ、外接円を持つような□ABCDができる確率を求めよ。

238 :132人目の素数さん:2018/07/24(火) 06:32:35.64 ID:0QsolIAt.net
>>237
小粋な問題でよく出来ていると思います。

239 :132人目の素数さん:2018/07/24(火) 07:15:16.26 ID:bmjGlIcJ.net
1156/1296かな?

240 :132人目の素数さん:2018/07/24(火) 07:35:11.42 ID:bmjGlIcJ.net
(∃ x θ)
x^2 = a^2 + b^2 -2ab cosθ
x^2 = c^2 + d^2 +2cd cosθ
|cosθ| < 1

-1<((a^2+b^2)-(c^2+d^2))/(2ab+2cd)<1

a<b+c+d ∧ b<c+d+a ∧ c<d+a+b ∧ d<a+b+c
余事象は
(X):a≧b+c+d or …
であるがこの4つの条件は排反であり
n(3≧b+c+d) = 1、n(3≧b+c+d) = 4、n(3≧b+c+d) = 10、n(3≧b+c+d) = 20
から(X)の場合の数は140。
よって求める場合の数は1156。

241 :132人目の素数さん:2018/07/24(火) 07:55:51.36 ID:0QsolIAt.net
>>240
どうしてそんなに解答が早いのか教えてください
予めソフトであたりをつけたりしていますか?

242 :132人目の素数さん:2018/07/24(火) 08:01:33.33 ID:bmjGlIcJ.net
>>241
a,b,c,dの条件出すまでは円に内接する四角形の話なので慣れてればそんなに難しくない。
そこからはお察しの通りソフトで答えだして、ああ、余事象数えた方が早そうだとあたりをつけますた。

243 :132人目の素数さん:2018/07/24(火) 10:08:38.72 ID:zLrO4Ocl.net
n(6≧a≧b+c+d) は x = a+1-(b+c+d)、y=8-(a+1)とおいて
n( b + c + d + x + y = 8) = C[7,4]
の方が良かった。

244 :132人目の素数さん:2018/07/24(火) 14:16:46.21 ID:PyFD9YFX.net
>>235
とても勘では解けませんでした
どうすればいいでしょうか

245 :132人目の素数さん:2018/07/24(火) 15:22:34.49 ID:ErzIUWjY.net
u=u(x,y,z)の各成分が
u(x,y,z)=(p(x,y,z),q(x,y,z),r(x,y,w)) で定義されているとし
R^3上の任意の点において rot uが
rot u=(0,0,0) を満たすとする。
また原点をOとし、xyz空間内の3点、A,B,Cの座標を
(x,0,0) , (x,y,0) , (x,y,z) とする。
曲線Cを点Oから線分OA,AB,BCを経てCに至る曲線とする。曲線Cの点Oから点Cに向かう経路による線積分で3次元スカラー場 F=f(x,y,z)を
f(x,y,z)=∫c u・ds で定義する。
(1)
スカラー場fに対して
f=∫[x_0]p(ξ,0,0)dξ+∫[y_0]q(x,η,0)dη+∫[z_0]r(x,y,ζ)dζ
が成り立つことを示せ。
(2)
ベクトル場u およびスカラー場f に対して
u=grad f
が成り立つことを示せ。
分かりにくいかもしれませんが、よろしくお願いしますm(_ _)m

246 :132人目の素数さん:2018/07/24(火) 20:38:05.63 ID:IHPWzHbl.net
>>228はどうでしょうか?

分からなくて困ってます。

どうかお助け下さい。

247 :132人目の素数さん:2018/07/24(火) 21:17:56.28 ID:rI+Bo7Br.net
>>236
これも
Two urns contain the same total numbers of balls, some blacks and some whites in each. From each urn are drawn n ( > 3) balls with replacement.
Find the number of drawings and the composition of the two urns so that the probability that all white balls are drawn from the first urn is equal to the probability that the drawing from the second is either all whites or all blacks.

248 :132人目の素数さん:2018/07/24(火) 22:41:35.80 ID:5xWSgyrB.net
>>247
Say that each urn contains m balls.

If the first urn contains c whites, then the probability that all white
balls are drawn from the first urn is c^n / m^n.

If the second urn contains a whites and b (=m-a) blacks, then the
probability that the drawing from the second is either all whites or all
blacks is (a^n + b^n) / m^n.

c^n / m^n = (a^n + b^n) / m^n
c^n = a^n + b^n

a > 0, b > 0, c > 0 ("some blacks and some whites in each"), so by FLT
there are no solutions.

249 :132人目の素数さん:2018/07/24(火) 22:54:46.80 ID:V4zvMXn+.net
>>244
C2級以上じゃないとつかえないけど。
MをC2級の可微分多様体で(Uα, fαβ)をアトラスとする。
(ただしfαβ: Uα(の一部)→Uβとする。以下 “の一部” はことわらない)
fαの引き起こすUα×R^n → Uβ×R^n を gαβとすればJ(gαβ)は左上隅と右下隅にJ(fαβ)がきて右上隅は0になるので det J(gαβ) > 0となり TM は向き付け可能とわかる。

250 :132人目の素数さん:2018/07/24(火) 23:03:01.48 ID:uw1A6Pfh.net
>>245
曲線? Aを
(0,0,0) → (0,y,0) → (0,y,z) → (x,y,z)、
曲線? Bを
(,0,0) → (x,0,0) → (x,0,z) → (x,y,z)、
で定めるとき
f(x,y,z)=∫A u・ds=∫B u・ds=∫C u・ds …(※)
を示せば grad f = uがわかる。

251 :132人目の素数さん:2018/07/25(水) 01:06:31.97 ID:8Fs+kj1b.net
関数項級数(n!/n^n)x^n (-e<x<e)が一様収束するか判定せよという問題です
優級数の方法で判定すると思うのですが上手くいかないので助けて下さい

252 :132人目の素数さん:2018/07/25(水) 09:54:24.77 ID:4IauL9c2.net
stirlingの公式じゃね?

253 :132人目の素数さん:2018/07/25(水) 10:58:27.80 ID:RVQLuNfx.net
数列{a_n}を、
a_1=1/2
a_(n+1)=(a_n)/{1+(a_n)^2}
により定める。

(1)lim[n→∞] a_n = 0 を示せ。

(2)lim[n→∞] n*(a_n) を求めよ。

254 :132人目の素数さん:2018/07/25(水) 11:47:10.47 ID:aSG76bcm.net
>>253
全然収束する気配ないけど。
ホントに収束するん?
Prelude> let a = iterate (¥x->x/(x^2+1)) (0.5)
Prelude> let b = zipWith (*) [1..] a
Prelude> map (b!!) [10^x|x<-[1..6]]
[2.201086145991507,7.036330812846756,22.342978061047436,70.70302944964894,223.60373468043463,707.1056090035287]

255 :132人目の素数さん:2018/07/25(水) 12:00:42.63 ID:naM52pOX.net
グラフ見たら収束しそうだけど
http://www.wolframalpha.com/input/?i=plot+y%3Dx%2F(1%2Bx%5E2),y%3Dx

256 :132人目の素数さん:2018/07/25(水) 12:06:30.75 ID:aSG76bcm.net
いや、anはもちろん収束する。
n*(a_n)のほう。
グラフ見ればわかるように収束はメチャメチャおそい。
y=f(x)、f(0)=0、でsup{|f’X(x)|} = r <1のときちょうど一位で収束する。
f’(0) = 1のケースでは一位で収束することはほとんど期待できない。
数値実験でも収束する気配ないし。

257 :132人目の素数さん:2018/07/25(水) 12:08:35.63 ID:aSG76bcm.net
ああ、∞が答えなんかな?まぁそれなら問題として成立してるな。
しょうもないけど。

258 :132人目の素数さん:2018/07/25(水) 12:33:39.82 ID:aSG76bcm.net
f(x) = x/(1+x^2)、b_n = (log n )/n とおくと
f(b_n) > b_(n+1) (n ≧9) と f’(x)>0 (x∈(0,1)) により
b_n < a_n → b_(n+1) < a_(n+1)。
また
a_21 = 0.14540247174206863
b_21 = 0.14497725893921062
により
a_n > b_n (n≧21)。
∴ lim n b_n = ∞ により lim n a_n = ∞。

259 :132人目の素数さん:2018/07/25(水) 13:22:00.75 ID:RVQLuNfx.net
>>254
収束します。

260 :132人目の素数さん:2018/07/25(水) 13:26:34.57 ID:RVQLuNfx.net
訂正
(誤)a_(n+1)=(a_n)/{1+(a_n)^2} 
(正)a_(n+1)=(a_n)/{1+(a_n)}^2

261 :132人目の素数さん:2018/07/25(水) 13:26:57.62 ID:RVQLuNfx.net
>>254
すいません問題を写し間違えました

262 :132人目の素数さん:2018/07/25(水) 13:32:42.04 ID:naM52pOX.net
わかっている問題を書き込みたいならよそへ行け

263 :132人目の素数さん:2018/07/25(水) 13:40:24.39 ID:RVQLuNfx.net
>>262
(2)の収束の証明が出きなかったんです。極限値は分かりましたけど、式変形が分かりません

264 :132人目の素数さん:2018/07/25(水) 15:18:04.37 ID:FxD1zx7b.net
杉浦光夫著『解析入門I』を読んでいます。

解析入門Iのp.139定理6.10の証明ですが、

「従って f^(-1) は y_0 で連続である。」

とありますが、なぜ、そう言えるのでしょうか?

265 :132人目の素数さん:2018/07/25(水) 15:39:06.52 ID:9UVBQKrl.net
>>253
 f(x) = x・e^{-1/(2xx)} の零点をニュートン法で求めると

 a_{n+1} = a_n - f(a_n)/f '(a_n)

   = a_n / {1+(a_n)^2}

になるが、この函数は C^∞ 級だが C^ω 級でないから、収束がおそい。

266 :132人目の素数さん:2018/07/25(水) 15:57:27.87 ID:HsCqfSb5.net
>>265
訂正
(誤)a_(n+1)=(a_n)/{1+(a_n)^2} 
(正)a_(n+1)=(a_n)/{1+(a_n)}^2
です。ご迷惑をおかけします。
(2)の極限値は分かるのですがノーヒントで証明が思いつきません。

267 :132人目の素数さん:2018/07/25(水) 16:06:47.38 ID:bWiMGJ2R.net
数学3 極限

lim x^3/(x^2−4)
x→2+0

無限大になるのは分かるんだけど、その過程教えてください

268 :132人目の素数さん:2018/07/25(水) 16:09:07.34 ID:aSG76bcm.net
>>266
極限値は何?

269 :132人目の素数さん:2018/07/25(水) 16:13:07.91 ID:FxD1zx7b.net
分子 → 8+

分母 → 0+

270 :132人目の素数さん:2018/07/25(水) 16:14:18.36 ID:bWiMGJ2R.net
>>269基本的な事忘れてましたありがとうございます

271 :132人目の素数さん:2018/07/25(水) 16:30:30.43 ID:aSG76bcm.net
1/a[n+1] = 1/a[n] + 2 + a[n]
により1/a[n]>2n。
とくに
limsup n a[n] ≧ 1/2。… (A)
e>0を任意にとるときn>1/(2e)にたいして
1/a[n+1]<1/a[n] + 2 + e
よってn>m>1/(2e)のとき
1/a[n]<1/a[m] + (2 + e) (n-m)
∴limsup 1/(n a[n]) < 2+e
eは任意であったから
limsup 1/(n a[n]) ≦ 2。
∴liminf n a[n] ≧ 1/2。… (B)
(A) (B)よりlim n a[n] = 1/2。

272 :132人目の素数さん:2018/07/25(水) 22:26:16.95 ID:6et0LE7N.net
スレ違いならすみません。
62390という数を、1510、1480、1465、1450、1435を自由に使って作ることは可能でしょうか?
また、このような問題の分野と、それを解決できるツールがあれば教えてください。

273 :132人目の素数さん:2018/07/25(水) 22:46:22.30 ID:RAUAZrJH.net
グラフ問題?について質問させて下さい

・最大マッチング問題(辺の数が最大となるような組み合わせを求める)
・最大重みマッチング問題(辺の数は最大でなくてもいいので重みを最大にする)

この2つを組み合わせて

辺の数が最大となるような組み合わせを求める
その組み合わせが複数ある場合は、重みが最大となる一つを選ぶ

という問題の解法を調べているのですが分かりません
参考になるようなリンク等ありましたら教えて頂けませんでしょうか

274 :132人目の素数さん:2018/07/25(水) 23:04:33.59 ID:8Htxhw8x.net
>>268
2です

275 :132人目の素数さん:2018/07/25(水) 23:12:52.73 ID:8Htxhw8x.net
3^n=k^2-40
を満たす自然数(k,n)をすべて求めよ。

276 :132人目の素数さん:2018/07/25(水) 23:25:48.95 ID:8Htxhw8x.net
平面の原点Oを中心とする半径1の円Cに、正{7+3*2^(n+1)}角形Vnが内接している。
Vnの一頂点をA(1,0)とする。y座標が正であるVnの頂点Pで、|AP-√3|を最小とするものはただ一つに定まることを示せ。

277 :132人目の素数さん:2018/07/25(水) 23:32:40.96 ID:6lMObFG1.net
>>275
3^n ≡ k^2 (mod 4)よりnは偶数。
3^n = l^2とおいて
k^2 - l^2 = 40。
これを解いて
(k,l) = (7,3),(11,9)
よって
(n,k) = (1,7), (2,11)。

278 :132人目の素数さん:2018/07/25(水) 23:38:28.62 ID:8Htxhw8x.net
xy平面の直線のうち、格子点を通り、かつx軸またはy軸に平行なものを格子直線と呼ぶ。
また点A,B,X,YをA(2,1),B(3,1),C(4,2),D(4,3)とする。

点(0,0)から点(5,0)まで格子直線を通って向かう経路のうち、以下の条件を満たすものの総数を求めよ。

(1)線分ABと線分CDを共に通る最短経路。

(2)線分ABと線分CDの少なくとも一方を通る最短経路。

(3)線分ABと線分CDの少なくとも一方は通らない最短経路。

(4)線分ABをちょうど2回通る経路のうち、長さが16以下のもの。

279 :132人目の素数さん:2018/07/25(水) 23:40:42.70 ID:8Htxhw8x.net
これ難問なんですけど早すぎじゃないですか
答えが微妙に違いますがほぼ合ってるので素晴らしいです

280 :132人目の素数さん:2018/07/25(水) 23:47:04.46 ID:8Htxhw8x.net
教えていただきたいのですが、
次の無限級数には何か名前があったり、あるいは値が正確に表現されていたりしますか?

Σ[k=1,∞] {1/(k^k)}

281 :132人目の素数さん:2018/07/25(水) 23:49:48.30 ID:6lMObFG1.net
まぁx^2 - dy^2 = Nの形の不定方程式は死ぬほど扱ってきたからなぁ。

282 :132人目の素数さん:2018/07/26(木) 00:24:14.83 ID:r9ee9dZW.net
>>276
∠AOB = π/3 である円周上の点Bをとり劣弧AB上でBに最も近い頂点をB、優弧AB上でのそれをDとする。
Bは劣弧CDを3頭分した点のうちCに近い側である。
よって∠BOC = θとすれば∠BOD = 2θである。
1/2((AD - √3) - (√3 - AC)) = sin (π/3 - θ) + sin(2π/3 - 2θ) - √3 > 0 if 0.2 であり
θ≦ π/57 < 0.2
であるから|AD - √3| > |√3 - AB|である。

283 :132人目の素数さん:2018/07/26(木) 00:58:36.67 ID:r9ee9dZW.net
>>266 >>274
1/2だろ?
*Main> let a = iterate (¥x->x/(1+x)^2) 0.5
*Main> let b = zipWith (*) [1..] a
*Main> mapM_ print $ map (b!!) [10^n|n<-[1..6]]
0.4706066031324342
0.4939644764076295
0.49909980269951604
0.4998810136240857
0.49998522003831564
0.4999982341343909

284 :132人目の素数さん:2018/07/26(木) 01:45:54.97 ID:DYWkG8lH.net
次元の狭間

285 :132人目の素数さん:2018/07/26(木) 02:17:22.91 ID:r9ee9dZW.net
>>282
訂正
×:よって∠BOC = θとすれば∠BOD = 2θである。
○:よって∠BAC = θとすれば∠BAD = 2θである。

286 :132人目の素数さん:2018/07/26(木) 06:31:42.56 ID:OBqa5DS5.net
>>277
最後、間違えてないか?

287 :132人目の素数さん:2018/07/26(木) 07:07:51.77 ID:N6IPLoB7.net
>>272
ナップザック問題かな?

288 :132人目の素数さん:2018/07/26(木) 09:34:31.15 ID:dlr7KsyL.net
ITストラテジストの試験と慶應義塾大学の入試ってどっちの方がムズイ?

289 :132人目の素数さん:2018/07/26(木) 10:56:16.53 ID:g7KCpv5X.net
重回帰式において、(yi-bar(y))(Yi-bar(Y))=(Yi-bar(Y))^2を証明してください
重回帰式の特徴よりbar(y)=bar(Y)です

290 :132人目の素数さん:2018/07/26(木) 12:43:31.11 ID:Tfbt9ILR.net
https://i.imgur.com/6Hy9i2Q.jpg
補足:図を使用せず、全て数式に則った証明をすること。

がなかなかに厳しいです助けてください

291 :132人目の素数さん:2018/07/26(木) 12:50:24.91 ID:cjFJlyDD.net
一対一かつ連続かつ逆写像も連続であることを示せば良いですね
e^zもlnzも連続だから難しいことはないはずです

292 :132人目の素数さん:2018/07/26(木) 16:04:38.40 ID:QncmjHnP.net
数学の研究職ってコミュ力必要?

293 :132人目の素数さん:2018/07/26(木) 16:12:31.10 ID:bHOX37HS.net
東京大学大学院数理科学研究科数理科学専攻博士課程修了するまで俺は自由になれない。
まずは東京大学理学部数学科に入ることからだな。

294 :132人目の素数さん:2018/07/26(木) 17:32:57.75 ID:rNwaGvJH.net
スターリングの公式を使えば
Σ[1,...,∞] 1/k^k
を求められますか?

295 :132人目の素数さん:2018/07/26(木) 18:32:20.62 ID:rNwaGvJH.net
pを1より大きい有理数として、無限級数
Σ[n=1,...,∞] (1/n^p)
を考える。
ある整数でないpを1つ決め、それに対してこの無限級数の値を計算せよ。

296 :132人目の素数さん:2018/07/26(木) 20:22:44.20 ID:TLtlgSCy.net
ポエムフレイバーは消さないと

297 :132人目の素数さん:2018/07/26(木) 20:55:20.15 ID:2ar1aFhZ.net
0228 132人目の素数さん 2018/07/23 17:37:06
cosh(ax)=bx
でxの求め方が分かりません。
教えて頂けないでしょうか。

これはどうでしょうか?
分からない場合は、分からないとレスを頂けないでしょうか。

298 :132人目の素数さん:2018/07/26(木) 20:59:00.39 ID:xTc7kgzz.net
中3です。高校のオープンキャンパスに行ったところ,以下の問題を宿題として出されました。
中学校の先生に質問しようとしましたが,あいにく数学の先生はすべて出張やお休みで不在でした。
解き方を教えてください。

「1/x+1/y=1/4を満たす整数x,yの組をすべて示せ。」

299 :132人目の素数さん:2018/07/26(木) 21:07:06.70 ID:xu9Vtb0J.net
>>228
実定数a,bに対して実数xの値を求めたいのなら、
数値的に求めるしか手はないんじゃないの。

300 :132人目の素数さん:2018/07/26(木) 21:19:54.89 ID:txCZZ21a.net
>>298
4x+4y=xy
(x-4)(y-4)=16

301 :132人目の素数さん:2018/07/26(木) 21:21:09.43 ID:gAFwlRJi.net
>>291
ありがとうございます
fの連続性は明らか(使っていいものかどうかわかりませんが…)、
fの単射性を示すところまでは行けました
fの全射性とf^-1の連続性を示す見当がつかないので具体的に手順を示していただけると助かります

302 :132人目の素数さん:2018/07/26(木) 21:34:45.95 ID:xTc7kgzz.net
>>300
早速ありがとうございます。
そうすると,(x,y)=(5,20),(6,12),(8,8)の3組でよいですか?
ところで,
4x+4y=xy
から
(x-4)(y-4)=16
への因数分解は
x(y-4)-4y=0
x(y-4)-4(y−4)-16=0
(x-4)(y−4)=16
という手順でよいですか? (でも,解答を教えてもらわなかったら,自力での発想は無理です。)

303 :132人目の素数さん:2018/07/26(木) 21:55:25.66 ID:K6EL/1Fv.net
>>301
zにたいして、w=ln|z|+i arg zと選ぶとe^w=zとなりますから、全射です

適当に定義域を制限すれば正則な対数関数の分枝を選ぶことが可能ですからf^(-1)は連続です

304 :132人目の素数さん:2018/07/26(木) 22:07:10.48 ID:K6EL/1Fv.net
連続性は普通にイプシロンデルタでもけそうですね

305 :132人目の素数さん:2018/07/26(木) 22:20:15.52 ID:txCZZ21a.net
>>302
整数とあるので負の整数も考えたら

306 :132人目の素数さん:2018/07/26(木) 23:02:41.59 ID:xTc7kgzz.net
>>305
見落としてました。ありがとうございます。

307 :132人目の素数さん:2018/07/26(木) 23:06:58.65 ID:o/PWI9Ie.net
>自力での発想は無理です。

歴戦のつわものが言うならアレだが、その辺の修行すらさぼってる雑魚がコレ言うと酷いな……

308 :132人目の素数さん:2018/07/26(木) 23:11:59.98 ID:K6EL/1Fv.net
>>307
ある無矛盾な公理系τの任意のモデルに対してある論理式φが常に真となるならば、τからφがLKにおいて証明可能となることを示せ

わからないんですか?
数理論理の修行はサボってるんですか?

309 :132人目の素数さん:2018/07/27(金) 00:09:42.92 ID:OTEY1FQ9.net
すいません、この問題教えてください

正八角形ABCDEFGHがあって、点P,QはそれぞれDEをDの方向に、AHをHの方向に延長した直線上の点で、∠CPD=∠GQHである。
いま点PがBC上にあって、AB=8cmのとき、六角形ABPEFQの面積から正八角形ABCDEFGHの面積を引いた値を求めよ。

310 :132人目の素数さん:2018/07/27(金) 01:43:36.34 ID:muIjq652.net
オウム幹部
富永昌宏 灘高校→東京大学医学部
(『大学への数学』誌の学力コンテストで3期連続全国1位を記録)

311 :132人目の素数さん:2018/07/27(金) 02:32:28.47 ID:VAeMRiWK.net
俺も尋常じゃないくらいの天才になりたかった・・・・・・・・・・。
というより、できれば全知全能の究極至高の存在になりたかった・・・・・・・・・。
どうすればいいのか・・・・・・・・・・・・・。

312 :132人目の素数さん:2018/07/27(金) 04:28:43.37 ID:eNyPVbQ1.net


313 :132人目の素数さん:2018/07/27(金) 07:25:11.55 ID:M0ybsptA.net
>>310
世間知らずではあったんだろうけどここまでの知能があってオウムの詭弁性に気づかないとは思えないんだがなあ
いったい自分の中ではどう処理していたんだろう?

314 :132人目の素数さん:2018/07/27(金) 07:29:25.27 ID:2WqeV1wz.net
詭弁だろうが何だろうがその組織が認めてくれる、その組織の中でなら自分を活かせる
オウムでしか自分は活躍できないみたいに思うパターンもあるようだ

315 :132人目の素数さん:2018/07/27(金) 09:19:55.15 ID:ZegudDuy.net
薬物で本人にとっては現実の神秘体験見せて騙した

コロッと騙された

316 :132人目の素数さん:2018/07/27(金) 11:31:59.25 ID:cJXcrs+F.net
自分を騙すのが上手いだけだろ

317 :132人目の素数さん:2018/07/27(金) 23:35:37.65 ID:tTBSyRDI.net
>>272
Haskell で書いてみた。
Timeoutで codepad では計算してくれない。orz
http://codepad.org/vnEr01Lq
でもlocalで動かしてみるとNothingになった。

318 :132人目の素数さん:2018/07/27(金) 23:42:19.98 ID:gJZlLJV9.net
>>272
1510/1510+1510/1510+....と62390回足せばできますね

319 :132人目の素数さん:2018/07/28(土) 00:07:27.41 ID:RIeXFC2a.net
a,b,p,qを有理数とし、さらにx=a+√pはy=b+q^(1/3)より大きいとする。
x>z>yなる有理数zが存在することを示せ。

320 :132人目の素数さん:2018/07/28(土) 00:59:59.64 ID:EnyRsA6W.net
>>319
x>y
[ 1/(x-y) ] + 1 = n(自然数)とおくと
1/(x-y) < n,
x-y > 1/n,
[nx] = m(整数)とおくと
(m+1)/n > x ≧ m/n,
また y < x - 1/n < m/n から,
∴ x > m/n > y,

321 :132人目の素数さん:2018/07/28(土) 02:56:58.03 ID:RIeXFC2a.net
a_1=a
a_(n+1)=r*a_n/{p+q(a_n)}^2
で与えられる数列{a_n}のn→∞における極限が収束するような自然数a,p,q,r の条件を述べよ(これが無条件に収束する数列であったとしても、そのことを理由を付けて述べよ)。
また、収束する場合の極限値をa,p,q,rで表せ。

322 :132人目の素数さん:2018/07/28(土) 03:12:28.41 ID:tdwrEsTz.net
>>299
cosh(ax)=bx
は数値的に求めるなら簡単なのですが、
数式的にxを求めたいのです。
分からないなら、分からないで構いません。
その場合は、分からないと言って頂けないでしょうか。

323 :132人目の素数さん:2018/07/28(土) 03:45:27.33 ID:yqfqIyrq.net
e^x=xに帰着されますよね、その問題
ランベルトのW関数という特殊関数使わないと書けませんから、わからない、でいいと思いますよ

324 :132人目の素数さん:2018/07/28(土) 11:03:42.17 ID:C5B6wQcd.net
高校生が「志願したい大学」 関東の総合1位は早大
 
文系は青学 、理系は日大 進学ブランド力調査 高校生新聞

https://headlines.yahoo.co.jp/hl?a=20180719-00010000-koukousei-soci

325 :132人目の素数さん:2018/07/28(土) 11:05:36.52 ID:Dc7fo6/B.net
初心者ですlog2は超越数なのですか?(logは自然対数)

326 :132人目の素数さん:2018/07/28(土) 11:15:25.42 ID:X+Gudecp.net
a1,…,anが相異なる代数的数のとき
e^a1,…,e^anはQの代数的閉包上線形独立である。(Lindemann)

327 :132人目の素数さん:2018/07/28(土) 12:24:13.42 ID:qIe5wP4f.net
なるほど

328 :132人目の素数さん:2018/07/28(土) 12:52:00.10 ID:wb0cTFEt.net
>>309
角度を計算すれば点Qが直線FG上にあることが分かる。
求める面積は△CPDと△GQHの面積の和。
それぞれの三角形は斜辺が8cmの直角二等辺三角形。

329 :132人目の素数さん:2018/07/28(土) 13:35:45.55 ID:RIeXFC2a.net
以下の6点を頂点とする正八面体Vがある。
A(√2,0,0)、B(0,√2,0)、C(-√2,0,0)、D(0,-√2,0)、E(0,0,√2)、F(0,0,-√2)

また、以下の3本の円柱を考える。
円柱S:x^2+y^2=1
円柱T:y^2+z^2=1
円柱U:z^2+x^2=1

このとき、領域
「『Vの内部』かつ『Sの外部』かつ『Tの外部』かつ『Uの内部』」
の体積を求めよ。

330 :132人目の素数さん:2018/07/28(土) 14:04:43.66 ID:z2BC7zek.net
1/(z^3+4z) C:|z|=3に沿う積分の値を求めよ
という問題が解けません。コーシーの積分公式或いはその拡張を用いるみたいなんですが、どなたかやり方を教えてください

331 :132人目の素数さん:2018/07/28(土) 16:18:56.28 ID:/1hrZIEH.net
確率変数の問題で50部と60部どっちが期待値いいのか調べる問題
https://i.imgur.com/FpPOYsW.jpg
余った一部ごとに15円損なら60部の方の計算は
(60-40)*15*0.25ですよね
なんで10かけてるの
普通に教科書間違ってるなら二度と使わんこれ

332 :132人目の素数さん:2018/07/28(土) 17:09:08.91 ID:/1hrZIEH.net
>>331
この問題答えなくて大丈夫です
他にも間違った解答あったから多分ミスばっかこの本
大学指定だったんだけどな
ガッカリ

333 :132人目の素数さん:2018/07/28(土) 19:29:31.48 ID:1yooDr8P.net
自然数を2個以上の連続する自然の和で表すことを考える。
(1)2020を2個以上の連続する自然数の和で表す表し方を全て求めよ
(2)a∈ℤ を0以上として、2^aは2個以上の連続する自然数の和で表せないことを示せ。

これの回答が画像なんですが、もっと綺麗な回答または別解があれば教えてください

https://i.imgur.com/ciTi3Zq.jpg

334 :132人目の素数さん:2018/07/28(土) 19:36:12.10 ID:1yooDr8P.net
流石に和の公式は使わなきゃだから難しいですかね

335 :132人目の素数さん:2018/07/28(土) 20:57:29.94 ID:BJbC0qJ1.net
ゲームを作っていて組み合わせがよく分からないので教えてください。
スライムがa-iまで9体居ます。
9体集まると特大スライムとなります。
しかし2体〜8体集まっても融合スライムとなります。
スライムを消す条件があって2-8体の融合スライムはアルファベットを指定しないと消せなくなりました。
そこで2-8体の融合スライムの組み合わせは何通りありますか?

336 :132人目の素数さん:2018/07/28(土) 21:02:54.11 ID:Ssa/WzHP.net
スライムタワーやスライムキングはないのか?

337 :132人目の素数さん:2018/07/28(土) 21:07:20.78 ID:yqfqIyrq.net
>>335
スライムを消す条件があって2-8体の融合スライムはアルファベットを指定しないと消せなくなりました。

とはどのようなことですか?

338 :132人目の素数さん:2018/07/28(土) 21:18:07.73 ID:BJbC0qJ1.net
>>337
例えばabのスライムが融合して融合スライムとなります。
当たり判定がaには当たっててbには当たってないので融合スライムは永久に倒せません。
融合スライムを融合スライム(abのスライム)と変数指定してないとabには融合スライムは倒せないのです…

339 :132人目の素数さん:2018/07/28(土) 21:24:50.00 ID:yqfqIyrq.net
>>338
あなたはそれでわかるのかもしれませんが、他の人はなんの話をしてるのかさっぱりわかりません
もう少し詳しくお願いしますね

340 :132人目の素数さん:2018/07/28(土) 21:36:32.15 ID:BJbC0qJ1.net
>>339
捕捉
エクセルでローグライクを作っていて縦横のマスが動けるマスでスライムがセル中で隣同士になったら融合します。
□__□←a.bのスライム
□□←隣り合う
[ΞΞ]←2マス使って融合する←このスライムは変数設定していないので倒せない。のです。
だから組み合わせを知りたくて。その組み合わせをマップにしてどの組み合わせの可能性残ってるかを演算したいのです。でも組み合わせがわからなくて…

341 :132人目の素数さん:2018/07/28(土) 21:43:01.32 ID:yqfqIyrq.net
>>340
どの組み合わせの話なのかはイマイチピンとこないのですが、おそらくアルゴリズムを変えた方がいいと思います

プログラム内では、融合スライムはどのように扱っていて、また当たり判定はどのようしているのですか?

342 :132人目の素数さん:2018/07/28(土) 21:59:29.80 ID:BJbC0qJ1.net
>>341
今試しで作っただけなので2体融合ならj-mとなっています。3体融合ならn-pという感じで8体用まであって…
フィールドはa-iの9体を倒したら次に進める階段が出現するようにしてあります。
はじめa-uまで倒したらとしましたが、融合スライムを倒したら進めなかったです。それもそのはずで、a-iを倒さないと進めないからです、そこで、
マップを書いて抜けないようにしたくて組み合わせが知りたいのです。
http://o.8ch.net/17x2s.png

343 :132人目の素数さん:2018/07/28(土) 22:06:26.39 ID:yqfqIyrq.net
>>342
思ってたのと違いますが、複雑になるだけな気がしますのでやはりプログラム変えた方が良いと思います

合体したら、攻撃の時は融合スライムとして扱うけど、倒した後は元のスライムとして扱ったらどうですか?
abの融合スライムを倒したら、aのスライムとbのスライムを倒したことにするんです
そしたらどれだけ融合しようが、全部のスライム倒せば自動的にa-iのスライム倒したことになりますよね

344 :132人目の素数さん:2018/07/28(土) 22:17:17.64 ID:BJbC0qJ1.net
>>343
ということは、例えば融合スライム2、融合スライム3があって
スライムのマスター9が0となる変数iの時0になったら階段出現、融合スライム2を倒したら融合スライムを消し変数に対し-2をする
融合スライム3を倒したら変数に対し-3をし0になるまで繰り返すという認識でよろしいでしょうか?

345 :132人目の素数さん:2018/07/28(土) 22:19:09.74 ID:yqfqIyrq.net
>>344
多分いいんじゃないんですか?

346 :132人目の素数さん:2018/07/28(土) 22:23:45.32 ID:BJbC0qJ1.net
>>345
ありがとーございます。
大変参考になりました。
プログラムもいける人が数学スレには居るんですね。

347 :132人目の素数さん:2018/07/29(日) 01:37:23.81 ID:MbuK+QQd.net
>>332
こういう本の著者に限って「単位やらんぞ」攻撃するから要注意。
日本の大学ってカスばっか?
訂正のチャンスを自ら潰すダメ講師

348 :132人目の素数さん:2018/07/29(日) 02:00:29.80 ID:MbuK+QQd.net
>>330
まづ部分分数に分解する。
1/[(zz+4)z] = {1/z - z/(zz+4)}/4 = 1/(4z) - (1/8){1/(z-2i) + 1/(z+2i)},
極は3つとも円周Cの内側ですね。

349 :132人目の素数さん:2018/07/29(日) 05:32:34.05 ID:T2+TKFwa.net
自分は、超絶ド底辺高校に進学してしまったということと、その高校を卒業してしまったということが悔しくて悔しくて仕方がない。
この事実が永遠に消えることは無いし、恐らくこの先一生それが付きまとうだろう。
例え東大卒になろうが所詮超絶ド底辺高校出身に変わりはない。
もう嫌だこの糞みてえな人生。
俺はこれから先どうすれば良いのか・・・・・。

350 :132人目の素数さん:2018/07/29(日) 05:51:34.51 ID:vEBWfNbf.net
N 自然数全体の集合
  ∃可逆写像f:N→N ∀可逆写像g:N→N ∃k k≦∀n g(n)<f(n)
は真ですか?

351 :132人目の素数さん:2018/07/29(日) 05:55:16.44 ID:T2+TKFwa.net
神様と数学はどっちの方が偉大ですか?

352 :132人目の素数さん:2018/07/29(日) 06:37:42.04 ID:dOyuex2D.net
>>350
その命題は明らかに偽やろ?
∀可逆写像f:N→N ∃可逆写像g:N→N ∀k k≦∃n g(n)≧f(n)
の方が明らかに真やん。g=fにすればいいだけなんだから。

353 :132人目の素数さん:2018/07/29(日) 07:09:54.30 ID:vEBWfNbf.net
>>352
ほんとだサンクス

354 :132人目の素数さん:2018/07/29(日) 07:22:40.04 ID:T2+TKFwa.net
自殺するか迷う。

355 :132人目の素数さん:2018/07/29(日) 09:30:23.62 ID:C8LkFFNw.net
らや

356 :132人目の素数さん:2018/07/29(日) 11:37:26.01 ID:EAyZdYxC.net
ただの荒らし

357 :132人目の素数さん:2018/07/29(日) 12:51:46.79 ID:7gQzAGuw.net
井山裕太さんとグレゴリー・ペレルマンさんはどっちの方が賢いですか?

358 :132人目の素数さん:2018/07/29(日) 14:37:51.07 ID:FjQ1wNJ0.net


359 :132人目の素数さん:2018/07/29(日) 14:52:09.10 ID:xMOG1tbT.net
東大院卒の脳神経外科医と東大院卒の宇宙飛行士はどっちの方が頭が良いですか?

360 :132人目の素数さん:2018/07/29(日) 14:59:49.59 ID:FjQ1wNJ0.net


361 :132人目の素数さん:2018/07/29(日) 15:04:23.91 ID:xMOG1tbT.net
アラン・コンヌとジョン・フォン・ノイマンはどっちの方が頭がいいの?

362 :132人目の素数さん:2018/07/29(日) 15:26:14.09 ID:UsYaJvOu.net


363 :132人目の素数さん:2018/07/29(日) 15:43:47.89 ID:rhCoBh5h.net
>>323
ラインベルトのW関数を見ました。
良く分かりませんでしたが、また考えてみます。
情報をありがとうございます。

364 :132人目の素数さん:2018/07/29(日) 19:20:18.56 ID:xIlkNxOR.net
𒄑𒂆𒈦
↑なんて読むの?

365 :132人目の素数さん:2018/07/29(日) 19:20:26.72 ID:xMOG1tbT.net
全知全能の究極至高神と無はどっちの方が凄いの?

366 :132人目の素数さん:2018/07/29(日) 19:22:38.95 ID:xIlkNxOR.net
>>364
ググったらすぐに意味は分かった。
発音はどうでもいいや。
じゃあの。

367 :132人目の素数さん:2018/07/29(日) 19:27:44.85 ID:DRAZ02jY.net
シュメール文字でしたっけ

368 :132人目の素数さん:2018/07/29(日) 19:58:06.78 ID:Nzd2wgGl.net
平面:x+y+z-1=0に垂直で原点(0.0.0)を通る直線の方程式を求めよ。
っていう問題で、平面の法線ベクトル(1.1.1)を使うってところまではわかるんですけど、そのあとどうすれば良いのかが分かりません。教えてください。

369 :132人目の素数さん:2018/07/29(日) 20:03:39.26 ID:aakQEixt.net
>>368
(x,y,z) = t (1,1,1) (t はパラメータ)
で出来上がり

370 :132人目の素数さん:2018/07/29(日) 20:09:52.43 ID:Nzd2wgGl.net
>>369
tって、どんな数字を入れても良いってことですよね?
原点を通るのに1つの直線に定まらないんですか?

371 :132人目の素数さん:2018/07/29(日) 20:12:58.73 ID:DRAZ02jY.net
直線はたくさんの点が集まって出来てますよね
tを変化させると点が集まって線になるんです

372 :132人目の素数さん:2018/07/29(日) 20:18:46.17 ID:aakQEixt.net
>>370
座標平面の直線の式とは同じ形にはならないので念のため
パラメータを消去すれば x=y=z となるが
実際の問題では >>369 の表式のほうが使いやすいことが多いだろう

373 :132人目の素数さん:2018/07/29(日) 20:25:12.84 ID:Nzd2wgGl.net
>>372
なるほど!
わかりました。ありがとうございます!

原点ではなく、例えば(2.3.5)を通るとすると、どうなりますか?

374 :132人目の素数さん:2018/07/29(日) 21:04:59.46 ID:K51cTAE8.net
アンケートです

【事実から確実に導かれる論理的帰結までは、事実の範疇だろう

「リンゴが落ちる」という現象を、
「万有引力によりリンゴと地球が引き合うためだ」と説明する部分は、事実としていいだろ?

厳密に言えば、二つ目の括弧は解釈だけれど、普通はこれを「事実とする」だろうが?】

この内容について「事実とする」ことを認める方、いらっしゃいますか?
感想を添えてご回答いただけると幸いです。
どうぞよろしくお願いいたします。

375 :132人目の素数さん:2018/07/29(日) 21:06:41.39 ID:MefUqIVV.net
はい

376 :132人目の素数さん:2018/07/29(日) 21:15:34.37 ID:DRAZ02jY.net
>>374
>【事実から確実に導かれる論理的帰結までは、事実の範疇だろう

>厳密に言えば、二つ目の括弧は解釈だけれど、普通はこれを「事実とする」だろうが?

質問自体がおかしい気がします

上の通りだとすれば、万有引力は確実に導かれる論理的帰結、となっていますが、下ではそれを解釈に過ぎない、と言っています

どちらなのですか?

377 :132人目の素数さん:2018/07/29(日) 22:15:09.50 ID:JdwFFF1G.net
>>373
235を足せよ

378 :132人目の素数さん:2018/07/29(日) 22:35:22.75 ID:ndWNCrQy.net
ω^4=1を満たす複素数をすべて答えよ

因数分解以降どう解けば良いですか

379 :132人目の素数さん:2018/07/29(日) 22:36:47.53 ID:Ary0Y+gs.net
僕は数検3級を受けました
答え合わせをしたいので
どなたか解答を教えて下さい
よろしくお願いします

380 :132人目の素数さん:2018/07/29(日) 23:00:26.43 ID:KHk+tqN9.net
任意の自然数nに対してα^nは無理数である。
このような無理数αの例を1つ挙げ、またαが無理数であることの証明を与えよ。

381 :132人目の素数さん:2018/07/29(日) 23:05:20.88 ID:oBbvklHp.net
1+√2

382 :132人目の素数さん:2018/07/29(日) 23:18:19.70 ID:DRAZ02jY.net
>>378
ω=±1,±i

383 :132人目の素数さん:2018/07/29(日) 23:25:18.06 ID:BCVUvsdG.net
>>378
2次式2つの積で書けたらあとは公式

384 :132人目の素数さん:2018/07/29(日) 23:39:45.56 ID:qMY51965.net
乗法群の単位元って1…ですよね…?
んで、加法群の単位元って0ですよね?

f^-1(1)が加法群となるのは何故ですかね?
なんか勘違いしてる?

385 :132人目の素数さん:2018/07/29(日) 23:55:54.95 ID:TlLd13hm.net
何一つ理解してない

386 :132人目の素数さん:2018/07/30(月) 00:14:28.76 ID:Zfc8Qfib.net
みんなむずかしそうなことを聞いていて申し訳ないのですが
写真の極限がわかりません…助けてください

387 :132人目の素数さん:2018/07/30(月) 00:14:56.17 ID:ZOh5g7QP.net
ええ...申し訳ないですが具体的に教えてください

388 :132人目の素数さん:2018/07/30(月) 00:15:25.37 ID:Zfc8Qfib.net
https://i.imgur.com/216jGEo.jpg
貼れてなかったorz
これです

389 :132人目の素数さん:2018/07/30(月) 00:16:47.78 ID:ZOh5g7QP.net
>>388 = >>387 = >>384>>386

390 :132人目の素数さん:2018/07/30(月) 00:16:49.29 ID:LR2oP5g2.net
θとか関係ないじゃんこれ

391 :132人目の素数さん:2018/07/30(月) 00:17:32.47 ID:ZOh5g7QP.net
>>389
等式中の>>388>>389の誤りです

392 :132人目の素数さん:2018/07/30(月) 00:42:01.07 ID:Ww1+Y1g7.net
>>376
回答がありました

もうちょっとがんばれ

「事実から確実に導かれる論理的帰結」の「導かれる」の部分が
「厳密には解釈」なんだよ

だが、「確実に」導かれるなら事実の範疇だろう?

393 :132人目の素数さん:2018/07/30(月) 00:42:57.92 ID:GNdiU6iv.net
>>384
0と書くか1と書くかは関係ない
ただ加法群(乗法群)と言えば演算は+(*)で書かれることが多く、その単位元を0(1)で表すことが多いというだけです
必ずしも0,1で書かなければならないわけでもなく、例えばn次可逆行列の全体GL(n)の単位元(つまり単位行列)は1ではなくEまたはIで書かれることが多いですよね
問題文は正確に書きましょう

エスパーすれば「加法群G、乗法群H、準同型f:G→Hとして、Hの単位元1に対してf^{-1}(1)がGの部分群になることを示せ」かな?
加法も乗法も関係なく成立するけど

394 :132人目の素数さん:2018/07/30(月) 09:46:32.12 ID:kQpZ55zq.net
結局「超準解析」とか「絶対数学」って非主流の数学なんですか?
モデルとしても机上の空論なんですか?

395 :132人目の素数さん:2018/07/30(月) 10:02:30.15 ID:OjocGL6M.net
馬鹿は絡みたがる

396 :132人目の素数さん:2018/07/30(月) 10:06:06.48 ID:kYeL0k3F.net
厨房だろ

397 :132人目の素数さん:2018/07/30(月) 10:09:43.42 ID:wgzFDqpM.net
わからないんですね

398 :132人目の素数さん:2018/07/30(月) 10:19:11.31 ID:Lq/N8S81.net
アラン・コンヌと釈迦はどっちの方が頭が良いですか?

399 :132人目の素数さん:2018/07/30(月) 10:19:52.85 ID:kYeL0k3F.net
劣等感ババアは煽りたがる(笑)

400 :132人目の素数さん:2018/07/30(月) 10:29:08.70 ID:qvxAXRne.net
全くまからないVとWがさっぱりあとUの567も
誰か解説お願いします



file:///D:/大学/img.pdf

401 :132人目の素数さん:2018/07/30(月) 10:33:17.86 ID:kYeL0k3F.net
劣等感ババアどうぞ(笑)

402 :132人目の素数さん:2018/07/30(月) 10:42:11.70 ID:wu2Q70UB.net
アラン・コンヌとグレゴリー・ペレルマンはどっちの方が頭が良いですか?

403 :132人目の素数さん:2018/07/30(月) 10:53:08.55 ID:yx+fxXON.net
中二病のレス乞食のおっさん

404 :132人目の素数さん:2018/07/30(月) 11:06:15.89 ID:+BJglcSi.net
神仏と無はどっちの方が凄いですか?

405 :132人目の素数さん:2018/07/30(月) 11:09:32.60 ID:ruJO5bbD.net
ここ質問スレとしてまともに機能してないよね
学部スレ、高校スレ、中高スレに誘導した方がいいかもしれない

406 :132人目の素数さん:2018/07/30(月) 11:14:17.09 ID:TVVJgEKF.net
どちらも今はあまり良くありません。子供時代は美しい金髪だったでしょう。

407 :132人目の素数さん:2018/07/30(月) 11:14:55.73 ID:NXtc3R80.net
ここは質問スレではありません
分からない問題を書くスレです

408 :132人目の素数さん:2018/07/30(月) 15:13:39.71 ID:rSe3jdja.net
>>384 >>393

f^{-1}(1) = Ker(f)

409 :132人目の素数さん:2018/07/30(月) 18:45:16.22 ID:NlalAfxL.net
>>405
ワッチョイをこの板じゃ入れられないのが惜しいな
荒らしは追い出せるし同じ質問者に詳しく何度も聞けるしで一石二鳥なのに

410 :132人目の素数さん:2018/07/30(月) 19:54:27.45 ID:yIr7IvD+.net
>>394
非主流よね
モデルとして机上の空論って何を言いたいの?

411 :132人目の素数さん:2018/07/30(月) 21:24:02.63 ID:YscM2T7E.net
問 媒介変数表示 x=cosθ y=sin2θ (−π≦θ≦π) で表される曲線の概形を書け

模範解答が、曲線がx軸に関して対称なので0≦θ≦π…@で考える。
dx/dθ=−sinθ (dy/dθはここでは省略)
@の範囲でdx/dθ=0を満たすθ=0、π

となってるんだけど、閉区間の端は微分出来ないなら、θ存在しなくね?
解説お願いします

412 :132人目の素数さん:2018/07/30(月) 22:07:12.34 ID:rvuO5ATY.net
片側極限
lim[θ→0+0](dx/dθ)=lim[θ→0+0](-sinθ)=-sin0=0
lim[θ→π-0](dx/dθ)=lim[θ→π-0](-sinθ)=-sinπ=0
というのを省略してるだけ

413 :132人目の素数さん:2018/07/30(月) 22:25:46.55 ID:GNdiU6iv.net
閉区間のおける微分可能性は、端点では片側微分可能ということで定義することが多い

414 :132人目の素数さん:2018/07/30(月) 23:42:15.57 ID:YscM2T7E.net
端点を含めずに、導関数=0を満たす値を考えている問題もありますが…

415 :132人目の素数さん:2018/07/30(月) 23:51:21.96 ID:TGmQCAIz.net
座標空間の直方体OABC-GHIJの各頂点の座標はそれぞれ
O(0,0,0)、A(1,0,0)、B(1,s,0)、C(0,s,0)、G(0,0,t)、H(1,0,t)、I(1,s,t)、J(0,s,t)
である。ただしs,tは正の実数である。
この直方体を平面z=0上の直線y=-xの周りに回転させる。

(1)直方体を回転させ、その折れ線HIJとz軸の正の部分とが初めて交わったとき、Aが移動した点をA'、Jが移動した点をJ'とする。
↑A'J'をs,tを用いて成分で表せ。

(2)初期状態から(1)の状況まで回転させるときの、直方体の通過領域をDとする。s=2、t=3のとき、Dに含まれる線分で最長のものの長さを求めよ。

416 :132人目の素数さん:2018/07/31(火) 00:27:05.56 ID:pm3dVnSO.net
確率変数x,yが独立かつ同一の確率分布に従うとはどういうことか。数式を用いて説明せよ。

独立は同時確率が周辺確率の積で表せることを書くとして、同一の確率分布に従うとは、どのように数式で書けば良いのですか?

417 :132人目の素数さん:2018/07/31(火) 00:28:11.43 ID:t9feO7CH.net
>>416
まるち

418 :132人目の素数さん:2018/07/31(火) 00:29:03.18 ID:qtXlvJ1k.net
>>416
周辺確率が同じ
何言ってんだかな

419 :132人目の素数さん:2018/07/31(火) 00:29:39.38 ID:qtXlvJ1k.net
>>417
で?

420 :132人目の素数さん:2018/07/31(火) 00:33:08.48 ID:v4R+9VCw.net
integral of √(1+cosx)/cosx
これって有名問題ですか?

421 :132人目の素数さん:2018/07/31(火) 00:56:26.85 ID:Tk7zeIY/.net
>>420
√(1+cosx)/cosx = √2 |cos(x/2)| / cos x
で三角関数の有理関数だから大学1回以降なら必須知識だな。
受験問題なら難しい。

422 :132人目の素数さん:2018/07/31(火) 01:09:23.47 ID:v4R+9VCw.net
>>421
有理関数の積分に帰着させて解いてたら三回置換して置換した変数のままの表記で原始関数を表すのはいけた気がしてるんですが本当に求めれたのか怪しいです

423 :132人目の素数さん:2018/07/31(火) 03:49:38.17 ID:2W1lOnbH.net
Cを複素平面、UをCの領域とし、fをU上の複素数値関数とする。
fがa∈Uで解析的なら、aを中心としUに含まれる任意の開円盤上でaを中心とするfのテイラー級数がfと一致することを証明したいのですが、これは名前のついている定理から導かれることでしょうか?

424 :132人目の素数さん:2018/07/31(火) 03:52:20.19 ID:2W1lOnbH.net
>>423
それともこれはfがU上で解析的としないと言えないことですか?

425 :132人目の素数さん:2018/07/31(火) 05:00:51.03 ID:WJMFWVUG.net
>>420-422

(σ/√2)log{[1+(√2)sin(x/2)]/[1-(√2)sin(x/2)]} + c,

σ = sgn(cos(x/2)),

426 :132人目の素数さん:2018/07/31(火) 06:57:08.24 ID:mB+/2YBY.net
宇宙飛行士とフィールズ賞受賞数学者はどっちの方が頭が良いですか?

427 :132人目の素数さん:2018/07/31(火) 06:57:34.89 ID:mB+/2YBY.net
宇宙飛行士とフィールズ賞受賞数学者はどっちの方が頭が良いですか?

428 :132人目の素数さん:2018/07/31(火) 07:00:17.56 ID:mytPqYND.net
>>423
一致の定理から言えますね

429 :132人目の素数さん:2018/07/31(火) 07:17:02.02 ID:mytPqYND.net
いや言えないかもしれませんね

430 :132人目の素数さん:2018/07/31(火) 07:20:06.88 ID:mytPqYND.net
>>423
f(z)=0(|z|<1)
=1(|z|≧1)

でz=0でテーラー展開可能で解析的ですが、連続関数ではないので半径があまり大きくなるとダメですね

431 :132人目の素数さん:2018/07/31(火) 08:37:34.17 ID:pm3dVnSO.net
>>418
任意の実数aに対して
P(X≦a)=P(Y≦a)
らしいですが何故でしょうか?

432 :132人目の素数さん:2018/07/31(火) 09:00:45.27 ID:cq4rdIWI.net
わからないんですね(笑)

433 :132人目の素数さん:2018/07/31(火) 09:22:45.41 ID:yf1KrkEn.net
>>432
ブーメランだし迷惑なのでやめてください

434 :132人目の素数さん:2018/07/31(火) 09:25:24.09 ID:OIXcWpPU.net
反例を探しています
「距離空間(X,dx)が点列コンパクトならば完備である」は成り立ちます
この逆が一般的には成り立たないとありますがそのような例が考えても作れませんでした

435 :132人目の素数さん:2018/07/31(火) 09:31:16.35 ID:mT9VJ48r.net
R

436 :132人目の素数さん:2018/07/31(火) 09:35:20.52 ID:OIXcWpPU.net
あー

437 :132人目の素数さん:2018/07/31(火) 09:45:56.20 ID:Bz7Wsu9E.net
世界最高の大学に入りたかった・・・。

438 :132人目の素数さん:2018/07/31(火) 09:52:11.21 ID:cq4rdIWI.net
2位じゃダメなんですか?

439 :132人目の素数さん:2018/07/31(火) 11:58:20.41 ID:Bz7Wsu9E.net
世界最高の大学に入りたかった・・・。

440 :132人目の素数さん:2018/07/31(火) 12:08:19.05 ID:v0e3DH0+.net
cos^2 (a+b) を、cos^2 (a) と cos^2 (b) だけで簡単に表現する式ありますか?

441 :132人目の素数さん:2018/07/31(火) 12:26:02.44 ID:YJY0cvHu.net
数学で世界最高の大学って、

UC Barkeley

ですか?

442 :132人目の素数さん:2018/07/31(火) 12:28:04.14 ID:EY865VSt.net
>>440
加法定理で終わり

443 :132人目の素数さん:2018/07/31(火) 12:45:08.98 ID:iC/sNhD3.net
>>434
実数全部の集合

444 :132人目の素数さん:2018/07/31(火) 12:46:03.96 ID:mNYIrrxJ.net
もし下記を求められる方がいましたら、ご教示頂けないでしょうか。



10000個のプログラムファイルがある。

特定のルートプログラムが1ファイルだけあり、ルートプログラムを参照しているプログラム(第一階層プログラムという)がX個ある。

次に、見つかったX個の第一階層プログラムから第二階層プログラムが見つかる可能性は10%である。

以降、第二階層プログラムから第三階層プログラムが見つかる可能性も10%である。

この操作を繰り返すと第N階層のNを求める一般式はどのようになるか?


なお、ある第M階層から第M+1階層を探すとき、第1〜M-1階層で既に見つかったプログラムファイルは対象外とする。


仕事で概算調査をしたいのですが、当方長い間数学から離れていまして、有識者方のお力お貸し頂けると嬉しいです。

445 :132人目の素数さん:2018/07/31(火) 12:49:03.85 ID:mNYIrrxJ.net
>>444
すみません、誤記がありましたので補足です。


この操作を繰り返し、全てのプログラムファイルの検索が完了した時点の第N階層のNを求める一般式はどのようになるか?

446 :132人目の素数さん:2018/07/31(火) 13:19:37.73 ID:YJY0cvHu.net
第N階層のNって何ですか?

447 :132人目の素数さん:2018/07/31(火) 13:23:59.66 ID:YJY0cvHu.net
確率が絡んでいるので、一般式は求まらないのではないでしょうか?

448 :132人目の素数さん:2018/07/31(火) 13:25:40.72 ID:wwzuizks.net
ベクトル空間U、Vに対し、線形写像f:U→V、g:V→Uがg(f(u))=u (u∈U) を満たす時、V=Im(f)+Ker(g) (+:直和記号) が成立することを示せ。

よろしくお願いします

449 :132人目の素数さん:2018/07/31(火) 14:13:34.82 ID:Ww7KNxQk.net
100円玉が2枚、10円玉が4枚入っている袋の中から2枚同時に取り出す。
取り出された10円玉の枚数をXとする。
このときE(3^E)を求めよ。

P(X=0)=1/15,P(X=1)=8/15,P(X=2)=6/15
までは計算したのですが、そのあとの
E(3^X)の求め方がわかりません…

すいません、よろしくお願いします。

450 :132人目の素数さん:2018/07/31(火) 14:26:34.30 ID:i8S2cV5s.net
>>448
e = fog : V→V とおく。
gf がU上の単射ゆえfも単射。∴ker g = ker fg = ker e。
gf がU上への全射ゆえgも全射。∴im f = ker fg = ker e。
ここで任意のVの元vはv=ev+(1-e)vとかけるがe^2=eよりe(1-e)=0だから(1-e)v∈ker e。
∴im eとker eはVを張る。
一方、v∈ Im e ∩ker eならばv=ewなるwがあり、ev=0だが
0=ev=e^2w=ew=v。
∴Im e ∩ker e=0。

451 :132人目の素数さん:2018/07/31(火) 15:41:09.67 ID:SitRTk0p.net
>>446
返信ありがとうございます。

10000個のプログラムを検査するとした場合、検査が全て完了した時点では第何階層になっているか、ということを求めたいのです。

10%で見つかるというのは10件検査して1件見つかるということです。

確率が絡むということでやはり数式にするには難しそうですね、、、

452 :132人目の素数さん:2018/07/31(火) 15:42:07.09 ID:G8y5o1AO.net
>>415
この(2)をお願いします

453 :132人目の素数さん:2018/07/31(火) 15:49:17.39 ID:cq4rdIWI.net
>>444
A→B→ルート
C→D
このような階層関係になっていると、BからCやDは逆算できませんが、このような場合はどうなっているんですか?

454 :132人目の素数さん:2018/07/31(火) 15:53:06.87 ID:wwzuizks.net
>>450
ありがとうございます

455 :132人目の素数さん:2018/07/31(火) 15:54:18.11 ID:G8y5o1AO.net
自然数nの2乗m=n^2に対し、m=n_0として、n_(k+1)を以下のいずれかの操作により定める。
(ア)n_(k+1)=n_(k)+1
(イ)n_(k+1)=n_(k)-1
(ウ)n_(k+1)=√{n_(k)}
いずれの操作が行われる確率も1/3である。
i=1,2,3,...に対して、n(i)が自然数となる確率をP(i)とする。
Σ[j=1〜∞] P(i)を求めよ。

456 :132人目の素数さん:2018/07/31(火) 15:57:56.89 ID:G8y5o1AO.net
一般に閉曲線内の領域に含まれる最長の線分の長さを求めるのは難しいのですか?

457 :132人目の素数さん:2018/07/31(火) 16:03:08.77 ID:i8S2cV5s.net
>>452
これは多分そんなに綺麗には解けない。
せいぜい直方体回す代わりに底面を回しても同じと気付いてちょっと楽できる程度。
あとはしょうもない計算ゴリゴリするしかなさそうなので誰もやらないと思う。

458 :132人目の素数さん:2018/07/31(火) 16:34:59.44 ID:rvsJSnBK.net
>>457
最長線分の構成方法を論じるのもしょうもないですか?

459 :132人目の素数さん:2018/07/31(火) 17:04:29.43 ID:i8S2cV5s.net
>>458
おお、そんな手があったか!これなら確かに鮮やかにとけてるなぁってうなるような解法があるなら書けばいいのでは?

460 :132人目の素数さん:2018/07/31(火) 17:10:55.75 ID:rvsJSnBK.net
3連続する3つの自然数p-1,p,p+1を選び、それら3数の積Pを十進法表記したときに現れる数字の種類をf(p)とする。
例えばp=5のとき、P=120であり、各位で数字1,2,0が現れているので、f(P)=3である。
f(P)=10となる最小のpを求めよ。

461 :132人目の素数さん:2018/07/31(火) 17:24:06.85 ID:s/IByTVc.net
世界最高の大学に入りたかった・・・。

462 :132人目の素数さん:2018/07/31(火) 17:33:55.25 ID:JQokAe8q.net
掛けるの記号×を・にするのってさ、入試本番で普通に使っていいの?
減点されたりしない?

463 :132人目の素数さん:2018/07/31(火) 17:35:32.43 ID:JQokAe8q.net
掛けるの記号×を・にするのってさ、入試本番で普通に使っていいの?
減点されたりしない?

464 :132人目の素数さん:2018/07/31(火) 17:43:14.39 ID:mytPqYND.net
中学生ならダメですね
高校生ならいいですね

465 :132人目の素数さん:2018/07/31(火) 17:53:06.88 ID:JQokAe8q.net
高校生です
大学入試なら普通に使っちゃっていいってことですね

466 :132人目の素数さん:2018/07/31(火) 22:09:35.80 ID:qsOK4DA4.net
>>460 1268

467 :132人目の素数さん:2018/07/31(火) 23:24:41.77 ID:WJMFWVUG.net
>>434

距離空間Xでは、
 コンパクト ⇔ 点列コンパクト ⇔ 「全有界かつ完備」

 「全有界」 ⇔ 任意のε>0に対し、Xを半径εの開球を有限個使って被覆できる。 ⇔ X上の任意の点列に対し、コーシー列である部分列が存在する。

全有界でないもの、たとえば X = [0,1]^∞

>>440

√ 使えばできそうだけど…

468 :132人目の素数さん:2018/08/01(水) 00:08:09.88 ID:QpFB5nHv.net
すみません、広義積分について質問です。
写真の(2)の広義積分の解き方なんですけど、x=sintと置換すれば普通に解けると思います。
普通に置換して解いてもいいのでしょうか。
それとも置換しないでlim(α→1)∫(0→α)としてと行って解かなければいけないのでしょうか。
また、置換しないで行う場合の解法を教えて欲しいです

https://i.imgur.com/PB0StXY.jpg

469 :132人目の素数さん:2018/08/01(水) 00:46:30.15 ID:ujmGY7/8.net
>>30
極限の中は普通のリーマン積分だし、置換積分していいよ
置換積分でなければ、xと2を分けて(1-x^2)^1/2を微分すると幸せになれそう

470 :132人目の素数さん:2018/08/01(水) 02:43:51.16 ID:Cawg9y0E.net
y=f(x)であるとき、常微分方程式

y" - y = exp(x)

の一般解はどのように求めたらいいですか?
特殊解が一つも見つからず困っています。

471 :132人目の素数さん:2018/08/01(水) 02:56:52.02 ID:8lbOLny2.net
>>470
Axe^x の形の特解をもつ

472 :132人目の素数さん:2018/08/01(水) 03:01:00.20 ID:Cawg9y0E.net
>>471
なるほど、確かに。未定係数法のテクニックみたいな物ですかね。
夜遅くにありがとうございます。

473 :132人目の素数さん:2018/08/01(水) 05:57:54.73 ID:b8S2BjTS.net
半径1の2つの円が外接している。それらの周及び外側の領域を、1辺の長さ1の正方形が、いずれの円の周にも接するように動く。
この正方形の周および内部が動いてできる領域は2つ存在するが、そのうちの1つの面積を求めよ。

474 :132人目の素数さん:2018/08/01(水) 10:38:33.36 ID:FslNfsBa.net
超一流の高校から超一流の大学に現役で入学したかった・・・・。

475 :132人目の素数さん:2018/08/01(水) 10:50:56.70 ID:FslNfsBa.net
でももう手遅れだから割とマジで自殺を視野に入れてる。

476 :132人目の素数さん:2018/08/01(水) 11:58:11.77 ID:VFdHBmYr.net
いつ自殺する予定なんですか?

477 :132人目の素数さん:2018/08/01(水) 11:59:40.48 ID:b8S2BjTS.net
>>475
自殺すると言う男性が実際に実行する確率は、個人によらずa(a≪1)であるという。
今後10000日で寿命を迎えるAさんについて、150日以内に自殺する確率Pと寿命を全うする確率Qをそれぞれaで表せ。
また確率の比P/Qの近似値を求めよ。

478 :132人目の素数さん:2018/08/01(水) 12:39:35.25 ID:idjJSs8a.net
https://i.imgur.com/SA1WQVS.png

円と放物線が接する条件を求める問題です。βが定数でrを問われています。
y>0という条件があるので、「yが正の重解」以外にも、
「正かつ0でない解と、負解を一つずつもつ」
でも、yが負の点は現実には現れないので、現実の作図には2点で接する状態になるのではないか?と思ったのですが、
どういう理屈で考慮せずともよいことになるのでしょうか?

実際の記述試験ではここを説明せず流してしまってもよいのでしょうか?

479 :132人目の素数さん:2018/08/01(水) 12:44:10.38 ID:v3TE18T6.net
>>470
定数係数線形常微分方程式の解法をそのまま使え

480 :132人目の素数さん:2018/08/01(水) 12:50:51.28 ID:v3TE18T6.net
>>478
「何を考慮せず」なのか分かる様に書け

481 :132人目の素数さん:2018/08/01(水) 13:13:36.34 ID:OqUs/T0w.net
ショボい質問ですまないが・・・・

コインを4回投げた時の出目は2*2*2*2で16通りだけど
組み合わせは5通りになる
この組み合わせを式で導く方法を教えてほしい

482 :132人目の素数さん:2018/08/01(水) 13:20:42.72 ID:VZ3sWgTl.net
2H4 = 5C4 = 5

483 :132人目の素数さん:2018/08/01(水) 13:23:18.35 ID:VZ3sWgTl.net
>>481
n 個のものから重複を許して r 個とる組合せの数(重複組合せ) n_H_r です。

n_H_r = (n+r-1)_C_r

が成り立ちます。

484 :132人目の素数さん:2018/08/01(水) 13:37:16.90 ID:7QVfCSfA.net
3の3乗根の2分の3乗分の1ってどうしてルート3分の1になるのですか?解説お願いします

485 :132人目の素数さん:2018/08/01(水) 13:40:48.71 ID:OqUs/T0w.net
>>482-483
ありがとうございました m(__)m

486 :132人目の素数さん:2018/08/01(水) 14:39:26.56 ID:cotyC2db.net
20分で70パーセントダウンロード完了してるコピーデータがある時、そのダウンロードが100パーセントになるのは何分かかるかわかる?

487 :132人目の素数さん:2018/08/01(水) 14:40:26.72 ID:2u/tfcdl.net
>>486
すいません。敬語使うべきでした。
わかる方おられましたら教えてください

488 :132人目の素数さん:2018/08/01(水) 14:41:07.25 ID:Kx2FTEDw.net
マイスター・エックハルトとレオンハルト・オイラーはどっちの方が凄いですか?

489 :132人目の素数さん:2018/08/01(水) 14:42:14.67 ID:zY4waAJF.net
発達障害だろ

490 :132人目の素数さん:2018/08/01(水) 14:44:14.59 ID:D0aYDpoH.net
1のπ乗根を複素平面上にプロットした時
その点は無限にあって半径1の円を描くでしょうか?但し(0,i)(−1,0)(0,−i)は除く

491 :132人目の素数さん:2018/08/01(水) 15:40:09.31 ID:b8S2BjTS.net
分かりづらい質問ですいません
任意の自然数a,bを用いた有理式で√2を挟むことを考えています

例えば
(1/a)+(1/3b) < √2 < (2ab+1)/a+b
は任意の自然数a,bに対して成り立ちます

このような式をA<√2<Bと表したとき、できる限りB-√2と√2-Aを小さくするにはどうしたらいいでしょうか。

492 :132人目の素数さん:2018/08/01(水) 16:15:30.88 ID:Kx2FTEDw.net
宇宙はチッコイですか?

493 :132人目の素数さん:2018/08/01(水) 16:34:06.08 ID:c2oWIv9N.net
お前の脳味噌ぐらいだ

494 :132人目の素数さん:2018/08/01(水) 16:42:40.02 ID:CziNQBVb.net
>>490
z^π=exp(πlog z)のlog zが一般の複素数zに定義できないからどういう風にlog zを定義するかに依る。

495 :132人目の素数さん:2018/08/01(水) 16:58:41.12 ID:lpAjVCDq.net
ハーバードかオックスフォードかケンブリッジかMITに入学したい。

496 :132人目の素数さん:2018/08/01(水) 16:59:08.53 ID:CziNQBVb.net
>>491
とりあえず
f_1=a、f_(n+1) = (f_n + 2/f_n)/2
とすれば
2/f_n < √2 < f_n
にはなるけど。
条件がそれだけならいくらでもありそう。

497 :132人目の素数さん:2018/08/01(水) 17:10:42.79 ID:lpAjVCDq.net
NASA長官とフランス共和国大統領はどっちの方が凄いですか?

498 :132人目の素数さん:2018/08/01(水) 17:18:48.55 ID:uLd5B6+9.net
発達障害のおっさん

499 :132人目の素数さん:2018/08/01(水) 17:22:19.12 ID:lpAjVCDq.net
天才になりたかった・・・・。

500 :132人目の素数さん:2018/08/01(水) 17:39:05.41 ID:lpAjVCDq.net
自殺したい。

501 :132人目の素数さん:2018/08/01(水) 19:57:04.38 ID:VZ3sWgTl.net
Int([a_1, b_1] × … × [a_n, b_n]) = (a_1, b_1) × … × (a_n, b_n)

を示せ。

502 :132人目の素数さん:2018/08/01(水) 20:00:08.83 ID:sbD88+5F.net
>>501
ごめん、分かりません。

503 :132人目の素数さん:2018/08/01(水) 20:12:34.93 ID:fUopbB/R.net
わからないんですね

504 :132人目の素数さん:2018/08/01(水) 20:32:35.77 ID:bW/c674g.net
俺もわかんない

505 :132人目の素数さん:2018/08/01(水) 20:34:58.37 ID:fUopbB/R.net
わからないんですね

506 :132人目の素数さん:2018/08/01(水) 21:15:45.86 ID:VZ3sWgTl.net
>>501

Int([a_1, b_1] × … × [a_n, b_n]) = (a_1, b_1) × … × (a_n, b_n)

を示せ。

x ∈ (a_1, b_1) × … × (a_n, b_n) とすると、

a_i < x < b_i for all i ∈ {1, …, n}

が成り立つ。

ε := min(min(x_1 - a_1, …, x_n - a_n), min(b_1 - x_1, …, b_n - x_n))

とおく。

B(x ; ε) ⊂ (a_1, b_1) × … × (a_n, b_n) が成り立つ:

y ∈ B(x ; ε) とする。

任意の i ∈ {1, …, n} に対して、

x_i - y_i ≦ |y_i - x_i| ≦ sqrt((y_1 - x_1)^2 + … + (y_n - x_n)^2) < ε ≦ x_i - a_i
y_i - x_i ≦ |y_i - x_i| ≦ sqrt((y_1 - x_1)^2 + … + (y_n - x_n)^2) < ε ≦ b_i - x_i

が成り立つ。


-y_i < -a_i, a_i < y_i
y_i < b_i


a_i < y_i < b_i


y ∈ (a_1, b_1) × … × (a_n, b_n)

以上より、

B(x ; ε) ⊂ (a_1, b_1) × … × (a_n, b_n) ⊂ [a_1, b_1] × … × [a_n, b_n] が成り立つ。


(a_1, b_1) × … × (a_n, b_n) ⊂ Int([a_1, b_1] × … × [a_n, b_n]) が成り立つ。

507 :132人目の素数さん:2018/08/01(水) 21:16:10.77 ID:VZ3sWgTl.net
逆に、

x ∈ Int([a_1, b_1] × … × [a_n, b_n]) とする。

x ∈ (a_1, b_1) × … × (a_n, b_n) ではないと仮定して矛盾を導く:

Int([a_1, b_1] × … × [a_n, b_n]) は定義により、 [a_1, b_1] × … × [a_n, b_n] に含まれるすべての R^n の開集合の和集合であるから、
Int([a_1, b_1] × … × [a_n, b_n]) ⊂ [a_1, b_1] × … × [a_n, b_n] が成り立つ。


x ∈ [a_1, b_1] × … × [a_n, b_n] - (a_1, b_1) × … × (a_n, b_n)


∃i ∈ {1, …, n} such that x_i = a_i or x_i = b_i

508 :132人目の素数さん:2018/08/01(水) 21:16:35.32 ID:VZ3sWgTl.net
x_i = a_i と仮定する。

Int([a_1, b_1] × … × [a_n, b_n]) は定義により、 [a_1, b_1] × … × [a_n, b_n] に含まれるすべての R^n の開集合の和集合であるから、
x ∈ U ⊂ [a_1, b_1] × … × [a_n, b_n] となるような R^n の開集合 U が存在する。
開集合の定義により、 B(x ; ε) ⊂ U となるような正の実数 ε が存在する。

y := (x_1, …, x_i - ε/2, …, x_n) とする。

ε/2 = sqrt((x_1 - x_1)^2 + … + (x_i - ε/2 - x_i)^2 + … + (x_n - x_n)^2) < ε であるから、

y ∈ B(x ; ε) ⊂ U ⊂ [a_1, b_1] × … × [a_n, b_n] である。

ところが、

y = (x_1, …, x_i - ε/2, …, x_n) = (x_1, …, a_i - ε/2, …, x_n) は [a_1, b_1] × … × [a_n, b_n] の元ではない。

これは矛盾である。

x_i = b_i と仮定する。

Int([a_1, b_1] × … × [a_n, b_n]) は定義により、 [a_1, b_1] × … × [a_n, b_n] に含まれるすべての R^n の開集合の和集合であるから、
x ∈ U ⊂ [a_1, b_1] × … × [a_n, b_n] となるような R^n の開集合 U が存在する。
開集合の定義により、 B(x ; ε) ⊂ U となるような正の実数 ε が存在する。

y := (x_1, …, x_i + ε/2, …, x_n) とする。

ε/2 = sqrt((x_1 - x_1)^2 + … + (x_i + ε/2 - x_i)^2 + … + (x_n - x_n)^2) < ε であるから、

y ∈ B(x ; ε) ⊂ U ⊂ [a_1, b_1] × … × [a_n, b_n] である。

ところが、

y = (x_1, …, x_i + ε/2, …, x_n) = (x_1, …, b_i + ε/2, …, x_n) は [a_1, b_1] × … × [a_n, b_n] の元ではない。

これは矛盾である。

509 :132人目の素数さん:2018/08/01(水) 21:19:13.95 ID:cVDrki+b.net
プレミアリーグ1チームの試合数計算したら
20チームがH&Aの総当たり戦
20C2の2倍で20チームで割ると答え出ると思うよね
なんか19試合になっておかしい
普通は38試合じゃん
理屈に合わない、どういうこと?マジで一日中悩んでる

510 :132人目の素数さん:2018/08/01(水) 21:23:31.75 ID:fUopbB/R.net
プレミアリーグは1992年にイングランドのプロサッカーリーグの改編に伴い、フットボールリーグから分離して新設された。20クラブが所属し、ホーム・アンド・アウェー方式による2回総当りで8月から翌年5月にかけて全38試合を戦う。

511 :132人目の素数さん:2018/08/01(水) 21:39:26.17 ID:0spl/5ES.net
>>509
1つのクラブから見た場合
自分以外の19クラブとホームで闘って19試合
自分以外の19クラブとアウェイで闘って19試合
合わせて38試合

これを試合総数から考えるとき、20C2×2を20で割ったのでは不都合
1つの試合を自分×相手と相手×自分の2回数える必要がある。
計算式は20P2×2÷20

512 :132人目の素数さん:2018/08/01(水) 21:40:28.94 ID:yK6pXi/o.net
チーム数じゃなくカード数の10で割ればいいんでね

513 :132人目の素数さん:2018/08/01(水) 21:42:34.65 ID:fUopbB/R.net
↑これが数学板の実力です↑
専門板なのに異常にレベルが低い
せいぜい数学の少しできる高校生レベル

514 :132人目の素数さん:2018/08/01(水) 21:52:55.73 ID:yK6pXi/o.net
NGID:fUopbB/R

515 :132人目の素数さん:2018/08/01(水) 22:49:34.39 ID:VZ3sWgTl.net
D が閉集合であるとき、 D と closure(Int(D)) の関係は一般にどんなものか?

516 :132人目の素数さん:2018/08/01(水) 22:52:22.93 ID:VZ3sWgTl.net
>>501

closure(Int([a_1, b_1] × … × [a_n, b_n])) = [a_1, b_1] × … × [a_n, b_n]

を示せ。

517 :132人目の素数さん:2018/08/02(木) 01:37:12.62 ID:ACGviExS.net
アンドリュー・ワイルズとNASAで最も賢い研究者はどっちの方が賢いですか?

518 :132人目の素数さん:2018/08/02(木) 01:52:55.15 ID:Fre00JeF.net
桃とスイカと梨を、全部で3個買います。
1つも選ばないものがあってもよいとすると、全部で何通りの選び方があるでしょう。

これを解く式を教えて下さい

519 :132人目の素数さん:2018/08/02(木) 01:57:03.58 ID:ACGviExS.net
全知全能の究極至高超絶絶頂極限神と無はどっちの方が凄いですか?

520 :132人目の素数さん:2018/08/02(木) 03:05:26.40 ID:3qObk5kB.net
>>518
シンプルな難問ですね
その店に合計n個の(桃、スイカ、梨)があるとして、
nC0からnC3までを足して、それを∫[0→3] x(1-x) dxで割ってください。
最後にnに3を代入してください

521 :132人目の素数さん:2018/08/02(木) 04:14:54.77 ID:XNOYdggT.net
>>496

a>0,
Max{a/√2,(√2)/a} = coth(2c),
とする。 n>1 に対して
f_n = (√2)coth((2^n)c) → √2, (n→∞)

522 :132人目の素数さん:2018/08/02(木) 04:22:48.67 ID:XNOYdggT.net
>>496
 g(x) = xx -2 にニュートン法を使ったでござるな。

>>521
倍角公式
 coth(2θ) = (1/2)(cothθ + 1/cothθ)

523 :132人目の素数さん:2018/08/02(木) 07:00:51.70 ID:n3f9mOLt.net
>>509
わからないときは数を減らして考えろ。
一試合に二つずつ出るんだから二倍しないと。

524 :132人目の素数さん:2018/08/02(木) 07:55:13.19 ID:GtVFRW6S.net
>>518
重複組み合わせでググれ

525 :132人目の素数さん:2018/08/02(木) 11:25:24.44 ID:5LqKc3SG.net
closure(Int([a_1, b_1] × … × [a_n, b_n])) = [a_1, b_1] × … × [a_n, b_n]

を示せ。

>>501

より

Int([a_1, b_1] × … × [a_n, b_n]) = (a_1, b_1) × … × (a_n, b_n)

であるから、

closure((a_1, b_1) × … × (a_n, b_n)) = [a_1, b_1] × … × [a_n, b_n]

を示せばよい。

x ∈ closure((a_1, b_1) × … × (a_n, b_n)) とする。

このとき、

x ∈ [a_1, b_1] × … × [a_n, b_n] ではないと仮定する。

仮定により、

x_i < a_i or b_i < x_i となるような i ∈ {1, …, n} が存在する。

526 :132人目の素数さん:2018/08/02(木) 11:25:44.97 ID:5LqKc3SG.net
x_i < a_i の場合を考える。

ε = a_i - x_i とおく。

y ∈ B(x ; ε) とする。

y_i - x_i ≦ sqrt((y_1 - x_1)^2 + … + (y_n - x_n)^2) < ε = a_i - x_i


y_i < a_i


y ∈ [a_1, b_1] × … × [a_n, b_n] ではない。


y ∈ (a_1, b_1) × … × (a_n, b_n) ではない。


x ∈ closure((a_1, b_1) × … × (a_n, b_n)) ではない。

これは仮定に矛盾する。

527 :132人目の素数さん:2018/08/02(木) 11:26:10.22 ID:5LqKc3SG.net
b_i < x_i の場合を考える。

ε = x_i - b_i とおく。

y ∈ B(x ; ε) とする。

x_i - y_i ≦ sqrt((y_1 - x_1)^2 + … + (y_n - x_n)^2) < ε = x_i - b_i


-y_i < -b_i, b_i < y_i


y ∈ [a_1, b_1] × … × [a_n, b_n] ではない。


y ∈ (a_1, b_1) × … × (a_n, b_n) ではない。


x ∈ closure((a_1, b_1) × … × (a_n, b_n)) ではない。

これは仮定に矛盾する。

528 :132人目の素数さん:2018/08/02(木) 11:26:29.13 ID:5LqKc3SG.net

closure((a_1, b_1) × … × (a_n, b_n)) ⊂ [a_1, b_1] × … × [a_n, b_n] である。

529 :132人目の素数さん:2018/08/02(木) 11:26:54.94 ID:5LqKc3SG.net
逆に、

x ∈ [a_1, b_1] × … × [a_n, b_n] とする。

ε を任意の正の実数とする。

i ∈ {1, …, n} に対し、

x_i = a_i のとき、
y_i ∈ (a_i, b_i) ∩ (a_i, a_i + ε/sqrt(n))

x_i = b_i のとき、
y_i ∈ (a_i, b_i) ∩ (b_i - ε/sqrt(n), b_i)

a_i < x_i < b_i のとき、
y_i ∈ (a_i, b_i) ∩ {(x_i - ε/sqrt(n), x_i) ∪ (x_i, x_i + ε/sqrt(n))}

とし、 y := (y_1, …, y_n) とする。

明らかに、

|y_i - x_i| < ε/sqrt(n) が成り立つ。

sqrt((y_1 - x_1)^2 + … + (y_n - x_n)^2) < sqrt(n * (ε/sqrt(n))^2) = ε

以上から、

任意の正の実数 ε に対し、

y ∈ B(x ; ε) ∩ (a_1, b_1) × … × (a_n, b_n)

となるような y が存在する。


x ∈ closure((a_1, b_1) × … × (a_n, b_n))

530 :132人目の素数さん:2018/08/02(木) 12:22:50.28 ID:RRXEEVFw.net
Int(A)=Aに含まれる最大の開集合=A-∂A
cl(A)=Aを含む最小の閉集合=A∪∂A

Dが閉集合なら
cl(Int(D))=D+∂D=D

531 :132人目の素数さん:2018/08/02(木) 12:49:57.92 ID:ttgSiwvk.net
E^2において一点集合Dは閉集合だが
closure(interior(D)) ≠ D

532 :132人目の素数さん:2018/08/02(木) 14:30:03.16 ID:3qObk5kB.net
任意の自然数a,bに対して
f(a,b)<√2<g(a,b)
かつ
「√2-f(a,b)<1/3 かつ g(a,b)-√2<1/3」
が成り立つような、定数でないa,bの有理式は存在しますか?

533 :132人目の素数さん:2018/08/02(木) 14:32:30.70 ID:/b0BSwha.net
>>529の y_i がいつでも取れるとは限らないってところがミスやね
a_i=b_i のとき、(a_i,b_i) は空

534 :132人目の素数さん:2018/08/02(木) 14:35:14.74 ID:5LqKc3SG.net
>>515

closure(Int(D)) ⊂ D

closure(Int(D)) は D からその孤立点を除去した集合。

535 :132人目の素数さん:2018/08/02(木) 15:07:10.85 ID:CON1WNYv.net
R×RにおいてD=R×{0}に孤立点はないが、closure(interior(D))は空集合。

536 :132人目の素数さん:2018/08/02(木) 15:12:06.82 ID:3qObk5kB.net
立方体の4頂点を結び正四面体Vを作る。
またVをある直線の周りに一回転させてできる立体をWとする。
立方体とWの共通部分の体積が最大となる直線のとり方を説明せよ。

537 :132人目の素数さん:2018/08/02(木) 15:15:14.81 ID:CON1WNYv.net
>>532
f(a,b) = 1/(a^2+b^2+1) * 0.00000000001 + 1.4142
g(a,b) = -1/(a^2+b^2+1) * 0.00000000001 + 1.4143
とか

538 :132人目の素数さん:2018/08/02(木) 15:21:40.76 ID:5LqKc3SG.net
>>515

closure(Int(D)) ⊂ D - {D の孤立点}

539 :132人目の素数さん:2018/08/02(木) 15:25:17.93 ID:CON1WNYv.net
>>536
最大値なし

540 :132人目の素数さん:2018/08/02(木) 15:27:28.31 ID:CON1WNYv.net
Xが離散位相空間のときXのすべての点は孤立点でclosure(interior(X)) = X。

541 :132人目の素数さん:2018/08/02(木) 15:48:20.74 ID:5LqKc3SG.net
>>515

closure(Int(D)) ⊂ D

542 :132人目の素数さん:2018/08/02(木) 16:05:46.15 ID:5LqKc3SG.net
Throughout, let X be a metric space with metric d

If U is an open set, what is the relation in general between
the set U and the interior of closure(U) ?

543 :132人目の素数さん:2018/08/02(木) 16:33:38.99 ID:5LqKc3SG.net
>>542

X = R

U = Int(U) かつ U ⊂ closure(U)

U = Int(U) ⊂ Int(closure(U))

U = (-1, 0) ∪ (0, 1)

closure(U) = [-1, 1]

Int(closure(U)) = (-1, 1) ≠ U

544 :132人目の素数さん:2018/08/02(木) 16:34:14.94 ID:5LqKc3SG.net
>>542

U = Int(U) かつ U ⊂ closure(U)

U = Int(U) ⊂ Int(closure(U))



--------------------------------
X = R

U = (-1, 0) ∪ (0, 1)

closure(U) = [-1, 1]

Int(closure(U)) = (-1, 1) ≠ U

545 :132人目の素数さん:2018/08/02(木) 16:38:39.71 ID:5LqKc3SG.net
Let f : X → Y. Show that f is continuous if and only if for each x ∈ X there is a neighborhood U of x such that
f|U is continuous.

546 :132人目の素数さん:2018/08/02(木) 17:14:57.14 ID:3qObk5kB.net
>>539
上限は立方体の体積に等しい?

547 :132人目の素数さん:2018/08/02(木) 17:57:27.34 ID:ttgSiwvk.net
直線と四面体の位置に制限がない。

548 :132人目の素数さん:2018/08/02(木) 17:59:57.09 ID:ttgSiwvk.net
いや、最大値あるね、
なんか、デタラメ詰将棋系くさいけど

549 :132人目の素数さん:2018/08/02(木) 18:11:39.89 ID:hISRJwlg.net
>>26
この問題の(1)について、
共有点がちょうど3つ存在する
⇔2つの放物線がx軸より下で接する
だと思ったのですが、
この予想の正否が分かる方はいますか?
おそらく、>>104さんはこの予想が正しいことを前提に解かれているのだと思いますが。
ちなみに私は高校3年生です。

550 :132人目の素数さん:2018/08/02(木) 18:33:36.64 ID:ttgSiwvk.net
>>549
それは明らかに正しいけどこの問題がデタラメ詰将棋系だと思われてるのはそこじゃない。
面積の小さい方がa.bのあたいに応じて変化して、片っ方はなんとかいけるけど、もう片方が全然綺麗な式にならん。
出題してるやつは多分山勘で綺麗に出る方の面積が小さいと思い込んでるんだと思う。

551 :132人目の素数さん:2018/08/02(木) 19:06:58.52 ID:hISRJwlg.net
ああ、y=(x-a)^2+bとx=y^2からxを消去した
4次方程式y=(y^2-a)^2+bが異なる3つの実数解を持つことが条件か
(x=y^2のグラフ上でy座標が同じになることはないため、
異なる実数解yの個数 = 共有点の個数)。
さらに、4次方程式の場合、
異なる3つの実数解 ⇔ 重解1つと異なる2実数解になるのか。
ただ、重解 ⇔ 接するが成り立つかは非自明だな。

552 :132人目の素数さん:2018/08/02(木) 21:40:05.97 ID:dgr6zwfG.net
>>470
知識ゼロの状態からだと

y''-y=0を解くとy=Ae^x+Be^(-x)
y=Ae^x+Be^(-x)+Ce^(αx)とおいてもうまくいかないから
y=Ae^x+Be^(-x)+u(x)e^(αx)とおくと
u''e^(αx)+2u'αe^(αx)+u(α^2)e^(αx)-ue^(αx)=e^x
⇔{u''+2u'α+u(α^2)-u}e^(αx)=e^x
よってα=1, u''+2u'=1

u''+2u'=0を解くとDe^(-2x)+E
u=De^(-2x)+E+Fx+Gとおくと
0+2F=1⇔F=1/2

以上より
y=Ae^x+Be^(-x)+(De^(-2x)+E+(1/2)x+G)e^x
=(A+E+G)e^x+(B+D)e^(-x)+(1/2)xe^x
=He^x+Ie^(-x)+(1/2)xe^x

解けたから満足

553 :132人目の素数さん:2018/08/02(木) 22:12:23.26 ID:KFEJtiya.net
ハーバード大学に首席合格したい。

554 :132人目の素数さん:2018/08/02(木) 23:19:12.80 ID:3qObk5kB.net
実数の列{a_n}は任意の自然数p,qに対して
|a_(p+q)-a_p-a_q|<1
を満たしている。
このとき、任意の自然数n,kに対して
|n*a_(n+k)-(n+k)a_n|<2(n+k)…(A)
が成り立つことを示せ。

追加問題
(A)をより厳しく評価せよ。
すなわち、任意のn,kに対して(A)の右辺を可能な限り小さくせよ。

555 :132人目の素数さん:2018/08/02(木) 23:21:49.44 ID:aXLu90aQ.net
失礼します
どうして停留点を求めた時以下の式になるのでしょう?

https://i.imgur.com/UwZNLM3.jpg

556 :132人目の素数さん:2018/08/02(木) 23:45:01.99 ID:xnKNqrPM.net
次の山型の数列にパルカスの三角形のような規則性って何かあるでしょうか。

1
0, 0
1, 0, 1
0, 1, 1, 0
1, 0, 4, 0, 1
0, 2, 3, 3, 2, 0
1, 0, 9, 2, 9, 0, 1
0, 3, 6, 12, 12, 6, 3, 0
1, 0, 16, 8, 36, 8, 16, 0, 1
0, 4, 10, 30,41, 41, 30, 10, 4, 0
・・・・・・

557 :132人目の素数さん:2018/08/03(金) 00:07:50.44 ID:EmlLNZvo.net
>>554
|pa_q - qa_p| < p+q …(※)
を示せば十分。
I) max{p,q} = 1のとき。
p = q = 1だから左辺=0より成立。
II) max{p,q}<k で成立と仮定して max{p,q} = k とする。
p=qなら左辺=0より成立。
q>pのとき r=q-p とおく。
|pa_q - qa_p|
=|p(a_q - a_p - a_r) + r a_p - p a_r|
≦p|a_q - a_p - a_r| + |r a_p - p a_r|
<p + r + p
=p+q。

558 :132人目の素数さん:2018/08/03(金) 01:24:35.22 ID:tsbZKQGa.net
半径4の円Cに半径1の円Dが外接している。
Dは反時計回りにC上を滑ることなく転がり、はじめにCと接していた点であるD上の点Pが再びCと接したところで停止する。
点Pが描く曲線とCで囲まれる領域をKとする(KはCの外部である)。
Kに含まれる線分のうち最長のものをLとするとき、以下の問いに答えよ。

(1)以下の(a),(b)の真偽を判定せよ。
(a)Lは点Pが描く曲線と共有点を持つ
(b)LはCと共有点を持つ

(2)Lの長さを求めよ。

559 :132人目の素数さん:2018/08/03(金) 01:58:19.46 ID:oE4kF5bF.net
1点aで複素微分可能でも、その点で正則(aのある開近傍Uが存在し、fはU上で正則)でないことはありますか?

560 :132人目の素数さん:2018/08/03(金) 01:59:34.05 ID:tRRMlHHD.net
>>556

B(2n,2r) = C(n,r)^2

あとはよく分からん。あべのパルカス

561 :132人目の素数さん:2018/08/03(金) 03:08:40.31 ID:SxDX8OCS.net
>>559
f(z)=0 (z=0またはzが無理数)
1/n(Re(z)が有理数で、既約分数表示した時の分母がn)

とすると、f(z)はz=0で連続かつ微分可能ですが、z=0の任意の開近傍はRe(z)が有理数となる点が存在し、その点では連続ではなく微分可能でもありません

562 :132人目の素数さん:2018/08/03(金) 03:36:51.46 ID:oE4kF5bF.net
>>561
ありがとうございます
f'(0)の値は何ですか?

563 :132人目の素数さん:2018/08/03(金) 10:52:25.49 ID:SxDX8OCS.net
0ですね

564 :132人目の素数さん:2018/08/03(金) 13:40:57.94 ID:Pirwc60W.net
∫[0→1] 1/((1+x^3)^(1/3)) dxを教えて下さい

565 :132人目の素数さん:2018/08/03(金) 13:48:02.99 ID:6rYEsJmV.net
1,1,1,2で10を作ってください。
(この種の問題はご存知かと思いますが、文字同士は必ず演算を用い、1と2を1回ずつ使って12とするなどはやめてください。)
四則演算では不可能であると判明したので、そのほかの演算を適宜使ってください。

566 :132人目の素数さん:2018/08/03(金) 13:57:19.25 ID:tsbZKQGa.net
>>565
二項演算★を以下のように定義する

実数a,b(a<b)に対して
a★a=0
a★b=10
式中では加算記号と減算記号に優先する

すると
1★1+1★2=0+10=10

567 :132人目の素数さん:2018/08/03(金) 13:58:58.30 ID:iL59RBCB.net
>>566
わかりやすい

568 :132人目の素数さん:2018/08/03(金) 14:48:15.88 ID:uEg3jUPY.net
>>565
10 = [tan log |log √√√√√√√√√√√√√√√√√√ sin 1)|] + 1 + 1 - 2

569 :132人目の素数さん:2018/08/03(金) 14:52:18.39 ID:9XotD0/A.net
>1と2を1回ずつ使って12

この種の問題では当然に許可すべき

570 :132人目の素数さん:2018/08/03(金) 16:41:08.70 ID:mQsg6A/0.net
ヘッセ行列とは、勾配ベクトルをベクトル微分したもの、という解釈でも良いんでしょうか?

571 :132人目の素数さん:2018/08/03(金) 17:17:18.39 ID:kYekzqNA.net
>>565
(1+1+1)!/tanh(ln(2))
Binomial[CatalanNumber[1+1+1],2]

572 :132人目の素数さん:2018/08/03(金) 19:02:27.57 ID:tsbZKQGa.net
>>558
傑作です。

573 :132人目の素数さん:2018/08/03(金) 19:36:04.71 ID:BwKQdpjH.net
>>545

Let f : X → Y. Show that f is continuous if and only if for each x ∈ X there is a neighborhood U of x such that
f|U is continuous


f : X → Y が連続であると仮定する。
任意の x ∈ X に対して、 X は x の近傍であり、 f|X = f は連続である。

逆に、任意の x ∈ X に対して、 f|U が連続であるような x の近傍 U が存在すると仮定する。

x0 を X の任意の元とする。仮定により、 f|U が連続であるような x0 の近傍 U が存在する。

f|U は x0 で連続だから、任意の正の実数 ε に対して、

d_U(x, x0) < δ ⇒ d_Y(f(x), f(x0)) < ε

となるような正の実数 δ が存在する。

U は X の開集合だから、 {x ∈ X | d_X(x, x0) < δ1} ⊂ U となるような正の実数 δ1 が存在する。

δ2 := min(δ, δ1) とおく。

{x ∈ X | d_X(x, x0) < δ2} ⊂ U だから、

{x ∈ X | d_X(x, x0) < δ2} = {x ∈ U | d_U(x, x0) < δ2} である。

d_U(x, x0) < δ2 ⇒ d_Y(f(x), f(x0)) < ε だから、

d_X(x, x0) < δ2 ⇒ d_Y(f(x), f(x0)) < ε である。


f : X → Y は連続である。

574 :132人目の素数さん:2018/08/03(金) 19:47:51.24 ID:BwKQdpjH.net
Let X = A ∪ B, where A and B are subspaces of X. Let f : X → Y;
suppose that the restricted functions

f|A : A → Y and f|B : B → Y

are continuous. Show that if both A and B are closed in X, then f is continuous

575 :132人目の素数さん:2018/08/03(金) 20:31:49.37 ID:3idna+6E.net
Stupid guy

576 :132人目の素数さん:2018/08/03(金) 20:33:25.53 ID:QRGAtQK1.net
Go hang yourself.

577 :132人目の素数さん:2018/08/03(金) 21:52:47.16 ID:BwKQdpjH.net
杉浦光夫著『解析入門I』を読んでいます。

p.170

例6
例7

におかしなところがあります。

例えば、例6ですが、


Σz^(2*n) / (2*n)! では、 a_(2*n+1) = 0, a_(2*n) = 1 / (2*n)! であり、
この場合 lim |a_n / a_(n+1)| は存在しない。


と書いてありますが、数列 {a_n / a_(n+1)} 自体が定義できないわけですから、

lim |a_n / a_(n+1)| も定義できないわけです。存在するしない以前の問題です。

578 :132人目の素数さん:2018/08/03(金) 22:34:42.40 ID:asTwelNd.net
NASAで最も賢い研究者と、科挙(一番難しい時代の)に一発且つ首席且つ最年少で合格した人はどっちの方が賢いですか?

579 :132人目の素数さん:2018/08/03(金) 22:48:03.01 ID:BwKQdpjH.net
>>574

Let X = A ∪ B, where A and B are subspaces of X. Let f : X → Y;
suppose that the restricted functions

f|A : A → Y and f|B : B → Y

are continuous. Show that if both A and B are closed in X, then f is continuous


X = (A - B) ∪ (B - A) ∪ (A ∩ B) (直和) である。

(A - B) ∪ B = X かつ B は X の閉集合だから、 A - B は X の開集合である。
(B - A) ∪ A = X かつ A は X の閉集合だから、 B - A は X の開集合である。
A, B は X の閉集合だから、 A ∩ B は X の閉集合である。

x0 を X の任意の元とする。

(1) x0 ∈ A - B の場合
(2) x0 ∈ B - A の場合
(3) x0 ∈ A ∩ B の場合

に場合分けして考える。

580 :132人目の素数さん:2018/08/03(金) 22:48:34.70 ID:BwKQdpjH.net
(1) x0 ∈ A - B の場合

A - B は X の開集合だから、 {x ∈ X | d_X(x, x0) < δ1} ⊂ A - B となるような正の実数 δ1 が存在する。

ε を任意の正の実数とする。

f|A : A → Y は連続だから、

d_A(x, x0) < δ2 ⇒ d_Y(f(x), f(x0)) < ε となるような正の実数 δ2 が存在する。

δ := min(δ1, δ2) とおく。

{x ∈ X | d_X(x, x0) < δ} ⊂ A - B ⊂ A だから、

{x ∈ X | d_X(x, x0) < δ} = {x ∈ A | d_A(x, x0) < δ} である。

また d_A(x, x0) < δ ⇒ d_Y(f(x), f(x0)) < εである。


d_X(x, x0) < δ ⇒ d_Y(f(x), f(x0)) < εである。


f : X → Y は連続である。

581 :132人目の素数さん:2018/08/03(金) 22:48:53.59 ID:BwKQdpjH.net
(2) x0 ∈ B - A の場合

B - A は X の開集合だから、 {x ∈ X | d_X(x, x0) < δ1} ⊂ B - A となるような正の実数 δ1 が存在する。

ε を任意の正の実数とする。

f|B : B → Y は連続だから、

d_B(x, x0) < δ2 ⇒ d_Y(f(x), f(x0)) < ε となるような正の実数 δ2 が存在する。

δ := min(δ1, δ2) とおく。

{x ∈ X | d_X(x, x0) < δ} ⊂ B - A ⊂ B だから、

{x ∈ X | d_X(x, x0) < δ} = {x ∈ B | d_B(x, x0) < δ} である。

また d_B(x, x0) < δ ⇒ d_Y(f(x), f(x0)) < εである。


d_X(x, x0) < δ ⇒ d_Y(f(x), f(x0)) < εである。


f : X → Y は連続である。

582 :132人目の素数さん:2018/08/03(金) 22:49:10.25 ID:BwKQdpjH.net
(3) x0 ∈ A ∩ B の場合

ε を任意の正の実数とする。

f|A : A → Y
f|B : B → Y

は連続だから、

d_A(x, x0) < δ1 ⇒ d_Y(f(x), f(x0)) < ε
d_B(x, x0) < δ2 ⇒ d_Y(f(x), f(x0)) < ε

となるような δ1, δ2 が存在する。

δ := min(δ1, δ2) とおく。

明らかに、

{x ∈ A | d_A(x, x0) < δ} ∪ {x ∈ B | d_B(x, x0) < δ} = {x ∈ X | d_X(x, x0) < δ}

である。

x ∈ {x ∈ A | d_A(x, x0) < δ} ⇒ d_Y(f(x), f(x0)) < ε
x ∈ {x ∈ B | d_B(x, x0) < δ} ⇒ d_Y(f(x), f(x0)) < ε

であるから、

x ∈ {x ∈ X | d_X(x, x0) < δ} ⇒ d_Y(f(x), f(x0)) < ε

である。


f : X → Y は連続である。

583 :132人目の素数さん:2018/08/03(金) 23:04:19.67 ID:Do86lxx7.net
17万円のうちの2万円って何パーセント?

584 :132人目の素数さん:2018/08/03(金) 23:53:13.70 ID:Qc3hZ1MO.net
科挙一発首席最年少合格者とマキシム・コンツェビッチはどっちの方が賢い?

585 :132人目の素数さん:2018/08/04(土) 01:17:58.33 ID:h2IIZ7/S.net
p匹の動物をグループ分けする。
1グループあたりq匹とし、rグループに分ける。
このとき何種類の組み合わせができるか。

動物はそれぞれ識別できるものとする。
グループは区別ができないものとする。
グループ分けの際に動物が余る場合もある。

586 :132人目の素数さん:2018/08/04(土) 02:27:29.73 ID:O25WHJ4j.net
>>585
「グループ分けの際に動物が余ることもある」をもうちょっと厳密に説明してくれ

587 :132人目の素数さん:2018/08/04(土) 02:31:58.92 ID:O25WHJ4j.net
m,nを自然数とする。
n個の箱にmn個のボールをでたらめに投げ入れる。

ボールが1つも入っていない箱が2箱できる確率p(m,n)をmとnで表せ。

また極限
lim[m→∞] p(m,n)/p(m+1,n)
を求めよ。

588 :132人目の素数さん:2018/08/04(土) 02:50:50.77 ID:ZD/Bfk7m.net
>>564
x = e^(-t) とおく。
(与式) = ∫[0,∞] 1/{1+e^(3t)}^(1/3) dt
= 0.937706990575338860724827668651595924153015324648714996550033613124860660612273505440145297648734359

>>576
「逝ってよし」

>>583
 11.7647 %

589 :132人目の素数さん:2018/08/04(土) 03:15:11.12 ID:ZD/Bfk7m.net
>>565

[ √{2/tan(1゚)} ] *1 *1
[ √{1/tan(1゚)} ] +1 +2
[ √{1/tan(2゚)} ] *(1+1)

590 :132人目の素数さん:2018/08/04(土) 03:27:27.24 ID:ZD/Bfk7m.net
>>565

[ exp(1+1+1) ] ÷ 2
[ exp(2+1) ] ÷ (1+1)
[ exp(2) ] +1 +1 +1
[ exp(1) + exp(1) + exp(1) ] +2

591 :132人目の素数さん:2018/08/04(土) 05:51:07.04 ID:Tryplpe/.net
>>577
定義できるけど存在しない
ってどういう状況なのかね?

592 :585:2018/08/04(土) 06:49:42.46 ID:h2IIZ7/S.net
>>586
p≧q×rという意味です。
たとえば9匹のマウスを、1グループあたり2匹ずつ、3グループに分けると3匹余ります。

他の条件としては下の2つがあります。
q≧1の自然数
r≧1の自然数

p:動物の総数
q:グループの数
r:1グループあたりの動物の匹数

593 :132人目の素数さん:2018/08/04(土) 08:03:29.14 ID:W7N0ST8g.net
>>587
C[n,2]((1-2/n)^mn - C[n - 2,1](1-3/n)^n + ‥)/C[n,2] ry

594 :132人目の素数さん:2018/08/04(土) 08:50:38.76 ID:LrWC+2Ba.net
>「連続体仮説は証明も反証もできない命題である」ということが明確に証明されている。

連続体仮説以外にこのような命題の例ってあるのでしょうか?

595 :132人目の素数さん:2018/08/04(土) 09:47:14.96 ID:5HaTOLT5.net
まず選択公理が浮かばないのはどうなん

596 :132人目の素数さん:2018/08/04(土) 10:23:33.07 ID:5kgzeyFd.net
>>450
解答が汚い
↓以下のようにシンプルに解答できる
v∈Vは、v=f(g(v))+(v-f(g(v)))と書ける。
g(v-f(g(v)))=g(v)-g(f(g(v))=g(v)-g(v)=0だからv-f(g(v))∈Ker(g)

v=f(u)∈Im(f)∩Ker(g)とすると、0=g(v)=g(f(u))=u。よって、v=f(u)=f(0)=0

597 :132人目の素数さん:2018/08/04(土) 10:24:06.37 ID:g/wipP8S.net
中学1年生数学の問題

丸暗記は簡単だが意味がわらからないので詳しく教えて
https://i.imgur.com/ttp1WG4.jpg

598 :132人目の素数さん:2018/08/04(土) 10:29:09.65 ID:5kgzeyFd.net
>>468
非積分函数=(1-x^2)^{-1/2}*(-2*x)*(-1/2) + 2*1/√(1-x^2)

599 :132人目の素数さん:2018/08/04(土) 10:53:41.44 ID:VblSSaDK.net
Let f : X → Y and g : Y → Z
Let x_0 be a limit point of X and
let y_0 be a limit point of Y

Consider the following three conditons:

(i) f(x) → y_0 as x → x_0
(ii) g(y) → z_0 as y → y_0
(iII) g(f(x)) → z_0 as x → x_0

(a) Give an example where (i) and (ii) hold, but (iii) does not

600 :132人目の素数さん:2018/08/04(土) 11:04:28.97 ID:VblSSaDK.net
>>599

f : R → R
f(x) = 0 for all x ∈ R

g : R → R
g(y) = 0 for all y ∈ R - {0}
g(0) = 1

f(x) → 0 as x → 0
g(y) → 0 as y → 0

but

g(f(x)) → 1 as x → 0

601 :132人目の素数さん:2018/08/04(土) 11:05:41.27 ID:VblSSaDK.net
Let f : X → Y and g : Y → Z
Let x_0 be a limit point of X and
let y_0 be a limit point of Y

Consider the following three conditons:

(i) f(x) → y_0 as x → x_0
(ii) g(y) → z_0 as y → y_0
(iII) g(f(x)) → z_0 as x → x_0

(b) Show that if (i) and (ii) hold and if g(y_0) = z_0, then (iii) holds.

602 :132人目の素数さん:2018/08/04(土) 11:14:51.17 ID:9bRCG7ss.net
>>597
どっちでも良いです

603 :132人目の素数さん:2018/08/04(土) 11:36:56.24 ID:yqmN+2Wy.net
>>602
有難うございます

そうするとこの問題自体が問題として成り立っていないと言うことでしょうか?

604 :132人目の素数さん:2018/08/04(土) 11:39:20.41 ID:9bRCG7ss.net
>>603
そのまま答え書くわけにもいきませんから、もし聞かれたらそういう風に書き直せば良いでしょう

605 :132人目の素数さん:2018/08/04(土) 12:55:36.96 ID:O25WHJ4j.net
平面上に直方体Tが置かれ、その辺の長さの比はa:b:cであるという。
平面上にあるTの一つの頂点を選び、そこを通る平面上の直線lを考える。
lの周りにTを一回転させてできる立体の体積をV_lとし、lを色々変化させるとき、体積比
{min(V_l)}/{Max(V_l)}
の値を求めよ。

606 :132人目の素数さん:2018/08/04(土) 13:07:06.32 ID:K+zoubgx.net
>>594
古いところではユークリッド幾何の平行線公理

607 :132人目の素数さん:2018/08/04(土) 13:07:35.07 ID:O25WHJ4j.net
空間の平面z=0上に円Cがある。
円Cの内部の点P(x,y,0)における方べきの値をz_Pとし、点Q(x,y,z_P)を考える。
PをC内で動かすとき、Qが動いてできる図形は回転放物面の一部であることを示せ。

(補足)本問で点Pにおける方べきの値とは、Pを通るある直線とCとの2つの交点をA,Bとしたときの、線分長の積PA・PBのことである。

608 :132人目の素数さん:2018/08/04(土) 13:42:57.77 ID:BZ9gQcLz.net
z_P = r^2 - |OP|^2

609 :132人目の素数さん:2018/08/04(土) 13:47:38.51 ID:O25WHJ4j.net
文字列A:aaaaaaに対し、以下の操作『T』を繰り返し行う。

『T』:文字列Aから1つの文字を無作為に選ぶ。
それが「a」であるならば「bb」に置き換え、それが「b」であるなら削除する。

Tをn回行ったときの文字列Aの長さの期待値をE(n)とするとき、以下の極限が収束するかどうかを述べよ。
収束する場合はその極限値を述べよ。
lim[n→∞] E(n)

(補足)
『T』を2回行って、1回目では左から2番目の「a」が選ばれ、2回目でら左から3番目の「b」が選ばれた場合、
aaaaaa→abbaaaa→abaaaa
となる。

610 :132人目の素数さん:2018/08/04(土) 14:21:27.87 ID:BZ9gQcLz.net
文字列きえたらT行えないじゃん
aaaaaa→bbbbbbbbbbbb→空

611 :132人目の素数さん:2018/08/04(土) 14:42:44.97 ID:5MQrEZdu.net
2a-1<√x<3a-1 a,x∈N を満たすxが17個の時のaの値

612 :132人目の素数さん:2018/08/04(土) 15:28:04.60 ID:VblSSaDK.net
>>601

W を z_0 = g(y_0) を含む Z の任意の開集合とする。

g(y) → z_0 as y → y_0 だから

y_0 を含むような Y の開集合 V で、

g(V - {y_0}) ⊂ W

となるようなものが存在する。

g(y_0) = z_0 ∈ W だから、

g(V) ⊂ W

である。

f(x) → y_0 as x → x_0 だから

x_0 を含むような X の開集合 U で、

f(U - {x_0}) ⊂ V

となるようなものが存在する。

g(f(U - {x_0}) ⊂ g(V) ⊂ W

であるから、

g(f(x)) → z_0 as x → x_0

である。

613 :132人目の素数さん:2018/08/04(土) 15:53:29.03 ID:O25WHJ4j.net
>>610
長さ0

614 :132人目の素数さん:2018/08/04(土) 15:54:00.85 ID:ZD/Bfk7m.net
>>564

∫[0→x] 1/{(1+x'^3)^(1/3)} dx' = x・F(1/3,1/3;4/3|-x^3)

F(a,b;c|z) は超幾何級数 (hyper-geometric function)

615 :132人目の素数さん:2018/08/04(土) 15:56:12.93 ID:VblSSaDK.net
Let f : R → R be defined by setting f(x) = sin(x) if x is rational, and f(x) = 0 otherwise.
At what points is f continuous?

616 :132人目の素数さん:2018/08/04(土) 16:01:10.91 ID:VblSSaDK.net
>>615

{π * n | n ∈ Z}

617 :132人目の素数さん:2018/08/04(土) 16:44:37.75 ID:W7N0ST8g.net
>>613
そんな事聞いてんじゃない。文字選べなかったらTは出来ないと言ってる。問題文の吟味が雑いんだよ。

618 :132人目の素数さん:2018/08/04(土) 17:20:50.50 ID:EVl9uXLt.net
>>604
これ中学の定期テストの問題なんです。

なぜ、まだ計算できる状態の式に戻すのか、それが「式を簡単にする」事なのか。
どうやっても腑に落ちません
https://i.imgur.com/ttp1WG4.jpg

619 :132人目の素数さん:2018/08/04(土) 17:25:19.72 ID:xpgQhtt/.net
>>618
先生に聞け
それか問題用紙の実物を画像で上げろ

620 :132人目の素数さん:2018/08/04(土) 17:41:18.25 ID:9bRCG7ss.net
>>618
処世術ってやつですよ
そういうもんなんだな、でいいじゃないですか
そのまま問題と同じ答え書いても意味ないんですから、何かしら書き換えないといけないわけです

621 :132人目の素数さん:2018/08/04(土) 18:01:20.95 ID:Ubw8oDop.net
>>618
その教師の、数式における項の書き順や符号の位置へのこだわりなのだろう。

622 :132人目の素数さん:2018/08/04(土) 18:43:34.15 ID:VblSSaDK.net
4つの面に 1 から 4 の数字の書かれた正四面体の形をしたサイコロがある。
このサイコロを5回振る。

i回目に出た目の数を i の位の数とし、5桁の整数 X を作る。

X に数字1と数字2がちょうど1回ずつ現れる確率を求めよ。

この問題に対する解答ですが、以下のように書いてあります:

解答:


5回の操作で、数字1と数字2がちょうど1回ずつ現れる場合の数は
Binomial(5, 1) * Binomial(4, 1) = 20 (通り)あり、


5回の操作で、数字1と数字2がちょうど1回ずつ現れる場合の数は、

Binomial(5, 1) * Binomial(4, 1) * 2^3

ですよね?上の解答は間違っていますよね?

ちなみに上の解答はチャート式の赤いやつの解答です。

ひどい参考書です。

623 :132人目の素数さん:2018/08/04(土) 18:49:04.16 ID:dnhPuN6B.net
8 8 3 5
で10にする。って問題です。
よろしくお願い申し上げます。

624 :132人目の素数さん:2018/08/04(土) 18:53:04.29 ID:VblSSaDK.net
ところで、質問ですが、

独立試行、反復試行の確率というのがあります。

これらの確率はすべて、

事象 A の起こる場合の数 / 起こりうるすべての場合の数

で計算できます。

なぜ各試行の確率の積でわざわざ計算するのでしょうか?

同じことですよね?

625 :132人目の素数さん:2018/08/04(土) 19:13:48.34 ID:EVl9uXLt.net
>>619
>>620
>>621
ありがとうございます

結局のところ深く考えても仕方ないのですね
担当教師に聞いてみます

626 :132人目の素数さん:2018/08/04(土) 19:20:56.40 ID:xpgQhtt/.net
8+√(8/(5-3))=10

627 :132人目の素数さん:2018/08/04(土) 19:26:56.19 ID:dnhPuN6B.net
>>626
ありがとうございます。

628 :132人目の素数さん:2018/08/04(土) 19:51:31.81 ID:U/6JsNI2.net
ちなみに>>585
(tCm×(t-m)Cm×...×(t-m(g-1))Cm)/g!
であってますか?

629 :132人目の素数さん:2018/08/04(土) 19:57:33.63 ID:W7N0ST8g.net
合ってない

630 :132人目の素数さん:2018/08/04(土) 20:18:41.68 ID:U/6JsNI2.net
失礼しました!
(pCq×(p-q)Cq×...×(p-q(r-1))Cq)/r!
でした!

631 :132人目の素数さん:2018/08/04(土) 20:38:53.74 ID:W7N0ST8g.net
合ってる希ガス

632 :132人目の素数さん:2018/08/04(土) 20:49:45.56 ID:rFWXxMVs.net
>>631
ありがとうございます!
数年ぶりの数学だったので助かりました!

633 :132人目の素数さん:2018/08/04(土) 21:20:57.66 ID:W7N0ST8g.net
>>609
の問題どう操作を選んでも18回でから文字列に到達するから答えは0なんだけど、
問題勘違いしてΣE(n)計算してた。
どうもaがx文字、bがy文字からスタートするとΣE(n)は
5/2*x^2 +3/2*x + (2*x+1)*y + y*(y-1)/2-x
になるみたい。整数値。ちょっと面白かった。

634 :132人目の素数さん:2018/08/05(日) 00:38:46.00 ID:xMCzGduU.net
プログラムを作る上で数学の知識が必要なのですが
rの値を求める式を作りたいです

https://i.imgur.com/SLSDdGo.png

Hは定数です

gとWはプログラムによって状況ごとに変わるのですが
いずれにしても絶対に直角三角形の形になります
なのでピタゴラスの定理が使えると思うのですが
頭が悪くてr=の式に治すことが出来ません
(右辺からrを無くして、左辺はrだけにしたい)

どなたか解ける人いますでしょか

635 :132人目の素数さん:2018/08/05(日) 00:56:36.59 ID:Kh2s9L1e.net
>>634
(g-1)^2を両辺にかけてまとめると
r^2×(g^2-2g)+2hr-h^2-w^2×(g-1)^2=0
となるので二次方程式の解の公式からrについて解くって感じじゃないでしょうか

636 :132人目の素数さん:2018/08/05(日) 02:11:48.29 ID:rWEeASLy.net
>>611
 2a-1 ≦ √x ≦ 3a-1,
x は (2a-1)^2 から (3a-1)^2 まで。
5aa -2a +1 = 17
 (a-2)(5a+8) = 0
 5a+8>0 ゆえ a=2.

637 :132人目の素数さん:2018/08/05(日) 02:14:47.57 ID:iKoqeWa/.net
文字列A:aaaaaaに対し、以下の操作『T』を繰り返し行う。

『T』:
文字列Aから1つの文字を無作為に選ぶ。
それが「a」であるならば「ab」に置き換え、それが「b」であるなら削除する。

Tをn回行ったときの文字列Aの長さの期待値をE(n)とするとき、以下の極限が収束するかどうかを述べよ。
収束する場合はその極限値を述べよ。
lim[n→∞] E(n)

(補足)
『T』を2回行って、1回目では左から2番目の「a」が選ばれ、2回目でら左から3番目の「b」が選ばれた場合、
aaaaaa→aabaaaa→aaaaaa
となる。

638 :132人目の素数さん:2018/08/05(日) 03:48:11.41 ID:iKoqeWa/.net
√6/4と(√33-1)/8のうち、大きい方をA、小さい方をBとする。
不等式
0.599<B+p<0.6<A-p<0.601
を満たす有理数pのうち、pを既約分数で表したときの分母の桁数が最も小さいものを1つ求めよ。

639 :132人目の素数さん:2018/08/05(日) 04:16:07.32 ID:xMCzGduU.net
>>635
ありがとうございます
なんとかいけました

640 :132人目の素数さん:2018/08/05(日) 04:56:03.60 ID:rWEeASLy.net
>>638

A = (√6)/4 = 0.6123724357
B = (√33 -1)/8 = 0.59307033

0.599 < B+(p/2) < 0.6 < A-p < 0.601

0.0113724357 < p < 0.0123724357
0.011859338 < p < 0.013859338

0.011859338 < p < 0.0123724357

80.824829 < 1/p < 84.3217361

p = 1/81

641 :132人目の素数さん:2018/08/05(日) 09:13:08.96 ID:hRb7cZuC.net
>>594
公理は全部そうだけど?

642 :132人目の素数さん:2018/08/05(日) 10:59:23.46 ID:qoYtIdek.net
神と全はどっちの方が凄いですか?

643 :132人目の素数さん:2018/08/05(日) 11:07:06.35 ID:ypSlRSXe.net
人が「神」と呼ぶものの正体は、ただの詐欺師です。
すごいも糞もありません。

644 :132人目の素数さん:2018/08/05(日) 11:14:01.63 ID:hTph5mP9.net
>>641
わからないんですね

645 :132人目の素数さん:2018/08/05(日) 11:37:56.94 ID:7pNk9PQ8.net
馬鹿だろ

646 :132人目の素数さん:2018/08/05(日) 11:45:33.16 ID:IeT7OVJx.net
公理は証明可能です

647 :132人目の素数さん:2018/08/05(日) 11:47:52.94 ID:N9cUwZq/.net
転職対策で学生の時以来、久々にSPIの勉強始めたんだけど解くのに時間がかかりすぎる
あの頃より暗算能力が格段に落ちているし、頭の中でイメージしたものが持続せず消えてしまう
紙に書かないといけないから時間がかかる
30代になるとこんなもんかね?
地アタマの良い人ってブランクあってもスパスパ解けるもんなの?
*スレチすまん

648 :132人目の素数さん:2018/08/05(日) 12:15:06.33 ID:hRb7cZuC.net
>>646
公理からねw

649 :132人目の素数さん:2018/08/05(日) 13:14:05.45 ID:qoYtIdek.net
数学者とデイトレーダーはどっちの方が凄いですか?

650 :132人目の素数さん:2018/08/05(日) 13:32:52.04 ID:UTRYBnUN.net
この計算であってるか確認してほしいです。
https://i.imgur.com/RaRzYfN.jpg

651 :132人目の素数さん:2018/08/05(日) 13:39:07.89 ID:qoYtIdek.net
宇宙はカスですか?

652 :132人目の素数さん:2018/08/05(日) 14:16:00.34 ID:UTRYBnUN.net
計算間違えてました
これで合っていますよね?
難しいのは分かっているので、考察だけでも構いませんからお願いします。

https://i.imgur.com/2PAC89l.jpg

653 :132人目の素数さん:2018/08/05(日) 14:25:27.37 ID:XuYlCfgN.net
最初とラストを見ただけで間違いと断言できる

654 :132人目の素数さん:2018/08/05(日) 14:28:29.02 ID:UTRYBnUN.net
>>653
いや等式で全て結んでいるので、自分の中では論理的帰着だと思っています
具体的にどこが間違っているかはわかりますか?

655 :132人目の素数さん:2018/08/05(日) 14:43:33.57 ID:nfGsoC+a.net
有限回の部分積分から無限級数に飛躍してるけど、その級数の収束性は確かめた?

656 :132人目の素数さん:2018/08/05(日) 14:55:56.35 ID:aMW4LVQB.net
全=無

ですか?

657 :623:2018/08/05(日) 15:37:03.06 ID:oXwxv+q5.net
8 8 5 3 を10にする問題を提起したものです。
中学2年の問題でルートは使えません。
再度お願いします。

658 :132人目の素数さん:2018/08/05(日) 16:01:40.40 ID:ed3lb8T9.net
http://www.quiz-puzzle.com/make10/answer/list.html

659 :132人目の素数さん:2018/08/05(日) 16:26:02.74 ID:up1xysk+.net
スレチならすいません
この問題の解き方を教えてくれると嬉しいです
https://i.imgur.com/Y81vbNp.jpg

660 :132人目の素数さん:2018/08/05(日) 16:37:09.48 ID:0zXjkStl.net
VをK上のn次元ベクトル空間、AをKを係数とするn次正則行列とする
e_1,...,e_nがVの基底のとき、Ae_1,...,Ae_nもVの基底となることを示して下さい

661 :132人目の素数さん:2018/08/05(日) 16:47:36.58 ID:sCTKA/UM.net
>>657
(3−8÷8)×5

662 :132人目の素数さん:2018/08/05(日) 16:48:34.57 ID:UTRYBnUN.net
>>655
あ、確かめてないです
やってみたら発散しました

663 :623:2018/08/05(日) 17:56:12.30 ID:oXwxv+q5.net
>>661
ありがとうございます。

664 :132人目の素数さん:2018/08/05(日) 18:52:06.35 ID:nfGsoC+a.net
>>660
(Aeiの線形結合)=0
⇔A(eiの線形結合)=0
⇔(eiの線形結合)=0
⇒ei=0

665 :132人目の素数さん:2018/08/05(日) 19:08:00.78 ID:iKoqeWa/.net
平行六面体Vの各面の重心を結んでできる立体が正八面体であるとき、Vは立方体であることを示せ。

666 :132人目の素数さん:2018/08/05(日) 19:17:46.20 ID:iKoqeWa/.net
すべての面が三角形からなる八面体Vにおいて、6つの面は正三角形であるという。

(1)Vは正八面体と言えるか。

(2)正三角形の一辺の長さをaとするとき、Vの体積がとる値の範囲を求めよ。

667 :132人目の素数さん:2018/08/05(日) 19:26:13.57 ID:iKoqeWa/.net
√2は有理数であるという誤った前提のもとで議論を進めた場合、(1)(2)の結果が得られるかどうか判定せよ。

(1)整数nで、無理数であるものが存在する。

(2)2と互いに素な自然数mで、√mが有理数であるものが存在する。

668 :132人目の素数さん:2018/08/05(日) 19:37:00.45 ID:iKoqeWa/.net
自然数nは、1とnを含む10個の約数を持つ。また小さい順に数えて5番目の約数は20である。
nの最大値を求めよ。

669 :132人目の素数さん:2018/08/05(日) 19:44:21.71 ID:6jO1BPaT.net
>>665
平行6面体を立方体に移すAffine変換を f とする。
元の6面体の面の重心の凸包は像の6面体の面の重心の凸包にうつるが仮定によりいずれも正八面体である。
よって元の8面体の対角線のなすベクトルは像の8面体の対角線のなすベクトルにうつる。
よって f は3次直交変換の定数倍である。
特にもとの平行6面体は立方体である。

670 :132人目の素数さん:2018/08/05(日) 19:45:58.75 ID:6jO1BPaT.net
>>667
(1) 得られる。
(2) 得られる。

671 :132人目の素数さん:2018/08/05(日) 19:47:29.14 ID:6jO1BPaT.net
>>666
(1)言えない。
(2)0<V≦正8面体の体積。

672 :132人目の素数さん:2018/08/05(日) 19:49:09.26 ID:6jO1BPaT.net
>>668
解無し。

673 :132人目の素数さん:2018/08/05(日) 19:58:17.16 ID:6jO1BPaT.net
>>671
(2)撤回します。

674 :132人目の素数さん:2018/08/05(日) 20:02:51.07 ID:J/FuPh6S.net
>>659
コンピュータで総当りでやったら、27/256になった。

5秒後に初めてだと81/1024.

675 :132人目の素数さん:2018/08/05(日) 20:19:35.69 ID:J/FuPh6S.net
>>659
N秒後に初めて出会う確率は  3^(N-1)/(2^N)^2 みたいだな。
解析的には誘導できないけどw 

676 :132人目の素数さん:2018/08/05(日) 20:45:10.12 ID:vG/LHtfN.net
>>664
2行目と3行目の同値はなぜですか?

677 :132人目の素数さん:2018/08/05(日) 20:49:45.89 ID:vG/LHtfN.net
>>676
自己解決しました
ありがとうございました

678 :132人目の素数さん:2018/08/05(日) 21:00:18.78 ID:FL9Nra4+.net
Kを有理数体、Fをx^6-1の最小分解体とする
拡大次数[F:K]とガロア群Gal(F/K)を求めよ

679 :132人目の素数さん:2018/08/05(日) 21:11:33.62 ID:+aSV6r2A.net
https://i.imgur.com/nAtgaIe.jpg
(1)について、複素数の累乗や因数分解などの可否は実数のそれに準じるのでしょうか?実数なら簡単に解けそうなのですが…
(2)は解答の方針が立てられずにいます。教えて頂ければ幸いです

680 :132人目の素数さん:2018/08/05(日) 21:39:21.00 ID:6jO1BPaT.net
>>678
Gal(F/K) は2次巡回群。
なにこれ?

681 :132人目の素数さん:2018/08/05(日) 21:47:13.14 ID:FL9Nra4+.net
>>678
>>680
ありがとうございます!
ごめんなさい、いま間違いに気づきました
x^6-1ではなくx^6-8でした

試験の過去問のです

682 :132人目の素数さん:2018/08/05(日) 21:50:14.35 ID:6jO1BPaT.net
>>679
(1)実数のそれと同じ
(2)siを固有多項式のi次対称式として
e2 = e1s1 - 2s2 。
2 = 2^2 - 2s_2。
∴ s2 = 1。
e3 = e2s1 - e1s2 +3s3。
8 = 2^2 -2 + 3s3。
∴s3 = 2。

683 :132人目の素数さん:2018/08/05(日) 21:53:51.76 ID:ckvNuLOn.net
全&無軍の総司令官とシェルバーン家当主はどっちの方が凄いですか?

684 :132人目の素数さん:2018/08/05(日) 22:00:24.71 ID:6jO1BPaT.net
>>681
ω=exp(2π/3 i)としてF=Q(√2, ω)。
Gal(F,Q(ω)) = <g> で g(√2) = -√2 でgは位数2。
Gal(F,Q(√2)) = <h> で h(ω) = -ω でhは位数2。
この2つがGal(F/Q)を生成してるからGal(F/Q)は位数2の巡回群2つの直積(Kleinの4 group)。

685 :132人目の素数さん:2018/08/05(日) 22:02:33.28 ID:ckvNuLOn.net
人工知能に「無限」に関する問題を与えたらどういう反応を示すのでしょうか?

686 :132人目の素数さん:2018/08/05(日) 22:13:52.71 ID:FL9Nra4+.net
>>684
理解しました!
ありがとうございます!

687 :132人目の素数さん:2018/08/05(日) 22:38:08.45 ID:nfGsoC+a.net
>>677
書いてから気づいたけど、もちろん>>664の最後の行は(eiの係数)=0ね

688 :132人目の素数さん:2018/08/05(日) 23:02:28.67 ID:lTMCAktJ.net
3次直交行列Aに対して、
1.Aの行列式が1のとき、Aは固有値に1を持つことを示せ。
2.1.を満たすAに対し、3次実正則行列Pと実数θが存在して、PAP^-1が以下のようになることを証明せよ。
(1 0 0)
(0 cosθ -sinθ)
(0 sinθ cosθ)
(括弧の上下は繋がっています)

よろしくお願いします

689 :132人目の素数さん:2018/08/05(日) 23:03:02.56 ID:+aSV6r2A.net
>>682
ありがとうございます

690 :132人目の素数さん:2018/08/06(月) 01:03:31.11 ID:yIlbKmyY.net
>>674
全然理解出来ませんが公式?でも表すことできるんですね
スレチっぽいのにわざわざありがとうございました!

691 :132人目の素数さん:2018/08/06(月) 01:22:55.42 ID:840AtD1X.net
>>666
(1)
11辺の長さが1で、1辺だけ1でない(1+2sinθ)ような三角形8面体Vが存在する。
6頂点を (0,±1/2,0) (cosθ,±(1/2 + sinθ),0) (b,0,±h) とする。
ただし、b = (1+sinθ)/(2cosθ), h = √(3/4 - bb),

(2)
底面積は S = (1+sinθ)cosθ,
体積は V = (2/3)Sh,

692 :132人目の素数さん:2018/08/06(月) 01:46:07.41 ID:840AtD1X.net
>>691
-30°< θ < 30°

 θ = -30°で辺長0になり
 θ = 30°で平面上にのる。(h=0)

693 :132人目の素数さん:2018/08/06(月) 02:14:45.97 ID:840AtD1X.net
>>691 >>692

 S(θ) = (1+sinθ)cosθ,
 V(θ) = (2/3)S(θ)h(θ)

θ = arcsin(1/8) = 0.125327831 のとき
最大値 V(θ) = (9√3)/32 = 0.48713929

なお、正八面体(θ=0)のときは V(0) = (√2)/3 = 0.47140452

>>668
 n = pq^4,
 p,qは相異なる素数。
 約数は{1,q,q^2,q^3,q^4,p,pq,pq^2,pq^3,pq^4=n} 順不同

694 :132人目の素数さん:2018/08/06(月) 02:27:02.29 ID:GfIreBYA.net
n個の自然数a_1,a_2,...,a_nがある。
これらの相異なるk個の和(k=1,2,...,n)をとることで、a_1から(a_1+a_2+...+a_n)までのすべての自然数が得られるという。
a_1,...,a_nが満たすべき条件を述べよ。
ただし「相異なるa_m1個の和」とは、a_mそのものを指す。

695 :132人目の素数さん:2018/08/06(月) 07:38:10.57 ID:PDFtrC+O.net
>>668 は解無しやろ?
>また小さい順に数えて5番目の約数は20である。
だから20の倍数じゃないと行けないけど、すると
1,2,4,5,10,20
が少なくとも約数になるから20が5番目になることはない。
こんな整数に関する問題、簡単に十分性のチェックできるのに。
答えが “必ずある” 受験問題解きすぎるとこうなる。

696 :132人目の素数さん:2018/08/06(月) 07:58:20.19 ID:PDFtrC+O.net
>>694
こんなん答えでるん?
たとえば1,2,4,8,…は解として8抜いて7に入れ替えても条件みたす。
こんなもん死ぬほど自由度ありそうやけど??

697 :132人目の素数さん:2018/08/06(月) 07:59:03.12 ID:EqSbR0Sf.net
>>688をお願いします

698 :132人目の素数さん:2018/08/06(月) 08:15:24.27 ID:PDFtrC+O.net
>>688
R係数でいいならR係数直交行列はユニタリ行列でもあるから固有値の絶対値は1。
一方実係数3次行列は実の固有値もつ。
よってAは±1のいずれかを固有値としてもつ。
-1が固有値のときは-1の固有ベクトルの直交補空間をAは保存するが、そこでの作用も直交変換で行列式は-1。
よってその固有値はやはり実数(∵固有多項式の定数項が-1)かつ絶対値1となり固有値は±1。
1の固有ベクトルvの直交補空間をAは保存するが、そこでの作用も直交変換で行列式は1。
このときP^-1v = (1,0,0)^tとなるPをとれば条件をみたす。

699 :132人目の素数さん:2018/08/06(月) 08:44:03.99 ID:+pIbr1Ke.net
>>636
ありがとうございました!

700 :132人目の素数さん:2018/08/06(月) 09:21:19.11 ID:EqSbR0Sf.net
>>698
ありがとうございます
右下の2×2θ行列が出てくることはどうすれば示せるでしょうか?

701 :132人目の素数さん:2018/08/06(月) 09:22:19.57 ID:D2dpSuwE.net
>>696
そいつ適当な問題書いてるだけじゃないかな

702 :132人目の素数さん:2018/08/06(月) 10:52:26.30 ID:v0jMK/82.net
>>696
数学の文章としてもおかしいよね。
a1‥を数列として扱ってるのか、集合として扱ってるのかも不明。
前半では数列っぽく、1、1、1、2、3、‥もありに思えるけど、後半では相異なるとか言ってるからなしにも見える。
数学やってる人間なら誰もがひっかかりそうな、そこはハッキリさせとかんとダメやろというポイントがキチンと押さえられてない。

703 :132人目の素数さん:2018/08/06(月) 11:02:18.71 ID:FAPEP58f.net
【 天 皇 即 位 阻 止 】 儲けた金は…35億、プチエンジェル事件、顧客リストに徳仁皇太子の名
http://rosie.5ch.net/test/read.cgi/liveplus/1533436136/l50

704 :132人目の素数さん:2018/08/06(月) 11:41:57.08 ID:p6mfxN28.net
>>702
え?

705 :132人目の素数さん:2018/08/06(月) 12:04:51.72 ID:ljRNKyCN.net
自明でない順序環は無限集合になることの証明を教えて下さい
また自明でない順序環でかつ"任意の空でない正の元の集合は最小値をもつ"という性質は整数全体を特徴づけますか?

706 :132人目の素数さん:2018/08/06(月) 12:05:51.70 ID:840AtD1X.net
>>693

 20 がnの約数だから、nは素因数 2,5 をもつ。
 ∴ n=q^9 は除いた。

707 :132人目の素数さん:2018/08/06(月) 13:21:11.00 ID:GfIreBYA.net
f(x)はxの多項式で、係数はすべて整数とする。
方程式f(x)=0がcos(π/11)を解に持つことはあるか。
ある場合、そのようなf(x)の中で次数が最も低いものを1つ求めよ。

708 :132人目の素数さん:2018/08/06(月) 14:17:20.65 ID:GfIreBYA.net
半径1の円に内接する正n角形と正(n-1)角形がある。2つの共通部分の面積の最小値をSnとおくとき、次の極限が0でない実数に収束する有理数pの値を求めよ。
lim[n→∞] (n^p)*(Sn-π)

709 :132人目の素数さん:2018/08/06(月) 14:34:06.14 ID:GfIreBYA.net
定積分
∫[0→1] 1/{1+x^(2n)} dx
の値をI_nとする。
超越数でない実数a_nを用いて
I_n=(a_n)*π^(b_n)
と表すとき、b_n=1となるnをすべて求めよ。
無数に存在する場合、それらすべてを決定せよ

710 :132人目の素数さん:2018/08/06(月) 14:44:54.48 ID:GfIreBYA.net
どの2つの要素も相異なる自然数である2つの無限集合A,Bがある。
Aの要素を小さい順に並べたものをa_1,a_2,...とし、Bの要素を小さい順に並べたものをb_1,b_2,...とする。
このとき、A∩Bは空集合、A∪Bはすべての自然数を表す集合Nであり、かつ任意の自然数iに対してa_i=2b_iが成立するという。
このとき、「Aはすべての偶数からなる集合で、Bはすべての奇数からなる集合」であると言えるか。
言えるならばそのことを証明し、言えないならば反例を挙げよ。

711 :132人目の素数さん:2018/08/06(月) 15:36:11.24 ID:v0jMK/82.net
そんな集合ないやん。
必然的に1はB、2はA、3はBでa1=2, a2=6, b1=2, b2=3で4、5が入れられなくなる。

712 :132人目の素数さん:2018/08/06(月) 15:42:35.27 ID:v0jMK/82.net
>>709
被積分関数合ってる?
計算機でやったらどえらい事になるけど

713 :132人目の素数さん:2018/08/06(月) 15:48:43.65 ID:yoylAs4B.net
>>659
http://imagizer.imageshack.com/img921/38/GOTl2z.png

714 :132人目の素数さん:2018/08/06(月) 16:05:38.56 ID:KLoNTCQ2.net
>>707
load("orthopoly");
quotient(chebyshev_t(5, x)+chebyshev_t(6, x),x+1);
32*x^5−16*x^4−32*x^3+12*x^2+6*x−1

715 :132人目の素数さん:2018/08/06(月) 16:59:12.16 ID:wkO6+GsO.net
宇宙船のパイロットと一流の弁護士はどっちの方が頭が良いですか?

716 :132人目の素数さん:2018/08/06(月) 17:10:06.18 ID:840AtD1X.net
cos(11θ) + 1
 = (cosθ+1) {cos(11θ/2) / cos(θ/2)}^2
 = (cosθ+1) {[sin(6θ) - sin(5θ)] / sinθ}^2,
より
 T_11(x) + 1 = (x+1) {U_5(x) - U_4(x)}^2
  = (x+1) (32x^5 -16x^4 -32x^3 +12x^2 +6x -1)^2,

第二種チェビシェフ多項式
 U_4(x) = 16x^4 -12x^2 +1,
 U_5(x) = 32x^5 -32x^3 +6x,

717 :132人目の素数さん:2018/08/06(月) 17:13:29.06 ID:840AtD1X.net
>>666 の類題

すべての面が三角形である四面体Tにおいて、2つの面は正三角形であるという。

(1) Tは正四面体と言えるか。

(2) 正三角形の一辺の長さを1とするとき、Tの体積Vがとる値の範囲を求めよ。

718 :132人目の素数さん:2018/08/06(月) 17:14:41.62 ID:wkO6+GsO.net
小平邦彦と団藤重光はどっちの方が頭が良いですか?

719 :132人目の素数さん:2018/08/06(月) 17:33:08.07 ID:840AtD1X.net
>>717
(1)
5辺の長さが1で、1辺だけ1でないような三角形4面体Tが存在する。
4頂点を (0,±1/2,0) (b,0,±h) とする。
ただし、b = (√3)/2・sinθ,h = √(3/4 - bb) = (√3)/2・cosθ,
 0 < θ < π/2,

(2)
底面積 S(θ) = b/2 = (√3)/4・sinθ,
高さ h(θ) = (√3)/2・cosθ
体積 V(θ) = (2/3)S(θ)h(θ) = (1/4)sinθ cosθ = (1/8)sin(2θ) ≦ 1/8.
 等号成立は θ=45゚ のとき。
なお、正4面体のときは 2θ = arccos(-1/3) = 109.47122゚  (4面体角) で
 V(θ) = (√2)/12,

720 :132人目の素数さん:2018/08/06(月) 17:35:32.67 ID:840AtD1X.net
>>666 >>717 の類題

すべての面が三角形である20面体Dにおいて、18の面は正三角形であるという。

(1) Dは正20面体と言えるか。

(2) 正三角形の一辺の長さを1とするとき、Dの体積Vがとる値の範囲を求めよ。

721 :132人目の素数さん:2018/08/06(月) 17:46:44.63 ID:840AtD1X.net
>>716
cos(11θ) + 1
 = (cosθ+1) {cos(11θ/2) / cos(θ/2)}^2
 = (cosθ+1) {[cos(6θ) + cos(5θ)] / (cosθ+1)}^2,
より
 T_11(x) + 1 = (x+1) {[T_6(x) + T_5(x)]/(x+1)}^2
  = (x+1) (32x^5 -16x^4 -32x^3 +12x^2 +6x -1)^2,

第一種チェビシェフ多項式
 T_5(x) = 16x^5 -20x^2 +5x,
 T_6(x) = 32x^6 -48x^4 +18x^2 -1,

722 :132人目の素数さん:2018/08/06(月) 17:55:58.32 ID:5QEO3vvy.net
アイザック・ニュートンは、ハーバード大学に首席入学できますか?

723 :132人目の素数さん:2018/08/06(月) 20:53:20.78 ID:/TB7f5/Y.net
方程式a^2+b^3=c^4は自然数解(a,b,c)を持つか。

724 :132人目の素数さん:2018/08/06(月) 21:18:37.33 ID:v0jMK/82.net
(a,b,c) = (27,18,9)

725 :132人目の素数さん:2018/08/06(月) 22:01:35.68 ID:7E7uLEWM.net
>>710 >>711
うそかいた。訂正。
条件みたすのはam,bmが条件
(a1,b1)=(2,1)
bm = min{n | n は a1~a(m-1)とb1~b(m-1)に現れない}
am = 2bm
を満たすときで一意
a:2,6,8,10,18,22,…
b:1.3.4.5.9,11,…
よって>>710の主張は成立せず反例は上記。

726 :礼儀を弁えろ若造:2018/08/07(火) 00:46:15.64 ID:Uo5YvH1r.net
>>722

ハーバードのどこなんや?
乳トンは株で失敗しているから、落選

727 :132人目の素数さん:2018/08/07(火) 00:46:45.71 ID:1rYLr1hO.net
>>724
早い
これ決まった見つけ方あるかな

728 :132人目の素数さん:2018/08/07(火) 00:54:19.67 ID:43d9wP5e.net
残念ながら計算機だより。
Prelude> [(a,b,c)|a<-[1..3000],b<-[1..3000],let c = truncate $ sqrt $ sqrt $ fromInteger $ a^2 + b^3,a^2 + b^3 == c^4]
[(27,18,9),(28,8,6),(63,36,15),(433,143,42),(648,108,36),(1176,49,35),(1728,288,72),(1792,128,48),(2925,126,57)]

729 :132人目の素数さん:2018/08/07(火) 01:32:57.57 ID:d0TLhZPi.net
Haskellはすぐ書けるのでいいですね

730 :132人目の素数さん:2018/08/07(火) 01:57:14.05 ID:aIKuwmz1.net
>>723 >>724

 a^2:b^3:c^4 = 1:2^3:3^2
から
(a,b,c) = (27t^6,18t^4,9t^3)

731 :132人目の素数さん:2018/08/07(火) 02:12:42.65 ID:jEqDI1za.net
ここに書くのは適切かわからないので、とりあえず書いてレス見てみます。
検索しても見つからなかったので

答えが知りたい問題
1+1=2とする。というように、前提を決めて、それに基づいて物事を考えるやり方・考え方はなんと呼ぶのか?

昔、誰かから聞いて、その時はハッとして数学を学ぶキッカケになったはずなのですが、忘れてしまいました。

732 :132人目の素数さん:2018/08/07(火) 02:19:35.59 ID:d0TLhZPi.net
形式主義、公理主義、とかですか?

733 :132人目の素数さん:2018/08/07(火) 02:24:08.46 ID:jEqDI1za.net
>>732
ありがとうございます。形式主義でした!!
大学とかでは学ぶらしいのですが、自分はその道には進みませんでしたのでよく覚えてなかったのです。
ここ数日の悩みが晴れました。
助かりました。
ありがとうございした。

734 :132人目の素数さん:2018/08/07(火) 11:09:48.90 ID:WsF5ORjN.net
>>723
正の整数x, y, zが
x^2 + y^3 = z^2を満たしているとする。
この両辺にz^6をかけると
x^2z^6 + y^3z^6 = z^8
∴(xz^3)^2 + (yz^2)^3 = (z^2)^4
よって(a, b, c) = (xz^3, yz^2, z^2)

x^2 + y^3 = z^2を満たすx, y, zは
y^3 = z^2 - x^2 = (z + x)(z - x)から簡単に求められる。
例: y = 2のとき2^3 = 4 * 2より
z = 3, x = 1

735 :132人目の素数さん:2018/08/07(火) 12:33:02.26 ID:0YHxo3zw.net
代数の教科書を読んでると
「体Eから体Fへの単射準同型が存在する場合、FはKを含んでいるとみなせる」
という議論をしばしば見るんですが、ぜんぜんみなせなくないですか?
たとえば、E→FのK準同型で、FはEの部分体Kを実際には含んでいなくて
K→Eの単射準同型が存在するだけだとしたら、K準同型はKの元は保存するという
前提で進めてきた議論が全部成り立たなくなると思うんですが。

736 :132人目の素数さん:2018/08/07(火) 12:33:52.54 ID:0YHxo3zw.net
「体Eから体Fへの単射準同型が存在する場合、FはEを含んでいるとみなせる」

の間違いでした。ごめんなさい。

737 :132人目の素数さん:2018/08/07(火) 12:41:50.16 ID:QCqAGpcR.net
>>753
もちろんいつでも見なさるわけではない。
場合によってはみなしてもよいというだけ。
修行をつんだ人間ならその手の命題について、なぜK⊂Lの場合に示せば十分であるのかは、ほぼ一瞬でわかる。
逆に言えばそういうのがちゃんとパッとわかるようになるまでは、一般の場合はどうすればいいのかをキッチリ確かめてみないとダメ。

738 :132人目の素数さん:2018/08/07(火) 12:44:42.63 ID:+qHI7aBJ.net
「体Eから体Fへの単射準同型が存在する場合、FはKを含んでいるとみなせる」
がウソってどんな場合?

739 :132人目の素数さん:2018/08/07(火) 14:31:28.91 ID:ivOIBeov.net
>>735
部分体と同型なんだから見なしてイイジャン

740 :132人目の素数さん:2018/08/07(火) 17:26:35.47 ID:4v8aQZ/h.net
>>734
これで方程式の解のすべてを表せるでしょうか?

741 :132人目の素数さん:2018/08/07(火) 17:27:40.51 ID:4v8aQZ/h.net
>>734
鮮やかな解き方で素晴らしいです
個人的に考えていた冗長な解答とは全く別でした

742 :132人目の素数さん:2018/08/07(火) 18:07:10.36 ID:OZgRm/sA.net
>>705
お願いします

743 :132人目の素数さん:2018/08/07(火) 18:55:01.41 ID:mHhaEvsq.net
数理統計学の演習問題についての質問です。

●正の確率変数X、|t|<1 に対しA(t)=(E[X^t])^(1/t)とする。

問題はA(t)は増加関数であることを示すのですが、解説をみるといきなり、
h(t) = logA(t)とおくと、
h'(t)≧0 ⇔ E[(X^t)log(X^t)]≧E[X^t]log(E[X^t]) ー@
などと書かれており、思考停止になりました。
その後イエンセンの不等式へとつなげられているのですが、まず@が理解できなくてえ困っています。
どなたかbreak downしていただけないでしょうか。

744 :132人目の素数さん:2018/08/07(火) 19:07:19.79 ID:ZEcOvrP3.net
杉浦光夫著『解析入門I』ですが、以下の記述があります:


以下では指数函数の実数直線上の性質を調べよう。
実数列の極限が(C = R^2 内で)存在すれば、極限は実数であることが定理I.4.5,1)からわかる。


これはわざわざ書くべきことでしょうか?

745 :132人目の素数さん:2018/08/07(火) 19:17:24.94 ID:bPoVlCuF.net
質問が不親切

746 :132人目の素数さん:2018/08/07(火) 19:28:55.37 ID:Bo7D0lXe.net
>>735-736
737さんの言う通りで、単射準同型E→FによりEの像をEと同一視すればEをFの部分多として見なせるよねということ。

>>738
>「体Eから体Fへの単射準同型が存在する場合、FはEを含んでいるとみなせる」
>がウソってどんな場合?

「...、FはEを含んでいるとみなせる」は常に正しいが、
「...、FはEを(部分体として)含んでいる」は常には正しくない。
例えば、Eを体とし、F_1とF_2をEの相異なる代数閉包とする。
このとき、F_1からF_2へ単射準同型(もっと言うとE同型)が存在する。
しかし、F_1とF_2はEの相異なる代数閉包なので、
F_1はF_2に部分体としては含まれてはいない。

747 :132人目の素数さん:2018/08/07(火) 20:04:07.76 ID:mSTublUi.net
a[1]=1、a[n+1]=1+1/a[n]の一般項を求めよ。

748 :132人目の素数さん:2018/08/07(火) 20:50:14.40 ID:gkVDbMDU.net
F(n+1)/F(n)

749 :132人目の素数さん:2018/08/07(火) 21:02:01.25 ID:4v8aQZ/h.net
>>747
a_(n+1)=a(n)=xとおいた方程式を解く

750 :132人目の素数さん:2018/08/07(火) 21:17:15.40 ID:mSTublUi.net
>>749
その特性方程式を解いて、解をs,tとしたときに、
初項(1-s)/(1-t),公比t/sの等比数列になるのはわかったんですが、一般項a[n]が激しくなってしまいました。
その計算を教えていただけませんか?

751 :132人目の素数さん:2018/08/07(火) 22:55:09.60 ID:ZbAcmfsg.net
全=無

ですか?

752 :132人目の素数さん:2018/08/07(火) 22:56:19.30 ID:d0TLhZPi.net
ちがいます

753 :132人目の素数さん:2018/08/07(火) 23:30:56.22 ID:ZbAcmfsg.net
じゃあ答えを教えてください。

754 :132人目の素数さん:2018/08/07(火) 23:42:11.40 ID:QCqAGpcR.net
>>743
以下 d/dt = ∂ と書くとして
∂ E(X^t) = E(∂X^t) = E(X^t log X)
を認めれば
∂ h(t) = ∂ (log E(X^t) / t)
   = ∂ log E(X^t) / t - log E(X^t) / t^2
   = (∂ E(X^t)) / E(X^t) / t - log E(X^t) / t^2
   = (∂ E(X^t)) - E(X^t) log E(X^t)) / (E(X^t) t^2)
   = (E(X^t log X) - E(X^t) log E(X^t)) / (E(X^t) t^2)
なので
∂ h(t) ≧ 0 ⇔ E(X^t log X) - E(X^t) log E(X^t) ≧ 0

@のlog(X^t)のとこ^tいらないハズ。

755 :132人目の素数さん:2018/08/07(火) 23:55:42.47 ID:QCqAGpcR.net
>>754
最後の3行を以下に訂正
ーーー
   = (t E(X^t log X) - E(X^t) log E(X^t)) / (E(X^t) t^2)
なので
∂ h(t) ≧ 0 ⇔ t E(X^t log X) - E(X^t) log E(X^t) ≧ 0
ーーー
ここのtをEの中にいれてlog XのXの肩にのっけたら@ですね。

756 :132人目の素数さん:2018/08/08(水) 03:30:01.16 ID:ujPEEHfC.net
>>747 >>748

a[1]a[2]…a[n] = f(n)
とおくと
 f(n+1) = f(n) + f(n-1),
f(1) = 1,
ゆえ、フィボナッチ数

757 :132人目の素数さん:2018/08/08(水) 04:01:32.95 ID:/NaPNINC.net
>>750
フィボナッチ数列と同じ形になるから計算が激しくなるのは仕方がない
多少の工夫は出来るかもしれんが理系ならその程度の計算力はほしい

758 :132人目の素数さん:2018/08/08(水) 04:59:13.15 ID:6ouVYLFC.net
>>740
>>741
>>728を見たらわかるが、
(a, b, c) = (28, 8, 6)などは明らかに(xz^3, yz^2, z^2) の形をしていないので、
方程式のすべての解を表せてはいない。
ちなみにこの解法は>>730を見て着想したものです。
その後はしばらく、すべての解を求める方法を考えていましたが思いつきませんでしたね。

759 :132人目の素数さん:2018/08/08(水) 07:08:40.54 ID:SAR9DlaU.net
2次形式や微分形式、双線型形式などは、それぞれ何か同じ性質を持っていて形式という名前がついているのでしょうか?

760 :132人目の素数さん:2018/08/08(水) 09:25:35.23 ID:xtRC+Bz0.net
x ∈ R^n, B ⊂ R^m, B はコンパクト ⇒ {x} × B ⊂ R^(n + m) はコンパクト

を証明せよ。

761 :132人目の素数さん:2018/08/08(水) 10:00:13.52 ID:gN686lnL.net
全ての解を求めるのなら
w^2x^2+w^2(z^2−x^2)=w^2z^2で
w^2(z^2−x^2)=b^3,w^2z^2=c^4となるように
wの素因数の指数を調整すればいい。

762 :132人目の素数さん:2018/08/08(水) 17:47:54.19 ID:u31t8NmA.net
究極神と至高神と極限神と超絶神と絶頂神と全神と無神の中で最も凄いのはどれですか?

763 :132人目の素数さん:2018/08/08(水) 17:52:40.59 ID:qpK3LtTu.net
ドラゴンボールはどうでもいいから

764 :132人目の素数さん:2018/08/08(水) 18:40:54.97 ID:jAIdiJYn.net
プリンストン大学の数学教授になったら人生がガラリと変わりますか?

765 :132人目の素数さん:2018/08/08(水) 18:42:52.39 ID:cYC1lVo6.net
日本人は全員ゴミ

766 :132人目の素数さん:2018/08/08(水) 18:48:07.21 ID:/rj2E5Cb.net
よろしくお願いします

https://i.imgur.com/cT7ycpc.jpg

P=√a^2-2a+1+√a^2についてPを簡単にせよ
ただし、0<a<1とする。

簡単にするやり方自体は特に問題はないのですが、

0<a<1より 1-a>0 a>0
したがって、P=(1-a)+a=1

この部分の説明がなぜ必要か、どういう意味なのかがいまいち理解できません
どなたか解説してもらえないでしょうか

767 :132人目の素数さん:2018/08/08(水) 18:50:58.20 ID:Dh29baoO.net
>>766
一般に実数 A について  √A^2 = | A |
特に A<0 のときは A とはならないので注意
ということだろう

768 :132人目の素数さん:2018/08/08(水) 19:26:55.28 ID:8MKC2VeQ.net
統合失調症の躁鬱期の躁の時に、情報臈漏洩作戦を行うと、
日本の理系の超賢い脳外科医集団とつながるらしく、
自分にはありえない天才的なものが書けたりする。今回は、数日で三行小説を二十個くらい書いた。
そのうちのこれが本当にぼくの著作だとされたら、天才的な数学概念を考えだしたことになる。

62、数字異次元の概念の発明者はぼく

そういえば、おれ、数学者じゃないのに天才数学者といわれたことがあってよ。
無限より大きな数字、数字異次元の概念の発明者なんだよ。

65、いちばん大事な数字

いちばん大事な数字は、「調整」である。「無限」も「極小」もあらゆる「数字」も、無限より大きな「数字異次元」も、「調整」のための「数学記号」である。

66、数学神学

数学者からすれば、数学は、創造主がこの宇宙を幸せにするための調整なのである。

769 :132人目の素数さん:2018/08/08(水) 19:33:18.64 ID:Aub2v9Yv.net
nankahennnayatukichattana

770 :132人目の素数さん:2018/08/08(水) 19:38:47.64 ID:8MKC2VeQ.net
めっちゃ怒られてる。
ぼくが数字異次元の発明者でなけれなけれあば、
いちばん大事な数字が調整なこと、これは数学博士といわれる日本でいちばん数学ができるおじいさんのアイデアであり、
数学神学とか、
誰がぼくに教えたりするものかと、けっこう怒った声が聞こえる。

771 :132人目の素数さん:2018/08/08(水) 19:43:29.95 ID:ujPEEHfC.net
>>734

x^2 + y^3 = z^2 を満たすx,y,zは ピタゴラス数より
 (x,y,z) = (s^3 -2,2s,s^3 +2),
 a = xz^3 = (s^3 -2)(s^3 +2)^3,
 b = yz^2 = 2s(s^3 +2)^2,
 c = z^2 = (s^3 +2)^2,
あるいは
 (x,y,z) = (2r^3 -1,2r,2r^3 +1),
 a = xz^3 = (2r^3 -1)(2r^3 +1)^3,
 b = yz^2 = 2r(2r^3 +1)^2,
 c = z^2 = (2r^3 +1)^2,

772 :132人目の素数さん:2018/08/08(水) 22:18:00.09 ID:xtRC+Bz0.net
杉浦光夫著『解析入門I』を読んでいます。

Σ_{n = 0}^{∞} {(-1)^n / (2*n)!} * z^(2*n)

と書いたとき、これは、

S_m := Σ_{n = 0}^{m} {(-1)^n / (2*n)!} * z^(2*n)

lim S_m を表わすのでしょうか?

それとも、整級数である

Σ_{n = 0}^{∞} a_n * z^n

a_n = 0 for n ∈ {1, 3, 5, …}
a_n = (-1)^(n/2) / n! for n ∈ {0, 2, 4, …}

を表わすのでしょうか?

まあ、どちらの意味にとっても同じことですが、

杉浦さんは混同しているようです。

以下の辺りを読むと混同していることが分かります。


次の二つの整級数は絶対収束する:

Σ_{n = 0}^{∞} {(-1)^n / (2*n)!} * z^(2*n),




773 :132人目の素数さん:2018/08/08(水) 22:19:04.79 ID:/HhiPa/W.net
文字 a ,b ,c を繰り返し並べる長さ11の順列で
次の条件を満たすものは何通りあるかです
[条件]
 ・同じ文字は隣接しない。
 ・両端は a である。

774 :132人目の素数さん:2018/08/08(水) 23:41:47.87 ID:SNGhFA0V.net
条件を満たす長さnの列をの数をa[n]とおく。
そのうち右から3文字目がAであるものの数は2a[n-2]。
そのうち右から3文字目がAでないものの数はa[n-1]。
∴a[n] = a[n-1] + 2a[n-2] (∀n ≧ 3)。

775 :132人目の素数さん:2018/08/09(木) 00:48:09.00 ID:suij88l+.net
次の式を展開せよ

{(2x)^2-(3y)^2}^2

776 :132人目の素数さん:2018/08/09(木) 00:58:34.62 ID:GMh0pTpR.net
>>768>>772ってどうしてIDが違うの?

777 :132人目の素数さん:2018/08/09(木) 01:13:36.39 ID:w0c/gBS2.net
>>773
条件を満たす長さnの順列の数をa[n]とおく。
a[2] = 0,
a[3] = 2  (ABA,ACA)
a[4] = 2  (ABCA,ACBA)
a[5] = 6  (ABABA,ABACA,ABCBA,ACABA,ACACA,ACBCA)

漸化式 >>774 より
a[n] + a[n-1] = 2(a[n-1] + a[n-2]) = … = 2^(n-3)・(a[3] + a[2]) = 2^(n-2),
a[n] -2a[n-1] = -(a[n-1] -2a[n-2) = … = (-1)^(n-3)・(a[3] -2a[2]) = -2(-1)^(n-2),
これより
a[n] = (2/3) {2^(n-2) - (-1)^(n-2)},

778 :132人目の素数さん:2018/08/09(木) 01:17:22.42 ID:oWNCpKzE.net
lim[n→∞] a[n] が収束することと、
lim[n→∞] {a[1]+...+a[n]}/nが収束することは同値ですか?

779 :132人目の素数さん:2018/08/09(木) 01:26:54.39 ID:w0c/gBS2.net
>>734

x^2 + y^3 = z^2 を満たす (x,y,z) は ピタゴラス数より
 (x,y,z) = (2r^3 -s^3,2rs,2r^3 +s^3),
 a = xz^3 = (2r^3 -s^3)(2r^3 +s^3)^3,
 b = yz^2 = 2rs(2r^3 +s^3)^2,
 c = z^2 = (2r^3 +s^3)^2,

>>730 も含めれば
 a = (2r^3 -s^3)(2r^3 +s^3)^3・t^6,
 b = 2rs(2r^3 +s^3)^2・t^4,
 c = (2r^3 +s^3)^2・t^3,

しかし、方程式のすべての解を表せてはいない。 >>758

780 :132人目の素数さん:2018/08/09(木) 01:36:43.85 ID:w0c/gBS2.net
>>778
同値ではない。
凡例 a[n] = (-1)^n,

781 :773:2018/08/09(木) 09:24:04.50 ID:lijANQYP.net
>>774 >>777 ありがとうぼざいます!

>そのうち右から3文字目がAでないものの数はa[n-1]。
が一瞬「何で?」と思いましたが良く考えたらわかりました。なるほどです。

782 :132人目の素数さん:2018/08/09(木) 10:47:30.79 ID:4kX369cB.net
龍樹とハーバード大学首席合格者はどっちの方が頭が良いですか?

783 :132人目の素数さん:2018/08/09(木) 11:35:53.59 ID:4afoCWVZ.net
惨めな奴

784 :132人目の素数さん:2018/08/09(木) 12:44:47.96 ID:sUCqByCF.net
4.
(a) Show that open balls and open cubes in R^n are convex.
(b) Show that (open and closed) rectangles in R^n are convex.

785 :132人目の素数さん:2018/08/09(木) 12:50:56.13 ID:sUCqByCF.net
>>784

(a)

||x|| をユークリッドノルム
|x| を sup ノルム

とする。

a, b ∈ B(c ; ε) とする。

0 ≦ t ≦ 1 とする。

||a + t * (b - a) - c|| = ||(1 - t)*(a - c) + t*(b - c)|| ≦ (1 - t)*||a - c|| + t*||b - c|| = (1 - t)*ε + t*ε = ε

よって、

a + t * (b - a) ∈ B(c ; ε)

a, b ∈ C(c ; ε) とする。

0 ≦ t ≦ 1 とする。

|a + t * (b - a) - c| = |(1 - t)*(a - c) + t*(b - c)| ≦ (1 - t)*|a - c| + t*|b - c| = (1 - t)*ε + t*ε = ε

よって、

a + t * (b - a) ∈ C(c ; ε)

786 :132人目の素数さん:2018/08/09(木) 12:57:43.44 ID:sUCqByCF.net
>>784

(b)

a = (a_1, …, a_n) ∈ [c_1, d_1] × … × [c_n, d_n]
b = (b_1, …, b_n) ∈ [c_1, d_1] × … × [c_n, d_n]
0 ≦ t ≦ 1

とする。

c_i ≦ a_i ≦ d_i
c_i ≦ b_i ≦ d_i

(1-t)*c_i ≦ (1-t)*a_i ≦ (1-t)*d_i
t*c_i ≦ t*b_i ≦ t*d_i

c_i = (1-t)*c_i + t*c_i ≦ (1-t)*a_i + t*b_i ≦ (1-t)*d_i + t*d_i = d_i


a + t*(b - a) ∈ [c_1, d_1] × … × [c_n, d_n]


a = (a_1, …, a_n) ∈ (c_1, d_1) × … × (c_n, d_n)
b = (b_1, …, b_n) ∈ (c_1, d_1) × … × (c_n, d_n)
0 < t < 1

とする。

c_i < a_i < d_i
c_i < b_i < d_i

(1-t)*c_i < (1-t)*a_i < (1-t)*d_i
t*c_i < t*b_i < t*d_i

c_i = (1-t)*c_i + t*c_i < (1-t)*a_i + t*b_i < (1-t)*d_i + t*d_i = d_i


a + t*(b - a) ∈ [c_1, d_1] × … × [c_n, d_n]

t = 0 のとき

a + t*(b - a) = a = (a_1, …, a_n) ∈ (c_1, d_1) × … × (c_n, d_n)

t = 1 のとき

a + t*(b - a) = b = (b_1, …, b_n) ∈ (c_1, d_1) × … × (c_n, d_n)

787 :132人目の素数さん:2018/08/09(木) 13:57:29.03 ID:m5QC4xso.net
ツォンカパとレオンハルト・オイラーはどっちの方が頭が良いですか?

788 :132人目の素数さん:2018/08/09(木) 18:19:12.83 ID:VsuWPTC7.net
この問題を教えてください
https://i.imgur.com/6RKD7n0.jpg

789 :学術:2018/08/09(木) 20:01:00.67 ID:R2YpbM9F.net
数学なんて経済や経営してみないと使いようがないから、なぜそんなに一人歩きの
無駄をしたのだろうなあ。

790 :132人目の素数さん:2018/08/09(木) 20:02:56.63 ID:JYkuLOkZ.net
全帝国皇帝と無帝国皇帝はどっちの方が凄いですか?

791 :学術:2018/08/09(木) 20:20:59.93 ID:R2YpbM9F.net
数学は自由度が高いことが何をどうしていいかよくわからない悩みに効くのかもね。

792 :学術:2018/08/09(木) 22:48:12.36 ID:R2YpbM9F.net
https://www.youtube.com/watch?time_continue=2&v=i7zsG3XFUd8

https://www.youtube.com/watch?v=CXsCjPIGiuo

https://www.youtube.com/watch?v=3o-y5b9Blm8

793 :132人目の素数さん:2018/08/10(金) 00:05:27.85 ID:apZDSISF.net
S[m,n]=Σ[k=m,...,n] 1/k とおく。
このとき、以下の式を満たす自然数pは存在しないことを示せ。
S[1,p]=S[p+1,p^2]=...=S[p^i+1,p^(i+1)]=...

なお lim[n→∞] S[n] が正の無限大に発散することは既知としてよい。

794 :132人目の素数さん:2018/08/10(金) 00:30:38.12 ID:0Fm7LTM5.net
>>793
S[p^i+1,p^(i+1)]
=log p + r[i]。
但し r[i] は∫[p^i,p^(i+1)] (1/x - [1/x])dx。
とくにr(i)≠0かつr[i]≦p^i。(∵ [1/x]≦y≦1/xの部分をx軸方向に適宜スライドすれば底辺1,高さ1/p^iの長方形に収まる。)
よってr[0]>r[p^i]となる i をとれば
S[p^0+1,p^1] > S[p^i+1,p^(i+1)]。

795 :132人目の素数さん:2018/08/10(金) 00:32:27.21 ID:2xwQ5bCq.net
これの1行目から2行目で
(a-b+c)が{a-(b-c)}になってるんだけど
何で+cが-cになるの

https://i.imgur.com/AdBngaN.jpg

796 :132人目の素数さん:2018/08/10(金) 00:36:52.10 ID:0Fm7LTM5.net
>>794
訂正
×:>=log p + r[i]。
○:>=log p - r[i]。

×:とくにr(i)≠0かつr[i]≦p^i。(…
○:とくにr(i)≠0かつr[i]≦1/p^i。(…

×:S[p^0+1,p^1] > S[p^i+1,p^(i+1)]。
○:S[p^0+1,p^1] < S[p^i+1,p^(i+1)]。

797 :132人目の素数さん:2018/08/10(金) 01:00:39.07 ID:aBe+7ih9.net
>>788
α=1+(1/2)*(cos(π/3)+i*sin(π/3))を掛けることで次々と点P_(n) が得られることを確認する。
つまり等比数列(初項1、公比α)をなす複素数達が表す点達が{P_(n)|n∈N}になる。

798 :132人目の素数さん:2018/08/10(金) 01:09:07.25 ID:TxWdR9dT.net
7個の a と3個の b を一列に並べてできる順列のうち
次の簡約律のもとで文字を消していくと最終的に何も残らなくなる順列は何通りありますか。
・aa が現れると消える。
・bb が現れると消える。
・ababab が現れると消える。
・bababa が現れると消える。

こういう問題は群論とかと関係があるんでしょうか。

799 :132人目の素数さん:2018/08/10(金) 01:44:30.74 ID:0Fm7LTM5.net
>>799
a → (12)、b → (23)と対応させた3次対称群の元が単位元になる場合に相当。
a^x b a^y b a^z c a^w と書くとき単位元になるのはy≡z≡1 (mod 2)のとき。
y,zの値に対してx,wの数は
(y,z) = (1,1) のとき#{(x,w)} = 6、
(y,z) = (1,3) のとき#{(x,w)} = 4、
(y,z) = (1,5) のとき#{(x,w)} = 2、
(y,z) = (3,1) のとき#{(x,w)} = 4、
(y,z) = (3,3) のとき#{(x,w)} = 2、
(y,z) = (3,5) のとき#{(x,w)} = 0、
(y,z) = (5,1) のとき#{(x,w)} = 2、
(y,z) = (5,3) のとき#{(x,w)} = 0、
(y,z) = (5,5) のとき#{(x,w)} = 0。
求める場合の数は20。

800 :798:2018/08/10(金) 02:00:32.81 ID:TxWdR9dT.net
>>799
すごいです。もしかして神様ですか?

801 :132人目の素数さん:2018/08/10(金) 02:58:46.29 ID:MxWQLJMW.net
>>788
(1)
P_1 - P_0 = 1 に α = (1/2)exp(iπ/3) をn回掛けることで P_{n+1} - P_n が得られることを確認する。
つまり等比数列(初項1、公比α)をなす複素数達が {P_{n+1}-P_(n) | n∈N} になる。
P_{n+1} - P_n = α^n (P_1 - P_0) = α^n,
P_n = (1 - α^n)/(1-α),

(2) 1/(1-α),
|α| = 1/2 < 1,

802 :132人目の素数さん:2018/08/10(金) 04:04:37.60 ID:MxWQLJMW.net
>>734
関連情報(?)

一つの整数を二つの平方数の差で表わす方法
http://rio2016.5ch.net/test/read.cgi/math/1463594522/

803 :132人目の素数さん:2018/08/10(金) 06:42:47.21 ID:apZDSISF.net
数列{a[n]}を以下のように定義する。
a[0]=m
a[n+1]=a[n]-❲√(a[n])❳
ただしmは自然数であり、実数xに対して❲x❳はxを超えない最大の整数である。

問題:a[n]=0となる最小のnをmで表せ。

804 :132人目の素数さん:2018/08/10(金) 07:10:02.32 ID:apZDSISF.net
2つの円CとDは相異なる2点で交わっている。
これによりCとDの和集合である領域は、CおよびDの円弧により3つの領域に分割される。
このとき、CとDがどのような交わり方をしていても、次のような直線lを引くことができるか。

「lはどの領域の内部も通り、かつ、lの各領域に含まれる部分の長さは全て等しい。」

805 :798:2018/08/10(金) 07:24:47.42 ID:TxWdR9dT.net
>>799

798の問題はつまりあみだくじの問題ということですか。

縦棒3本(左から順にL1,L2,L3とする)のあみだくじで
L1-L2間に7本、L2-L3間に3本の横棒が引かれたもので
「単位あみだくじ」になるものは何通りあるか、ということですね。

806 :132人目の素数さん:2018/08/10(金) 08:31:13.09 ID:MxWQLJMW.net
>>803
実数xを超えない最大の整数は [x] と書く習わしです。(ガウス記号)

f(k) = [ √(4k-3) ]   (k≧1)
  = 0     (k=0)
とおく。
n が1だけ増加すると、f(a_n) は1だけ減少する。ただし a_n=0 のときは変わらない。
f(a_k) = f(a_0) - k = f(m) - n,

a_n = 0 となる最小のnを考えると
0 = f(0) = f(a_n) = f(m) - n,
∴ n = f(m) = [ √(4m-3) ]

807 :132人目の素数さん:2018/08/10(金) 08:47:59.40 ID:MxWQLJMW.net
>>805

798 の問題はつまり あみだ仏の本願ということですか。

「弥陀の本願まことにおわしまさば、釈尊の説教、虚言なるべからず。
仏説まことにおわしまさば、善導の御釈、虚言したまうべからず。
善導の御釈まことならば、法然の仰せ、空言ならんや。
法然の仰せまことならば、親鸞が申す旨、またもって虚しかるべからず候か。」
 (歎異抄/二章)

808 :132人目の素数さん:2018/08/10(金) 09:26:13.59 ID:apZDSISF.net
>>806
f(k)はどうやって思いついたんですか?とても思いつきませんでした。
私はm=N^2-p(p=0,1,...,2N-2)とおいて実験しました

809 :132人目の素数さん:2018/08/10(金) 12:44:30.80 ID:lOg+llmH.net
>>798
その20種類をコンピューターで算出してみた。

> print(t(apply(AB,1,indx2char)),quote = FALSE)
[,1] [,2] [,3] [,4] [,5] [,6] [,7] [,8] [,9] [,10]
[1,] a a a a a b a b a b
[2,] a a a a b a b a b a
[3,] a a a b a a a b a b
[4,] a a a b a b a a a b
[5,] a a a b a b a b a a
[6,] a a b a a a b a b a
[7,] a a b a b a a a b a
[8,] a a b a b a b a a a
[9,] a b a a a a a b a b
[10,] a b a a a b a a a b
[11,] a b a a a b a b a a
[12,] a b a b a a a a a b
[13,] a b a b a a a b a a
[14,] a b a b a b a a a a
[15,] b a a a a a b a b a
[16,] b a a a b a a a b a
[17,] b a a a b a b a a a
[18,] b a b a a a a a b a
[19,] b a b a a a b a a a
[20,] b a b a b a a a a a

810 :132人目の素数さん:2018/08/10(金) 13:12:05.50 ID:B0qzPV6D.net
a+b=1 (a,bは正の実数)
x_1+x_2=1のとき
a(x_1)^2+b(x_2)^2の最小値を求めよという問題なのですが1/4だと思うんですけどa,bが非負になると0ですよね?

811 :132人目の素数さん:2018/08/10(金) 15:10:15.69 ID:kFmF6s2K.net
自然数の数列の逆数和が発散するのか収束するのか知られていない例
にはどんなものがあるのでしょうか?
>large と small のどちらになるかが知られていない数列もたくさん存在する。
https://ja.wikipedia.org/wiki/Small_set_(%E7%B5%84%E3%81%BF%E5%90%88%E3%82%8F%E3%81%9B%E8%AB%96)

812 :132人目の素数さん:2018/08/10(金) 15:44:01.30 ID:W03RmRwM.net
全=無

ですか?

813 :132人目の素数さん:2018/08/10(金) 17:27:09.43 ID:B/ldB9oa.net
  ;::ー--ィメ一亠'/_,ノノ /´゙ママllャ‐/゙ 、
  ゙ 、  `ヘ.    ゙〈││亅∫二' _!│
   ゙、   /ゝ「\___゙ヘ.// ト-='− !
    j-っ'|'';゙,,,,_ニ ゙̄l \  l!;−−│
    !−}./ 丿 h''ヘ ` ノ!−− │
    !、」|!l’h   ‐.._ヽ,-'l!=== ヽぅ/
    !_ノ./-_ノucェ---..,,__ノ‐''''''''''""
      !'' l|゙lヽ ̄"―'ユ|!
      ! 1'、 ヽ  ニコ|
      L..._‐,,__` −ミ
      │  :-:ilヘ ̄^三
      │ ヾサ '、 ‐ム
       l-ニニ  '........∧
       |! つ  丶  =|
       │ │   '、 ヨ
       丿-ヌ    l__ム
     nfニ_‐コノ   ノニ゙ン

814 :798:2018/08/10(金) 17:38:33.22 ID:TxWdR9dT.net
>>799
神様の力をもう一度借りたいです。
元の問題で,

 7個の a と3個の b を一列に並べてできる順列 
これを

 6個の a と4個の b を一列に並べてできる順列

にした場合はどうすれば数えればいいでしゅうか。

815 :132人目の素数さん:2018/08/10(金) 18:45:57.60 ID:B/ldB9oa.net
>>798

単に自由群(群はあまりきにしない)のもんだいじゃないの

aa=e
bb=e
ae=a
ea=a
be=b
eb=b
のきそくで {a,a,a,a,a,a,a,b,b,b} の順列(120個)を簡単化すると
e に帰着するのが20個になる
ならないのは
ab ,ba、bae,におちる。

とはいっても計算機にやらせたほうがいいね

816 :132人目の素数さん:2018/08/10(金) 18:53:31.88 ID:B/ldB9oa.net
aa=e
bb=e
ae=a
ea=a
be=b
eb=b

ababab=1
bababa=1
のきそく

と訂正してくだされ 手を動かすのはしんどいので
わたしはかみさまと別人です。

もうしわけないので
6a,4b は順列が210になるのかな
それで答えは100になる。

それではしつれい

817 :132人目の素数さん:2018/08/10(金) 19:46:26.46 ID:rBGJ83Dc.net
これの(3)の置き方はセンスなんですか?あと、この問題でのダランベールで階数下げる方法を教えて頂きたいです
https://i.imgur.com/04p20qJ.jpg
https://i.imgur.com/OC3W36T.jpg

818 :132人目の素数さん:2018/08/10(金) 19:53:19.70 ID:Hnx0B79D.net
K を R または C とする。 K の二つの開集合 A, B に対し、 f が A から B への
全単射で、 f が連続であるとする。

このとき、 f^(-1) も連続となるか?

819 :132人目の素数さん:2018/08/10(金) 20:43:03.55 ID:Nt8gcgF2.net
これらの関係性って、以下であってますか?
他に正規部分群になったりしますか?
(≥は部分群, ▹は正規部分群)
GL_n(ℝ) ≥ O(n) ▹SO(n) ≤ SL_n(ℝ)
⊲ GL_n(ℝ) ≥ SO(n)

この2つ以外に正規部分群になる組がないということに確証が持てません

820 :132人目の素数さん:2018/08/10(金) 20:57:51.48 ID:IvuMWXPQ.net
>>814
>>799と同じじゃないの?
a^x b a^u b a^y b a^v a^w
とおいて u≡v (mod 2) が必要でそれぞれ
u≡v≡0 (mod 2)のときはx+y+zは任意、
u≡v≡1 (mod 2)のときはy≡0 (mod 2)。
(u,v) = (0,0) → #{(x,y,z)} = 28、
(u,v) = (0,2),(2,0) → #{(x,y,z)} = 15、
(u,v) = (0,4),(2,2),(4,0) → #{(x,y,z)} = 6、
(u,v) = (0,6),(2,4),(4,2),(6,0) → #{(x,y,z)} = 1、
(u,v,y) = (1,1,0) → #{(x,z)} = 5、
(u,v,y) = (1,1,2),(1,3,0),(3,1,0) → #{(x,z)} = 3、
(u,v,y) = (1,1,4),(1,3,2),(3,1,2),(1,5,0),(3,3,0),(5,1,0) → #{(x,z)} = 1、
全部足して100。

821 :132人目の素数さん:2018/08/10(金) 21:16:02.50 ID:9N8PyoKh.net
全=無

ですか?

822 :132人目の素数さん:2018/08/10(金) 21:38:51.06 ID:apZDSISF.net
高校の微積分までを使って解ける統計学の面白い問題はありませんか?

823 :132人目の素数さん:2018/08/10(金) 21:51:04.45 ID:9N8PyoKh.net
全てのプログラミング言語を自由自在に操れるようにすることって可能ですか?

824 :132人目の素数さん:2018/08/10(金) 22:38:41.42 ID:Hlm8Oe3x.net
>>815
100個コンピューターで表示させてみた。
1 b b b b a a a a a a
2 b b b a a b a a a a
3 b b b a a a a b a a
4 b b b a a a a a a b
5 b b a b b a a a a a
6 b b a b a a b a a a
7 b b a b a a a a b a
8 b b a a b b a a a a
9 b b a a b a a b a a
10 b b a a b a a a a b
11 b b a a a b b a a a
12 b b a a a b a a b a
13 b b a a a a b b a a
14 b b a a a a b a a b
15 b b a a a a a b b a
16 b b a a a a a a b b
17 b a b b a b a a a a
18 b a b b a a a b a a
19 b a b b a a a a a b
20 b a b a a b a b a a
21 b a b a a b a a a b
22 b a b a a a a b a b
23 b a a b b b a a a a
24 b a a b b a a b a a
25 b a a b b a a a a b
26 b a a b a b b a a a
27 b a a b a b a a b a
28 b a a b a a b b a a
29 b a a b a a b a a b
30 b a a b a a a b b a
31 b a a b a a a a b b
32 b a a a b b a b a a
33 b a a a b b a a a b
34 b a a a b a a b a b
35 b a a a a b b b a a
36 b a a a a b b a a b
37 b a a a a b a b b a
38 b a a a a b a a b b
39 b a a a a a b b a b
40 b a a a a a a b b b
41 a b b b b a a a a a
42 a b b b a a b a a a
43 a b b b a a a a b a
44 a b b a b b a a a a
45 a b b a b a a b a a
46 a b b a b a a a a b
47 a b b a a b b a a a
48 a b b a a b a a b a
49 a b b a a a b b a a
50 a b b a a a b a a b

825 :132人目の素数さん:2018/08/10(金) 22:38:59.86 ID:Hlm8Oe3x.net
51 a b b a a a a b b a
52 a b b a a a a a b b
53 a b a b b a b a a a
54 a b a b b a a a b a
55 a b a b a a b a b a
56 a b a a b b b a a a
57 a b a a b b a a b a
58 a b a a b a b b a a
59 a b a a b a b a a b
60 a b a a b a a b b a
61 a b a a b a a a b b
62 a b a a a b b a b a
63 a b a a a a b b b a
64 a b a a a a b a b b
65 a a b b b b a a a a
66 a a b b b a a b a a
67 a a b b b a a a a b
68 a a b b a b b a a a
69 a a b b a b a a b a
70 a a b b a a b b a a
71 a a b b a a b a a b
72 a a b b a a a b b a
73 a a b b a a a a b b
74 a a b a b b a b a a
75 a a b a b b a a a b
76 a a b a b a a b a b
77 a a b a a b b b a a
78 a a b a a b b a a b
79 a a b a a b a b b a
80 a a b a a b a a b b
81 a a b a a a b b a b
82 a a b a a a a b b b
83 a a a b b b b a a a
84 a a a b b b a a b a
85 a a a b b a b b a a
86 a a a b b a b a a b
87 a a a b b a a b b a
88 a a a b b a a a b b
89 a a a b a b b a b a
90 a a a b a a b b b a
91 a a a b a a b a b b
92 a a a a b b b b a a
93 a a a a b b b a a b
94 a a a a b b a b b a
95 a a a a b b a a b b
96 a a a a b a b b a b
97 a a a a b a a b b b
98 a a a a a b b b b a
99 a a a a a b b a b b
100 a a a a a a b b b b

826 :132人目の素数さん:2018/08/10(金) 22:40:40.61 ID:Hlm8Oe3x.net
>>822
ゴルゴ13は100発100中
ゴルゴ14は10発10中
ゴルゴ15は1発1中
とする。
各々10000発撃ったとき各ゴルゴの命中数の期待値はいくらか?

ドツボ13は100発0中
ドツボ14は10発0中
ドツボ15は1発0中
とする。
各々10000発撃ったときドツボの命中数の期待値はいくらか?

827 :132人目の素数さん:2018/08/11(土) 01:24:35.12 ID:R/gFC10O.net
>>826

確率はゼロですな

戦時中地上砲火で敵戦闘機を撃墜したのが、数万発で一発ということだった。
地上のテストの評価報告は、>>826 とおなじであった。(要するに当たらない)

それゆえ 弾の無駄だが、敵機も緊張するのか爆弾も外れることが多かった。
数学的説明は上官がわめいていた。

ロシアは優秀なポント。。。何とかという人が、対空砲火オペのりろんつくっていたらしけどね

828 :132人目の素数さん:2018/08/11(土) 04:50:49.64 ID:sk10gcdk.net
「整数の集合は和と積の演算において環になる」
という言い方に違和感があるんですが。
「整数の集合が環になるように、和と積の演算を定義した」
というべきじゃないんですか?

829 :132人目の素数さん:2018/08/11(土) 04:54:39.06 ID:sk10gcdk.net
前者の言い方だとまるで演算が先にあって、それがたまたま環の演算の規則に合致していた
ように聴こえるんですが。実際は、環の演算の規則に合致するように演算を定義したんですよね?

830 :132人目の素数さん:2018/08/11(土) 06:09:29.18 ID:OesSEWnz.net
>>827
3打数1安打と300打数1安打の期待値は違うんじゃ?
最尤値は同じだろうけど。

831 :132人目の素数さん:2018/08/11(土) 06:10:30.49 ID:OesSEWnz.net
>>829
零の零乗とかもそう?

832 :132人目の素数さん:2018/08/11(土) 06:22:41.86 ID:OesSEWnz.net
>>826
命中率の事前確率を一様分布とする という設定がないと計算できない。

833 :132人目の素数さん:2018/08/11(土) 06:26:26.71 ID:OesSEWnz.net
>>822
あるタクシー会社のタクシーには1から通し番号がふられている。

タクシー会社の規模から保有タクシー台数は100台以下とわかっている。

この会社のタクシーを5台みかけた。最大の番号が60であった。

この会社の保有するタクシー台数の期待値は?

834 :132人目の素数さん:2018/08/11(土) 06:33:26.99 ID:/7veEAAF.net
>>828
環っていう概念がないところですでに和と積が定義されているのに?

835 :132人目の素数さん:2018/08/11(土) 06:33:51.37 ID:/7veEAAF.net
>>829
演算が先だよ?

836 :132人目の素数さん:2018/08/11(土) 06:37:27.57 ID:/7veEAAF.net
>>829
たまたま環の定義に合致していたんだけど?
てゆーか
環の概念を
整数やら多項式やら
和と積が定義されているいろいろな集合に
共通するように定義したんだけど?

837 :132人目の素数さん:2018/08/11(土) 07:30:11.28 ID:jvzdrX0f.net
aとbを無理数とし、a<bとする。
このとき、a<c<bなる無理数cが存在することを示せ。

838 :132人目の素数さん:2018/08/11(土) 08:50:37.23 ID:KjzsAEhK.net
(2a+b)/3, (a+2b)/3が有理数ならa,b共に有理数。

839 :132人目の素数さん:2018/08/11(土) 09:07:15.36 ID:O3XHe6Z3.net
>>837
[ 1/(b-a) ] + 1 = n とおく。
b-a > 1/n,
a < m/n < b なる有理数 m/n がある。(mは整数)
 c = (a + m/n)/2,(m/n + b)/2,等など。

>>810
最小値はないが、下限は0
(a,b,x1,x2) = (ε,1-ε,1,0) のとき ε
(a,b,x1,x2) = (1-ε,ε,0,1) のとき ε

840 :132人目の素数さん:2018/08/11(土) 11:59:57.72 ID:fhmCrAJF.net
条件付き確率の問題です。

袋 1 には赤玉 4 個、青玉 6 個、袋 2 には赤玉 5 個、青玉 4 個が入っている。抽選により1つの袋を選び、
その中から玉を1個取り出すとき、それが青玉である確率を求めよ。

(1/2)*(6/10) + (1/2)*(4/9) が答えですが、分からない点があります。

袋 1 が選ばれるという事象を A とする。
青玉が取り出されるという事象を B とする。

P(A ∩ B) = P(A) * P_A(B), P(A) = 1/2, P_A(B) = 6/10 だから乗法定理により

P(A ∩ B) = (1/2) * (6/10)

というような解説を目にします。

ところが、 P_A(B) の定義は、

P_A(B) := P(A ∩ B) / P(A)

です。従って、 P_A(B) を計算するには、 P(A ∩ B), P(A) の値が必要になります。

これは循環論法ではないでしょうか?

P_A(B) を直接何らかの方法で求めているようですが、これはどういうことでしょうか?

841 :132人目の素数さん:2018/08/11(土) 12:42:10.95 ID:nt+CHb9r.net
PA(B)は、Aが起きた時にBが起こる確率です

今の場合、袋1を選択した時、青を選ぶ確率です
6/10ですね

842 :132人目の素数さん:2018/08/11(土) 12:48:11.65 ID:fhmCrAJF.net
>>841

ですが、P_A(B) の定義は、

P_A(B) := P(A ∩ B) / P(A)

です。

ですので、これを計算するには、求める答えである P(A ∩ B) が分からないと計算できないはずです。

843 :132人目の素数さん:2018/08/11(土) 13:06:10.84 ID:2lsAzaWv.net
なるほどなるほど、つまり内積空間における角の定義はcosθ=(略)だから、内積を求めるためにA・B=|A||B|cosθは使えない(使ったら循環論法になる)という主張ですね?

844 :132人目の素数さん:2018/08/11(土) 13:27:49.29 ID:fhmCrAJF.net
>>843

角を

θ := <A, B> / |A|*|B|

で定義するならば、使えないと思います。

845 :132人目の素数さん:2018/08/11(土) 13:28:21.17 ID:fhmCrAJF.net
>>843

角を

cos(θ) := <A, B> / |A|*|B|

で定義するならば、使えないと思います。

846 :132人目の素数さん:2018/08/11(土) 13:29:25.45 ID:fhmCrAJF.net
内積を求めるその公式には何のありがたみもありません。

847 :132人目の素数さん:2018/08/11(土) 13:31:29.99 ID:fhmCrAJF.net
同じように、乗法公式とわざわざ名前の付けられている

P_A(B) := P(A ∩ B) / P(A)

という式には何のありがたみもありません。

848 :132人目の素数さん:2018/08/11(土) 13:37:12.85 ID:fhmCrAJF.net
同じように、乗法公式とわざわざ名前の付けられている

P(A ∩ B) = P(A) * P_A(B)

という式には何のありがたみもありません。

849 :132人目の素数さん:2018/08/11(土) 13:52:01.98 ID:/xq25TtK.net
神様は数学の支配下にあるのでしょうか?

850 :132人目の素数さん:2018/08/11(土) 14:40:35.27 ID:/7veEAAF.net
>>848
>ID:fhmCrAJF
全然ダメダメ

851 :132人目の素数さん:2018/08/11(土) 15:09:52.19 ID:1YQQpFZX.net
>>842
それは定義ですか?
定理ではありませんか?

852 :132人目の素数さん:2018/08/11(土) 15:50:57.24 ID:sTs4VXir.net
劣等感のほうがまだマシだな
それとも劣等感の新ネタか?

853 :132人目の素数さん:2018/08/11(土) 18:01:26.60 ID:3cONG44t.net
複素数平面上に三角形をなす3つの点と対応する複素数、O(0),A(α),B(β)をとる

Oは原点

重心Gをあらわす複素数がα*β/3となるための条件は|α-1|=1であることを証明せよ

という問題なのですがまったく解けません

助けてください

854 :132人目の素数さん:2018/08/11(土) 18:07:28.54 ID:fhmCrAJF.net
>>851

赤いチャート式に載っている「定義」です。

855 :132人目の素数さん:2018/08/11(土) 18:13:58.22 ID:3cONG44t.net
自力で解けました
ありがとうございました

なんで本番でできなかったんだろう・・・・・

856 :132人目の素数さん:2018/08/11(土) 18:15:14.63 ID:4h9sumgz.net
>>653
α=2、β=i のとき|α-1| = 1だけど重心はα*β/3になんかならないけど?

857 :132人目の素数さん:2018/08/11(土) 19:35:18.98 ID:YJB4cadW.net
物質が何も無い無限大の空間で自分一人だけ永遠にポツンと存在し続けたらどうなるのでしょうか?

858 :132人目の素数さん:2018/08/11(土) 20:13:16.75 ID:O/h5+IQW.net
>>840
AとBが独立事象じゃないんじゃない?

P(A ∩ B) = P(A) * P_A(B) ≠ P(A)*P(B)

859 :132人目の素数さん:2018/08/11(土) 20:15:38.01 ID:O/h5+IQW.net
A-Bag : Red-4 Blue-6
nonA-Bag : Red-5 Blue-4

A: picking A-Bag
B: picking Blue

P_Y(X)=P(X|Y)

P(A)=1/2
P(¬A)=1/2

P(A∩B)=P(A)P(B|A) = 1/2*6/10 = 0.3
= P(B)P(A|B)

P(B) = P(B|A)P(A) + P(B|¬A)P(¬A)= (1/2)*(6/10) + (1/2)*(4/9)=47/90
P(A)P(B)=1/2*47/90=47/180=0.2611111

P(A|B)=P(A)P(B|A)/P(B)
=P(A)P(B|A)/{P(A)P(B|A) + P(¬A)P(B|¬A)}
=(1/2*6/10) / (1/2*6/10 + 1/2*4/9)
= 27/47

860 :132人目の素数さん:2018/08/11(土) 20:28:09.48 ID:otNCH/Gz.net
>>854
当該ページを見せてよ

861 :132人目の素数さん:2018/08/11(土) 20:38:40.80 ID:fhmCrAJF.net
>>860

赤いチャート式に限らず、普通の高校の教科書でもこの定義は書いてあるかと思います。

862 :132人目の素数さん:2018/08/11(土) 21:55:41.21 ID:3t3xr1cT.net
ax^2+bxy+cy^2 (a,b,c:整数)はb^2-4acが平方数のとき有理数係数の1次式の積に書けることを示して下さい

863 :132人目の素数さん:2018/08/11(土) 22:09:28.21 ID:pW3k6Y87.net
a=b=c=0

864 :132人目の素数さん:2018/08/11(土) 22:17:08.72 ID:YJB4cadW.net
今、二項定理の勉強をしていて、疑問に思ったことがあるので質問します。
http://o.8ch.net/18h5y.png

865 :132人目の素数さん:2018/08/11(土) 22:20:38.32 ID:pW3k6Y87.net
これビットマップで描いたの?

866 :132人目の素数さん:2018/08/11(土) 22:20:55.92 ID:YJB4cadW.net
もう少し丁寧に書きます。
http://o.8ch.net/18h66.png

867 :132人目の素数さん:2018/08/11(土) 22:23:10.16 ID:YJB4cadW.net
参考書をもう一度読み返したら分かったっぽいです。

868 :132人目の素数さん:2018/08/11(土) 22:24:50.93 ID:YJB4cadW.net

http://o.8ch.net/18h6c.png

869 :132人目の素数さん:2018/08/11(土) 22:25:45.95 ID:YJB4cadW.net

http://o.8ch.net/18h6f.png

870 :132人目の素数さん:2018/08/11(土) 22:28:30.27 ID:YJB4cadW.net

http://o.8ch.net/18h6p.png

871 :132人目の素数さん:2018/08/11(土) 22:29:13.63 ID:pW3k6Y87.net
夏本番って感じでいいね

872 :132人目の素数さん:2018/08/11(土) 22:31:30.76 ID:YJB4cadW.net

http://o.8ch.net/18h6u.png

873 :132人目の素数さん:2018/08/11(土) 22:33:14.06 ID:YJB4cadW.net

http://o.8ch.net/18h6y.png

874 :132人目の素数さん:2018/08/11(土) 22:36:01.65 ID:YJB4cadW.net

http://o.8ch.net/18h75.png

875 :132人目の素数さん:2018/08/11(土) 22:39:30.70 ID:YJB4cadW.net

http://o.8ch.net/18h7c.png

876 :132人目の素数さん:2018/08/11(土) 22:42:24.25 ID:/7veEAAF.net
>>854
定義であろうが定理であろうが
成立する等式な訳でね

877 :132人目の素数さん:2018/08/11(土) 22:42:35.41 ID:YJB4cadW.net

http://o.8ch.net/18h7p.png

878 :132人目の素数さん:2018/08/11(土) 22:44:15.13 ID:YJB4cadW.net

http://o.8ch.net/18h80.png

879 :132人目の素数さん:2018/08/11(土) 22:45:48.73 ID:YJB4cadW.net

http://o.8ch.net/18h87.png

880 :132人目の素数さん:2018/08/11(土) 22:46:53.69 ID:YJB4cadW.net

http://o.8ch.net/18h8e.png

881 :132人目の素数さん:2018/08/11(土) 22:48:53.57 ID:YJB4cadW.net

http://o.8ch.net/18h8l.png

882 :132人目の素数さん:2018/08/12(日) 00:09:03.60 ID:qDsr7SqG.net
>>861
どこにでも書いてある式なのは確かだが
定義と書いてあったかね?

883 :132人目の素数さん:2018/08/12(日) 00:12:20.85 ID:1ANJkydr.net
画像を挙げられないのなら本のISBNおよびページ数を書け
明日本屋で見てきてやる

884 :132人目の素数さん:2018/08/12(日) 00:28:09.37 ID:5SbEEqNY.net
お忙しいところ恐縮ですが
http://o.8ch.net/18hfh.png

885 :132人目の素数さん:2018/08/12(日) 00:30:13.60 ID:QY55nO4t.net

http://o.8ch.net/18hfm.png

886 :132人目の素数さん:2018/08/12(日) 00:31:31.94 ID:QY55nO4t.net

http://o.8ch.net/18hfq.png

887 :132人目の素数さん:2018/08/12(日) 00:39:14.59 ID:iGIEhgMa.net
地元の銭湯が根こそぎターゲットになってて、どこの銭湯に行っても動画で見覚えがあるぞw

888 :132人目の素数さん:2018/08/12(日) 00:44:40.70 ID:ePffXMZT.net
なんの話だよ
ホモ動画サイトか?

889 :132人目の素数さん:2018/08/12(日) 00:47:26.96 ID:QY55nO4t.net
天上神とオムニバースはどっちの方が凄いですか?

890 :132人目の素数さん:2018/08/12(日) 00:55:02.29 ID:fj2vnC7N.net
>>889
俺の次に凄い

891 :132人目の素数さん:2018/08/12(日) 00:58:19.39 ID:QY55nO4t.net
マキシム・コンツェビッチとハーバード大学首席合格者とウィリアム・ジェイムズ・サイディズは誰が一番頭が良いですか?

892 :132人目の素数さん:2018/08/12(日) 01:03:53.88 ID:QY55nO4t.net
アルキメデスとウィリアム・ジェイムズ・サイディズはどっちの方が頭が良いですか?

893 :132人目の素数さん:2018/08/12(日) 01:12:22.64 ID:QnRFj99l.net
>>862
bb-4ac = dd とおく。
bとdの奇偶は同じだから、(b±d)/2 = k,L は整数。
k + L = b,
kL = (bb-dd)/4 = ac,

a≠0 のとき (ax+ky)(ax+Ly)/a,
a=0 のとき (bx+cy)y,
あるいは
c≠0 のとき (kx+cy)(Lx+cy)/c,
c=0 のとき x(ax+by),

894 :132人目の素数さん:2018/08/12(日) 01:35:23.78 ID:6yX/ZJW6.net
>>893
ありがとうございます

895 :132人目の素数さん:2018/08/12(日) 01:52:38.18 ID:fj2vnC7N.net
いえ、どういたしまして

896 :132人目の素数さん:2018/08/12(日) 01:57:37.06 ID:6GKJfetr.net
ax^2+bxy+cy^2=p(a,b,c:整数、p:非負整数)はb^2-4acの値によりどのような曲線となるか。

897 :132人目の素数さん:2018/08/12(日) 03:21:48.52 ID:QnRFj99l.net
>>887 >>892
地元にアルキメデスがいて Eureka!を期待してんぢゃね?

898 :132人目の素数さん:2018/08/12(日) 03:22:24.55 ID:QnRFj99l.net
>>887 >>892
地元にアルキメデスがいて Eureka!を期待してんぢゃね?

899 :132人目の素数さん:2018/08/12(日) 03:31:33.39 ID:6GKJfetr.net
x,y,zは自然数で、そのいずれか1つは素数である。このとき、
(y^2)/{x(1-y)(z-x)}
が整数となるための条件を求めよ。

900 :132人目の素数さん:2018/08/12(日) 10:05:27.52 ID:HVefTO+M.net
(y^2,y-1) = 1よりy=2。
x(z-x) = 1,2,4より
(1,2,2),(1,2,3),(1,2,5),(2,2,1),(2,2,3),(2,2,4),(4,2,3),(4,2,5)。

901 :132人目の素数さん:2018/08/12(日) 10:08:13.81 ID:HVefTO+M.net
>>900
(y^2,y-1) = 1、(y-1)|y^2よりy=2。
x(z-x) = 1,2,4より
(1,2,2),(1,2,3),(1,2,5),(2,2,1),(2,2,3),(2,2,4),(4,2,3),(4,2,5)。

902 :132人目の素数さん:2018/08/12(日) 10:34:49.28 ID:lwgDlbqX.net
A ⊂ R^m
f : A → R
∂f/∂x_1, ∂f/∂x_2, ∂f/∂x_m が存在し、有界

とする。

このとき、 f は全微分可能であることを示せ。

903 :132人目の素数さん:2018/08/12(日) 10:35:31.91 ID:DDIZULYD.net
嫌です

904 :132人目の素数さん:2018/08/12(日) 11:09:53.35 ID:FG0t7/CX.net
f+sin(r)sin(4θ) (r≠0), 0(r=0)
において
∂/∂x=cosθ∂/∂r−sinθ/r∂/∂x
∂/∂y=sinθ∂/∂r+cosθ/r∂/∂θ
より∂f/∂x. ∂f/∂yは存在して有界。
しかし原点で全微分可能でない。

905 :132人目の素数さん:2018/08/12(日) 11:32:36.74 ID:xJJIoznm.net
稠密な集合上連続な関数は、その閉包で連続な関数に一意に延長できることの証明ってどこに載っているでしょうか?

906 :132人目の素数さん:2018/08/12(日) 16:01:26.37 ID:lwgDlbqX.net
>>904

ありがとうございます。


A ⊂ R^m
f : A → R
∂f/∂x_1, ∂f/∂x_2, ∂f/∂x_m が存在し、有界

とする。

このとき、 f は連続であることを示せ。

907 :132人目の素数さん:2018/08/12(日) 16:31:24.97 ID:J1D7nioX.net
Apple(アメリカにある本社)に就職して、
新型iPhoneやiPadを開発するには、大学で何を専攻した方が良いのでしょうか?
電気電子工学とかコンピュータ科学とか数学とか機械工学とか物理学とかですか?

908 :132人目の素数さん:2018/08/12(日) 18:04:24.71 ID:J1D7nioX.net
誰か二項定理を教えてください。お願いします。

909 :132人目の素数さん:2018/08/12(日) 18:49:47.12 ID:J1D7nioX.net
テスト。

910 :132人目の素数さん:2018/08/12(日) 19:13:07.82 ID:ZANWutS0.net
ヒマラヤさんは何年二項定理がわからないんですか?

911 :132人目の素数さん:2018/08/12(日) 19:22:34.40 ID:ns/pkk0J.net
>>908
Σ(a+b)^n=Σ(i,j)a^ib^j

912 :132人目の素数さん:2018/08/12(日) 19:31:34.80 ID:J1D7nioX.net
>>911
何それ?

913 :132人目の素数さん:2018/08/12(日) 22:45:51.75 ID:5SbEEqNY.net
>>905

自分で証明できないの?
じゃ だめだ

914 :132人目の素数さん:2018/08/13(月) 08:20:38.04 ID:oNr4PWVz.net
数列の圧縮の理論的限界ってあるんですか?

915 :132人目の素数さん:2018/08/13(月) 08:22:18.33 ID:qdWJ+OM4.net
エントロピー?

916 :132人目の素数さん:2018/08/13(月) 08:26:02.41 ID:AgeFDqH3.net
>>912
二項定理

917 :132人目の素数さん:2018/08/13(月) 09:02:06.18 ID:vi9rwATJ.net
有理関数体の次元は可算でしょうか?非可算でしょうか?
またどのような基底がとれますか?

918 :132人目の素数さん:2018/08/13(月) 11:24:12.15 ID:ZQRgNv1f.net
それくらい即座に分かれ

919 :132人目の素数さん:2018/08/13(月) 12:16:08.84 ID:Rnbq7Gne.net
この問題を解いて欲しいっす

➖➖➖問題➖➖➖➖➖➖➖➖➖➖➖➖➖
実数x,yの方程式
a * x^s + b * y^t = 0
が特異点をもたないための実数a,bの必要十分条件を求めよ。
なお s, t も実数とする。

補足
見辛いですが、累乗の指数s はxのみ、tはyのみに掛かっています。
➖➖➖➖➖➖➖➖➖➖➖➖➖➖➖➖➖➖

920 :132人目の素数さん:2018/08/13(月) 12:19:11.57 ID:fSolFrxP.net
(f(z))^2 = z for all z ∈ {z ∈ C | |z| = 1}

となるような複素関数を f は不連続であることを証明せよ。

921 :132人目の素数さん:2018/08/13(月) 12:22:53.35 ID:gWLs6hnh.net
特異点(とくいてん、英: singularity)とは、ある基準 (regulation) の下、
その基準が適用できない (singular) 点である。
したがって、特異点は基準があって初めて認識され、「—に於ける特異点」「—に関する特異点」という呼ばれ方をする。
特異点という言葉は、数学と物理学の両方で用いられる。

922 :132人目の素数さん:2018/08/13(月) 13:25:44.47 ID:GERIo4qO.net
>>920
fが連続のときg(z) = f(z)^2として
H(f): H_1(S^1) → H_1(S^1)がm倍写像ならH(g)は2m倍写像。
それが恒等写像となることはない。

923 :132人目の素数さん:2018/08/13(月) 13:46:14.79 ID:Rnbq7Gne.net
>>919
特異点とは尖点と自己交差点のことッス。

924 :132人目の素数さん:2018/08/13(月) 13:52:30.52 ID:2jT63fTE.net
方程式の尖点、自己交差点とは、はてさて

925 :132人目の素数さん:2018/08/13(月) 14:16:47.26 ID:QY0gs/b9.net
問題の設定がメチャクチャやん。
(-2)^πとかどうすんねん。

926 :132人目の素数さん:2018/08/13(月) 16:21:57.42 ID:AgeFDqH3.net
>>923
孤立点もな

927 :132人目の素数さん:2018/08/13(月) 16:35:46.00 ID:HLcmiwIv.net
これらが全て分かりません。
誰か途中式と答えを教えてください。

https://m.imgur.com/kpD68LW

928 :132人目の素数さん:2018/08/13(月) 17:40:43.98 ID:c6T1rAtc.net
>>927
物理板

929 :132人目の素数さん:2018/08/13(月) 17:43:15.61 ID:HLcmiwIv.net
超天才数学者になりたい。

930 :132人目の素数さん:2018/08/13(月) 17:43:51.39 ID:c6T1rAtc.net
代数的数と超越数の和は常に無理数であるか。

931 :132人目の素数さん:2018/08/13(月) 18:00:36.60 ID:yi1f/u5u.net
定義から明らかでないかい

932 :132人目の素数さん:2018/08/13(月) 18:31:33.12 ID:AgeFDqH3.net
>>917
有理関数体上1次元

933 :132人目の素数さん:2018/08/13(月) 20:35:04.06 ID:fSolFrxP.net
順序集合 A が least upper bound property をもつならば A は greatest lower bound property をもつことを示せ。

934 :132人目の素数さん:2018/08/13(月) 20:47:25.61 ID:fSolFrxP.net
順序集合 A が least upper bound property をもつならば A は greatest lower bound property をもつことを示せ。

S を空でない下に有界な A の部分集合とする。
L を S のすべての下界からなる集合とする。
S は下に有界だから、 L は空ではない。
s を S の任意の元とする。
L の任意の元を l とすると、 l は S の下界だから、 l ≦ s が成り立つ。
∴ s は L の上界である。

以上より、 L は空でなく上に有界である。よって、 least upper bound property により、最小上界 x が存在する。

上で示したように、 S ∋ s とすると、 s は L の上界である。 x は L の最小上界だから、
x ≦ s

∴ x は S の下界である。
∴ x ∈ L

x は L の上界だから、 L の任意の元 l に対し、 l ≦ x が成り立つ。

∴ x は L の最大の元である。

935 :132人目の素数さん:2018/08/13(月) 22:09:38.27 ID:ga/l3GQB.net
2/sinA=√6/sin120゜の左辺と右辺にどのようなことをすれば
SinA=~の形になりますか

936 :132人目の素数さん:2018/08/13(月) 23:32:53.16 ID:AgeFDqH3.net
>>934
Z+{∞}

937 :132人目の素数さん:2018/08/14(火) 06:42:48.59 ID:nCq8AFJk.net
流体の運動から考えると初期値と弱解が俺の意味で小
さい場合にエネルギー保存則からは自明な俺の不等式
を数学的な仮定すれば後は弱解の可微分性を証明する
だけでミレニアム問題を半分解決できる。俺の意味で
小さいという仮定は流体の運動エネルギーが有限とい
う仮定と同値だから俺の意味で小さい弱解はもはや小
さくないだろうと思う。@reviewer_amzn_m

938 :132人目の素数さん:2018/08/14(火) 06:44:56.75 ID:nCq8AFJk.net
数学でわかるのは数学の話だけだと言う人がいるけど
単に数学の経験不足だろう

例えば数学を本気で長い間学べば身の回りの論理につ
いて不備や仕組みがよくわかるようになる

物理学の理解も深まる

939 :132人目の素数さん:2018/08/14(火) 09:12:11.39 ID:C0s6zS96.net
>>935
両辺を2で割って両辺を逆数にするといいです(分母分子をひっくり返す)

940 :132人目の素数さん:2018/08/14(火) 09:13:00.24 ID:mNEX2sty.net
70億人でじゃんけんして1人だけ勝つ確率

941 :132人目の素数さん:2018/08/14(火) 09:56:38.99 ID:ACVOgXMK.net
(1/3)^699999999

942 :132人目の素数さん:2018/08/14(火) 09:57:43.25 ID:ACVOgXMK.net
700000000(1/3)^699999999 orz

943 :132人目の素数さん:2018/08/14(火) 12:09:11.81 ID:7Ak8Eky0.net
S = {(x, y) | y = x + 1, 0 < x < 2}

S を含むような最小の R 上の同値関係 T を求めよ。

944 :132人目の素数さん:2018/08/14(火) 12:25:14.99 ID:vXAVUkp6.net
背理法を使わないと証明できないことが証明されている命題ってあるのでしょうか?

945 :132人目の素数さん:2018/08/14(火) 13:00:56.91 ID:2L4MQPbj.net
確率の質問です
A,Bの2人の誕生日(月日のみで年は関係がない)が同じになる確率はいくらか。
但し簡単の為閏年は4年に1度あるものとし、その他の暦は現在のものを採用する。
難問ですので分かる範囲で教えてください
それと、
(365×3/365×4+1)×1/365
+
(365+1/365×4+1)×1/366
は違うでしょうか?
条件が少ないと感じていますが、もしそうでもこの確率の表すところは分かると思うのでよろしくお願いします。

946 :132人目の素数さん:2018/08/14(火) 13:16:29.05 ID:uBc/Ymgt.net
>>945
Aの誕生日が2/29以外で、Bもその日になる確率は
{(365×4)/(365×4+1)}×{4/(365×4+1)}

AもBも誕生日が2/29となる確率は
{1/(365×4+1)}^2

947 :132人目の素数さん:2018/08/14(火) 13:17:33.96 ID:uBc/Ymgt.net
>>944
無いっていうか
背理法は背理法を使わない表現に書き直せるはず

948 :132人目の素数さん:2018/08/14(火) 13:20:13.02 ID:2L4MQPbj.net
>>946
自分もそう思ったんですが4年1周期だけで考えていいのかが分からなくて...

949 :132人目の素数さん:2018/08/14(火) 13:20:35.48 ID:2L4MQPbj.net
普通に1年365日で考えるときは1年1周期で考えるからそれと同じことか

950 :132人目の素数さん:2018/08/14(火) 13:21:22.31 ID:2L4MQPbj.net
因みに1番初めに書いた式は何を表していてどこが間違ってますかね?

951 :132人目の素数さん:2018/08/14(火) 13:25:56.22 ID:uBc/Ymgt.net
>>948
うるう年になったら1日の確率は1/366
それ以外は 1/365だから
4年1周期で考えないと
1日の重みが変ってしまい、修正が面倒
どの日も同じ重みで考えるためには
4年まとめるのが簡単だろう

952 :132人目の素数さん:2018/08/14(火) 14:11:38.93 ID:hy3L2E4X.net
年と言うより日で考えたい派

953 :132人目の素数さん:2018/08/14(火) 14:52:12.68 ID:JH2oN9kP.net
ノルム空間Vにおいて三角不等式
| ||x||−||y|| |≦||x−y|| ( ∀x, y∈V )
を示せ.

宜しくお願い致します。

954 :132人目の素数さん:2018/08/14(火) 14:54:50.20 ID:IA4dHF2A.net
>>951
西暦が4の倍数の年はうるう年とする。
・ただし、4の倍数であっても100の倍数の年は平年とする。
・ただし、100の倍数であっても400の倍数の年はうるう年とする。

365+(100-4+1)/400=365.2425

955 :132人目の素数さん:2018/08/14(火) 16:02:41.64 ID:E5Co1yJF.net
自分は尋常じゃないくらい頭が悪いのですが、超猛烈に努力を積み重ねていけば、
グレゴリー・ペレルマンさんやマキシム・コンツェビッチさんみたいな超絶の天才になれるのでしょうか?

956 :132人目の素数さん:2018/08/14(火) 17:22:36.88 ID:zSBdKb8r.net
できます。
頑張ってください。

957 :132人目の素数さん:2018/08/14(火) 17:34:57.47 ID:YemIKJ2g.net
>>954
>但し簡単の為閏年は4年に1度あるものとし、

958 :132人目の素数さん:2018/08/14(火) 18:57:58.91 ID:E5Co1yJF.net
全知全能の存在が無限の無限乗の無限乗の無限乗の無限乗の無限乗・・・・・(これが無限回続く)
以上居て、ガチで戦ったらどうなるのでしょうか?

959 :132人目の素数さん:2018/08/14(火) 23:20:34.15 ID:EV0NsKgJ.net
>>939
ありがとうございます

960 :132人目の素数さん:2018/08/15(水) 01:55:12.46 ID:HKfY+w2Q.net
>>953

x = (x-y) + y,

||x||^2 = (x,x)
 = (x-y,x-y) + (x-y,y) + (y,x-y) + (y,y)
 = ||x-y||^2 + (x-y,y) + (y,x-y) + ||y||^2
 ≦ ||x-y||^2 + ||x-y||・||y|| + ||y||・||x-y|| + ||y||^2
 = (||x-y|| + ||y||)^2,

 ||x|| - ||y|| ≦ ||x-y||,
xとyを入れ替えると
 ||y|| - ||x|| ≦ ||y-x|| = ||x-y||,

961 :132人目の素数さん:2018/08/15(水) 02:38:48.29 ID:6Xw6APxq.net
全ての原始ピタゴラスが表せることの証明について質問します。

方針:以下の順番で証明していきます。
1:a と b のどちらか一方のみ奇数で他方は偶数。 c は奇数である。
2:(c+a)/2,(c−a)/2 はともに平方数である。
3:公式の導出

このうち、2を背理法で証明する解説を読み、わからない点があります。

2の証明
(c+a)/2 と(c−a)/2 のどちらか一方でも平方数でないとすると,それらの積は平方数であるので,
(c+a)/2 と (c−a)/2 は共通因数 p≥2 を持つ。
つまり,c+a=2up,c−a=2vp ただし u,v は自然数,と書ける。
これを a,c について解くと,
a=(u−v)p,c=(u+v)p
よって,a,c はともに p の倍数となり,さらに b も p の倍数となるので原始ピタゴラス数であるという仮定に矛盾。

上記の「(c+a)/2 と (c−a)/2 は共通因数 p≥2 を持つ」という部分が理解できません。
どなたかわかりやすいように解説をお願いします。

962 :132人目の素数さん:2018/08/15(水) 06:54:03.79 ID:SQV3T5wE.net
S = {(x, y) | y = x + 1, 0 < x < 2}

S を含むような最小の R 上の同値関係 T を求めよ。

963 :132人目の素数さん:2018/08/15(水) 07:54:55.61 ID:Hyxd+nJo.net
>>961
1から、aが奇数で b が偶数と仮定していて
(c+a) と (c-a) も偶数

(c+a)/2 の各素因数 p の次数に注目して
p^(2n) = (p^n)^2
p(2n+1) = {(p^n)^2} p
という変換を行えば

(c+a)/2 = A^2 M
(c-a)/2 = B^2 N
の形に書ける。ただし M, Nは 1次の素数の積
(c+a)/2, (c-a)/2 の少なくとも一方が平方数でないなら
M,N の少なくとも一方は 1ではない

(AB)^2 MN = {(c+a)/2} {(c-a)/2} = (b/2)^2
なので、MN は平方数であり、M,N は同じ素因数を持たなければならず M = N ( > 1 )
すなわち、(c+a)/2, (c-a)/2 は(2以上の)共通の因数Mを持つ

964 :132人目の素数さん:2018/08/15(水) 11:59:28.69 ID:p/Nzh/yc.net
>>962
a〜b ⇔ 0<a<2 b = a+1 or 0<b<2 a = b+1 or a=b

965 :132人目の素数さん:2018/08/15(水) 13:12:35.51 ID:p/Nzh/yc.net
>>964
>>>962
>a〜b ⇔ 0<a<2 b = a+1 or 0<a<1 b = a+2 or 0<b<2 a = b+1 or 0<b<1 a = b+2 or a=b

966 :132人目の素数さん:2018/08/15(水) 14:18:51.23 ID:bGX6pl5F.net
コンパクトリーマン面は必ず射影空間に埋め込めますか?

967 :132人目の素数さん:2018/08/15(水) 15:48:28.81 ID:ArxgJrwy.net
レオンハルト・オイラーと量子コンピュータはどっちの方が賢いですか?

968 :132人目の素数さん:2018/08/15(水) 16:53:37.18 ID:SQV3T5wE.net
Theorem 7.4.

Let A be open in R^n; let f : A -> R^n; let f(a) = b.
Suppose that g maps a neighborhood of b into R^n, that g(b) = a, and

g(f(x)) = x

for all x in a neighborhood of a. If f is differentiable at a and if g is differentiable at b, then

Dg(b) = [Df(a)]^(-1).

↑は、 James R. Munkres著『Analysis on Manifolds』に書いてある定理です。

なぜ↓のように書かなかったのでしょうか?

Let A be open in R^n.
Let f : A -> R^n.
Let B be open in R^n.
Let g : B -> R^n.
Let a ∈ A.
Let b ∈ B.
Let f(a) = b.
Let f be differentiable at a.
Let g be differentiable at b.
Let g(f(x)) = x for all x in a neighborhood of a.

Then,

Dg(b) = [Df(a)]^(-1).

969 :132人目の素数さん:2018/08/15(水) 16:54:50.76 ID:Txdk00jN.net
バカっぽいからです

970 :132人目の素数さん:2018/08/15(水) 21:39:33.27 ID:7wLXI4Pw.net
>>969
斜め上の迷答だなw

971 :132人目の素数さん:2018/08/15(水) 21:54:35.86 ID:7wLXI4Pw.net
{ [If f is differentiable at a] and [if g is differentiable at b] },
then
[ Dg(b) = [Df(a)]^(-1) ].

{ P and Q} , then R. と言ってるのに、>>968 の下の書き方はand条件部分が異なってるだろ。
>なんか Munkres さんの本を読んだ後に杉浦光夫著『解析入門I』を読むと非常に優しい本であると感じますね。
この読み方でよく言えるねw

972 :132人目の素数さん:2018/08/15(水) 22:13:04.18 ID:7wLXI4Pw.net
Suppose {that g maps a neighborhood of b into R^n }, [that { g(b) = a }, and
{ g(f(x)) = x }
for all x in a neighborhood of a ].
2番目のThat節は、1番目のThat節中のR^n に掛かっているのに分かっていないだろ。本当にバカだな。

973 :132人目の素数さん:2018/08/15(水) 22:33:22.61 ID:vEIGFrFN.net
Q.数学を学ぶ利点は何か?

974 :132人目の素数さん:2018/08/15(水) 22:58:11.01 ID:36LGswby.net
そのような無益な問いに悩まされることがなくなること。

975 :132人目の素数さん:2018/08/15(水) 23:08:27.51 ID:um9UF8tj.net
分からない問題はここに書いてね446
https://rio2016.5ch.net/test/read.cgi/math/1534342085/

976 :132人目の素数さん:2018/08/16(木) 09:59:55.66 ID:5SAQATYI.net
>>963
ありがとうございます
ただ僕は中学生なので、恥ずかしながら、書いていただいた内容がまだ完全に理解しきれていません
また質問するかもしれませんが、その時はよろしくお願いします

977 :132人目の素数さん:2018/08/16(木) 13:15:05.05 ID:Tp/l7Aeb.net
> ID:7wLXI4Pw
別に
>>968
の下の書き方で良いよ

978 :132人目の素数さん:2018/08/16(木) 13:16:31.74 ID:Tp/l7Aeb.net
>>969
正解

979 :132人目の素数さん:2018/08/16(木) 18:18:20.34 ID:GbAIDwkg.net
X ⊂ Y ⊂ Z を距離空間とする。

以下を示せ。

(1)
X が Y の開集合
Y が Z の開集合



X は Z の開集合

(2)
X が Y の開集合
X が Z の開集合



Y は Z の開集合

(3)
X が Z の開集合
Y が Z の開集合



X は Y の開集合

980 :132人目の素数さん:2018/08/16(木) 18:42:39.92 ID:GbAIDwkg.net
X ⊂ Y ⊂ Z を距離空間とする。

以下を示せ。

(1)
X が Y の開集合
Y が Z の開集合



X は Z の開集合

(2)
X が Z の開集合



X は Y の開集合

981 :132人目の素数さん:2018/08/16(木) 20:21:08.53 ID:ALKOqWVk.net
頑張って超いっぱい勉強して、東京大学理科I類でも受験しようかな。
そして、最も難しい学問である数学を専攻しようかな。

982 :高添沼田の親父「糞関東連合テメエらまとめてぶち殺すっ!!」:2018/08/16(木) 21:17:46.97 ID:dZ5ratnn.net
高添沼田(葛飾区青戸6−23−21ハイツニュー青戸103号室)の挑発
高添沼田の親父「関東連合文句があったらいつでも孫を金属バットで殴り殺しに来やがれっ!! 関東連合の糞野郎どもは俺様がぶちのめしてやるぜっ!! 賞金をやるからいつでもかかって来いっ!!糞バエ関東連合どもっ!! 待ってるぜっ!!」 (挑戦状)

983 :132人目の素数さん:2018/08/17(金) 01:56:03.09 ID:DsWMw13x.net
この問題の解き方と解答を教えてもらいたい

P、Q、R、S、T、Uの6人が円形のテーブルのまわりに座らせる。
テーブルの席には番号が振られてある場合
P、Qが隣り合わせになるような座り方は何通りか?

984 :132人目の素数さん:2018/08/17(金) 04:42:24.03 ID:5QyvDwxU.net
>>983
n=6 とおく。

{P,Q} の席 … nとおり
P,とQの入替 … 2 とおり
他の席の入替 … (n-2)! とおり

2n・(n-2)! = 288

985 :132人目の素数さん:2018/08/17(金) 09:58:38.96 ID:Xs+I9BdE.net
こんな応用問題とか
P、Q、R、S、T、Uの文字の書かれたビーズでブレスレッドを作る。回転させたりひっくり返して同じになるブレスレッドは1種類と数える。
何種類のブレスレッドが作れるか?

P、Q、Q、R、R、Rでは何種類か?

尚、ちょっと思いついただけで
正解は準備してないので、悪しからず。

986 :132人目の素数さん:2018/08/17(金) 11:38:24.59 ID:xYp1uw0D.net
仏になるのとリーマン予想を証明するのはどっちの方が難しいですか?

987 :132人目の素数さん:2018/08/17(金) 12:56:23.44 ID:aWr8etgk.net
高校数学のデータの分析のところで質問です。下の画像の問題は、データ修正前も修正後も共分散がともに0ではないのですか?
もしそうなら国語と数学の相関係数も修正前にしろ後にしろ0にならないのですか?

https://i.imgur.com/6CnxCSA.jpg

988 :132人目の素数さん:2018/08/17(金) 13:13:02.39 ID:nzH46HUP.net
>>987
全部問題の下の解説に書いてある
問題も含めて10回読み直せ

989 :132人目の素数さん:2018/08/17(金) 19:30:23.36 ID:ZX0wk38j.net
ホトケニナルノガイイ

990 :132人目の素数さん:2018/08/18(土) 18:46:08.84 ID:+ZAzv04a.net
>>987
4人だけの共分散は0
問われているのは全員での共分散。

991 :132人目の素数さん:2018/08/18(土) 21:12:36.78 ID:DegCYDqX.net
4.
Let g : R^2 -> R^2 be given by the equation

g(x, y) = (2*y*e^(2*x), x*e^y).

Let f : R^2 -> R^3 be given by the equation

f(x, y) = (3*x - y, 2*x + y, x*y + y^3).

(a) Show that there is a neighborhood of (0, 1) that g carries in a one-to-one fashion onto a neighborhood of (2, 0).

(b) Find D(f 〇 g^(-1)) at (2. 0).

992 :132人目の素数さん:2018/08/18(土) 21:24:54.21 ID:nZNQvP8k.net
この問題を解く上で、解けない連立方程式が現れました。
このアプローチは間違っていないですがなぜ解けないのでしょうか?
https://i.imgur.com/fJJ5A05.jpg

993 :132人目の素数さん:2018/08/18(土) 21:33:06.49 ID:LEs4WroI.net
>>992
式の本数(3本あるように見えるが実は2本しかない)と未知数の個数があってないので一意には決まらない
y,z を x で表して x をパラメータと思う

994 :132人目の素数さん:2018/08/18(土) 22:28:23.22 ID:DegCYDqX.net
>>991

(a)

Dg(x, y) = { {4*y*e^(2*x), 2*e^(2*x)}, {e^y, x*e^y} }

Dg(0, 1) = { {4, 2}, {e, 0} }

det(Dg(0, 1)) = -2*e ≠ 0

逆関数定理により、 (a) が成り立つ。

995 :132人目の素数さん:2018/08/18(土) 22:43:29.14 ID:DegCYDqX.net
>>991

(b)

(-1/(2*e)) * { {e, -10}, {-e, 0}, {-e, 4} }

996 :132人目の素数さん:2018/08/18(土) 23:12:04.74 ID:Mm94oxYG.net
どうしてcosの値が出るのかわかりません。
ちなみに、QP=2でRのy座標が5√3/6です。
https://i.imgur.com/plDMfUa.jpg

997 :132人目の素数さん:2018/08/18(土) 23:17:55.14 ID:LEs4WroI.net
>>996
円の半径か放物線の x^2 の係数は?

998 :132人目の素数さん:2018/08/19(日) 11:00:34.69 ID:ny+9RsJV.net
線対称性のある図形で
「回転対称性なし」かつ「対称軸が複数本」という
条件を満たす図形ってあり得ますでしょうかね

999 :132人目の素数さん:2018/08/19(日) 11:17:50.82 ID:gT6uuqxK.net
対称軸が2本のとき
回転対称性なし⇔対称軸が全部平行

1000 :132人目の素数さん:2018/08/19(日) 11:19:00.70 ID:euxGpNnr.net
1000

1001 :2ch.net投稿限界:Over 1000 Thread
2ch.netからのレス数が1000に到達しました。

総レス数 1001
279 KB
掲示板に戻る 全部 前100 次100 最新50
read.cgi ver 2014.07.20.01.SC 2014/07/20 D ★